Pathology for the Dental Hygienist 7th Edition by Ibsen TEST BANK. cHAPTERS 1-10 IN 229 Pages.

Page 1


Chapter 01: Introduction to Preliminary Diagnosis of Oral Lesions Ibsen: Oral Pathology for the Dental Hygienist, 7th Edition MULTIPLE CHOICE 1. Which descriptive term is described as a segment that is part of the whole? a. Bulla b. Vesicle c. Lobule d. Pustule ANS: C

A lobule is described as a segment or lobe that is part of a whole. A bulla is a large, elevated lesion that contains serous fluid and may look like a blister. A vesicle is a small, elevated lesion that contains serous fluid. Pustules are circumscribed elevations containing pus. REF: Vocabulary, Clinical of Soft Tissue Lesions, page 1

OBJ: 1

2. A lesion with a sessile base is described as a. an ulcer. b. stemlike. c. pedunculated. d. flat and broad. ANS: D

Sessile describes the base of a lesion that is flat and broad. An ulcer is a break in the surface epithelium. A stemlike lesion is referred to as pedunculated. A pedunculated lesion is stemlike or stalk-based (similar to a mushroom). REF: Vocabulary, Clinical Appearance of Soft Tissue Lesions, page 1 OBJ: 1 3. Which condition is not diagnosed through clinical appearance? a. Mandibular tori b. Fordyce granules c. Black hairy tongue d. Compound odontoma ANS: D

The compound odontoma is initially identified radiographically as a radiopaque area in which tooth structure can be identified. No clinical component exists. Mandibular tori are identified clinically as areas of exostosis on the lingual aspects of mandibular premolars. Fordyce granules are yellow clusters of ectopic sebaceous glands diagnosed through clinical appearance. Black hairy tongue is diagnosed clinically. The filiform papillae on the dorsal tongue elongate and become brown or black. Causes include tobacco, alcohol, hydrogen peroxide, chemical rinses, antibiotics, and antacids. REF: Radiographic Diagnosis, page 9 4. Another name for geographic tongue is

OBJ: 3


a. b. c. d.

median rhomboid glossitis. benign migratory glossitis. fissured tongue. black hairy tongue.

ANS: B

Benign migratory glossitis is another name for geographic tongue. Research suggests that median rhomboid glossitis is associated with a chronic fungal infection from Candida albicans. Sometimes the condition resolves with antifungal therapy. Fissured tongue is seen in 5% of the population. It is a variant of normal. Genetic factors are typically associated with the condition. Black hairy tongue is caused by a reaction to chemicals, tobacco, hydrogen peroxide, or antacids. The filiform papillae on the dorsal tongue become elongated and are dark brown to black. REF: Geographic Tongue, page 24

OBJ: 7

5. This bony hard structure in the midline of the hard palate is genetic in origin and inherited in

an autosomal dominant manner. The diagnosis is made through clinical appearance. Which condition is suspected? a. Palatal cyst b. Torus palatinus c. Mixed tumor d. Ranula ANS: B

A torus palatinus is developmental and bony hard and is found on the midline of the palate. Diagnosis is made on the basis of clinical appearance. A palatal cyst appears radiolucent on a radiographic examination and is not diagnosed through clinical appearance. A mixed tumor or pleomorphic adenoma is a benign tumor of salivary gland origin, found unilaterally off the midline of the hard palate. It is composed of tumor tissue that is not bony hard to palpation. Ranula is a term used for a mucocele-like lesion that forms unilaterally on the floor of the mouth. REF: Torus Palatinus, page 21

OBJ: 4

6. The gray-white opalescent film seen on the buccal mucosa of 85% of black adults is a variant

of normal that requires no treatment and is termed a. linea alba. b. leukoedema. c. leukoplakia. d. white sponge nevus. ANS: B

Leukoedema is a diffuse opalescence most commonly seen on the buccal mucosa in black individuals. Linea alba is a “white line” that extends anteroposteriorly on the buccal mucosa along the occlusal plane. It is most prominent in patients who have a clenching or grinding habit. Leukoplakia is a clinical term for a white lesion, the cause of which is unknown. White sponge nevus is a genetic (autosomal dominant) trait. Clinically, it is characterized by a soft white, folded (or corrugated) oral mucosa. A thick layer of keratin produces the whitening. REF: Leukoedema, page 23

OBJ: 8


7. Which condition most likely responds to therapeutic diagnosis? a. Angular cheilitis b. Amelogenesis imperfecta c. Paget disease d. Stafne bone cyst ANS: A

Angular cheilitis most commonly responds to antifungal therapy once nutritional deficiencies have been ruled out. Amelogenesis imperfecta is a genetic condition associated with abnormal development of the enamel. Paget disease is a chronic metabolic bone disease. A highly elevated serum alkaline phosphatase level contributes significantly to the diagnosis. A Stafne bone cyst is determined through surgical diagnosis in which entrapped salivary gland tissue is identified. REF: Therapeutic Diagnosis, page 18

OBJ: 3

8. The gingival enlargement in this patient was caused by a calcium channel blocker. Which

medication is the likely cause? a. Dilantin b. Nifedipine c. Quinidine d. Clozapine ANS: B

Nifedipine is a calcium channel blocker. Dilantin is an anticonvulsant used to prevent or control seizures. Quinidine is an antiarrhythmic agent used to treat cardiac arrhythmias. Clozapine is an antipsychotic used in the management of psychotic symptoms in schizophrenia. REF: Historical Diagnosis, Fig. 1.38, page 17

OBJ: 3

9. Radiographic features, including cotton-wool radiopacities and hypercementosis, are

especially helpful in the diagnosis of a. Paget disease. b. dentinogenesis imperfecta. c. anemia. d. diabetes. ANS: A

Paget disease is a chronic metabolic bone disease. Radiographically, cotton-wool radiopacities and hypercementosis are characteristic features. Dentinogenesis imperfecta is a genetic condition involving a defect in the development of dentin. Anemia, a decrease in red blood cells, requires blood tests to determine the etiologic factors. Diabetes is a chronic disorder of carbohydrate metabolism characterized by abnormally high blood glucose levels. REF: Laboratory Diagnosis, Fig. 1.40, pages 16, 18

OBJ: 3

10. In internal resorption, the radiolucency seen on radiographic examination is usually a. well circumscribed. b. diffuse.


c. multilocular. d. unilocular. ANS: B

Diffuse borders are ill defined, making it impossible to detect the exact parameters of the lesion. Therefore treatment is more difficult. Well circumscribed describes borders that are specifically defined. Exact margins of the lesion are identified. Multilocular has also been described as resembling “soap bubbles”; lobes seem to fuse together to make up the lesion. This term has been used to describe the odontogenic keratocyst. Unilocular means having one compartment or unit that is well defined. This term is often used to describe the radicular cyst. REF: Vocabulary, Radiographic Terms Used to Describe Lesions in Bone, page 5 OBJ: 1 11. Which condition is diagnosed through clinical appearance? a. Fordyce granules b. Unerupted mesiodens c. Periapical cemento-osseous dysplasia d. Traumatic bone cyst ANS: A

Fordyce granules are diagnosed on the basis of their clinical appearance. They are ectopic sebaceous glands seen on the lips and buccal mucosa. Clinically, they appear as yellow lobules in clusters and are considered a variant of normal. Unerupted mesiodens requires a radiographic image for diagnosis. Periapical cemento-osseous dysplasia requires a radiographic image, specific patient history, and a pulp test to evaluate tooth vitality. Traumatic bone cyst requires a radiographic image and surgical intervention to establish a diagnosis. REF: Clinical Diagnosis, page 7 | Fordyce Granules, page 20

OBJ: 3

12. Retrocuspid papillae are located on the a. palate. b. floor of the mouth. c. gingival margin of the lingual aspect of mandibular cuspids. d. canine eminence. ANS: C

Retrocuspid papillae are located on the gingival margin of the lingual aspect of mandibular cuspids. Retrocuspid papillae are not located on the palate. Retrocuspid papillae are not located on the floor of the mouth. Retrocuspid papillae are not located on the canine eminence. REF: Retrocuspid Papilla, page 22

OBJ: 3

13. Which condition is not considered a variant of normal? a. Fordyce granules b. Leukoedema c. Linea alba d. Pyogenic granuloma ANS: D


Pyogenic granuloma is a reactive inflammatory response to injury. It is not a variant of normal. Fordyce granules are seen in more than 80% of adults over 20 years of age and are considered a variant of normal. Leukoedema is observed in about 85% of black individuals and is considered a variant of normal. Linea alba is located on the buccal mucosa along the occlusal plane of the teeth. It is most prominent in patients who have a clenching or bruxism habit. It is so common that it is considered a variant of normal. REF: Box 1-1, Case Study, page 20 | Variants of Normal, pages 21-23 OBJ: 6 14. When antifungal therapy is used to treat angular cheilitis, which diagnostic process is being

applied? a. Microscopic b. Laboratory c. Surgical d. Therapeutic ANS: D

Therapeutic diagnosis is used here in the treatment and management of angular cheilitis, which is most commonly a fungal condition. A careful patient history should be obtained to rule out a contributory nutritional deficiency. Microscopic diagnosis requires a biopsy. Laboratory diagnosis involves the use of clinical laboratory tests, including blood chemistries and urinalysis. Surgical diagnosis requires surgical intervention. REF: Therapeutic Diagnosis, page 18

OBJ: 3

15. A lesion with a stemlike base is described as a. sessile. b. macular. c. pedunculated. d. lobulated. ANS: C

Pedunculated means the lesion has a stemlike or stalklike base similar to that of a mushroom. A sessile base is broad and flat. A macular lesion is flat, does not protrude, and is distinguished by its color. A freckle is an example of a macule. Lobulated means the lesion consists of lobules making up the whole. REF: Vocabulary, Clinical Appearance of Soft Tissue Lesions, page 1 OBJ: 8 16. A small, circumscribed lesion, usually less than 1 cm in diameter, that is elevated or protrudes

above the surface of normal surrounding tissue is termed a a. papule. b. macule. c. vesicle. d. bulla. ANS: A


A papule is defined as a small circumscribed lesion, usually less than 1 cm in diameter, that is elevated or protrudes above the surface of normal surrounding tissue. A macule is flat and does not protrude above the surface of normal tissue. A vesicle is elevated and contains serous fluid. A bulla is elevated, contains serous fluid, and looks like a blister. REF: Vocabulary, Clinical Appearance of Soft Tissue Lesions, page 1 OBJ: 1 17. Which condition is considered a genetic/inherited disorder? a. Linea alba b. Amelogenesis imperfecta c. Necrotizing ulcerative gingivitis d. Internal resorption ANS: B

Amelogenesis imperfecta represents a group of inherited conditions affecting the enamel of teeth. Linea alba is a variant of normal characterized by the white line that extends anteroposteriorly on the buccal mucosa, along the occlusal plane. Necrotizing ulcerative gingivitis is an ulcerating gingival condition caused by anaerobic bacteria. Internal resorption is usually associated with an inflammatory response in the pulp. REF: Historical Diagnosis, page 15

OBJ: 3

18. Ectopic geographic tongue can be found in which location? a. On the lateral border of the tongue b. Within bone c. On mucosal surfaces other than the tongue d. The dorsal surface of the tongue ANS: C

Ectopic geographic tongue is also called stomatitis areata migrans and is found on mucosal surfaces other than the tongue. Geographic tongue may be seen on any surface of the tongue, including the lateral borders. Ectopic geographic tongue is a mucosal condition and is unrelated to the bone. Geographic tongue may be seen on any mucosal surface and is commonly seen on the dorsal surface of the tongue. REF: Geographic Tongue, page 24

OBJ: 7

19. A Stafne bone cyst contains a. salivary gland tissue. b. an empty void. c. inflammatory cells. d. an epithelium-lined cyst containing serous fluid. ANS: A

Stafne bone cyst is a developmental invagination in the lingual aspect of the mandible that is filled with salivary gland tissue. An empty void describes the contents of a traumatic bone cyst. A Stafne bone cyst has no inflammatory response. It is developmental and requires no treatment. A Stafne bone cyst is not a true cyst and does not have an epithelial lining. It contains normal salivary gland tissue. REF: Surgical Diagnosis, page 18

OBJ: 3


20. Which papillae are elongated in the condition black hairy tongue? a. Circumvallate b. Foliate c. Filiform d. Fungiform ANS: C

The filiform papillae are composed primarily of keratin and are the papillae that elongate in black hairy tongue. Circumvallate papillae on the posterior dorsal tongue do not elongate. Foliate papillae are located on the posterior lateral borders of the tongue. They are vertical, slightly exophytic folds of tissue located in the same area as the lingual lymphoid tissue (lingual tonsils) and do not elongate. Fungiform papillae on the dorsal areas do not elongate. REF: Hairy Tongue, page 24

OBJ: 3

21. Diagnosis of anemia is best accomplished through which examination? a. Laboratory tests b. Evaluation of the color of the gingiva c. Bleeding on probing d. Patient medical history ANS: A

Laboratory blood tests provide the best information with which to diagnose anemia. Although the color of the gingiva may be a clinical sign, a diagnosis of anemia, or more specifically the type of anemia, must be made through laboratory testing. Bleeding on probing is not useful in the diagnosis of anemia. The patient’s medical history may be helpful in the diagnosis of anemia, but it is only contributory. The final diagnosis comes from appropriate laboratory tests. REF: Laboratory Diagnosis, page 16

OBJ: 3

22. A radiolucency that “scallops around the roots of teeth” is often used to describe which

condition? a. Traumatic bone cyst b. Stafne bone cyst c. Lingual mandibular bone concavity d. Median palatine cyst ANS: A

Scalloping around the roots is a term often used to describe the radiographic appearance of a traumatic bone cyst. A Stafne bone cyst usually presents as an oval radiolucency anterior to the angle of the ramus and inferior to the mandibular canal. A lingual mandibular bone concavity is the same lesion as a Stafne bone cyst. A median palatine cyst is a developmental cyst that presents as a unilocular radiolucency found in the midline of the hard palate. REF: Vocabulary, Radiographic Terms Used to Describe Lesions in Bone, page 5 OBJ: 3 23. When geographic tongue occurs on the dorsal tongue, the erythematous areas are described as a. inflamed.


b. depapillated. c. allergic. d. fungal. ANS: B

In geographic tongue, the erythematous areas on the dorsal tongue are devoid of filiform papillae and therefore are appropriately described as depapillated. The erythematous areas seen on the dorsal tongue in geographic tongue are not caused primarily by inflammation. Geographic tongue is not an allergic reaction. Geographic tongue is not a fungal infection. REF: Geographic Tongue, page 24

OBJ: 7

24. It has been suggested that Candida albicans is associated with which condition? a. Median rhomboid glossitis b. Linea alba c. Leukoedema d. Retrocuspid papillae ANS: A

Research has suggested that median rhomboid glossitis may be associated with Candida albicans. Linea alba is a variant of normal. Leukoedema is a variant of normal. Retrocuspid papillae are developmental and found on the gingiva of the lingual aspect of mandibular canine teeth. REF: Median Rhomboid Glossitis, page 23

OBJ: 7

25. Which term is used to describe a radiographic lesion with borders that are specifically defined,

revealing the exact margins and extent of the lesion? a. Unilocular b. Well circumscribed c. Diffuse d. Multilocular ANS: B

Well circumscribed defines a lesion with borders that are specifically defined and in which one can see the exact margins and extent of the lesion. Unilocular means having one compartment or unit that is well defined (as in a radicular cyst). Diffuse describes a lesion with borders that are not well defined. Multilocular lesions are described radiographically as resembling “soap bubbles” (i.e., a lesion with many lobes beyond the confines of one distinct area). REF: Vocabulary, Radiographic Terms Used to Describe Lesions in Bone, page 5 OBJ: 1 26. Fordyce granules a. are ectopic sebaceous glands. b. are seen on the dorsal surface of the tongue. c. require antibiotic treatment. d. are lesions that require biopsy for diagnosis. ANS: A


Fordyce granules are ectopic sebaceous glands. Fordyce granules are seen on the buccal and labial mucosa, not the dorsal tongue. Fordyce granules require no treatment. Fordyce granules are diagnosed through clinical appearance. Biopsy is not necessary. REF: Fordyce Granules, page 20

OBJ: 4

27. The base of this lesion is correctly described as a. pedunculated. b. lobule. c. bulla. d. sessile. ANS: D

The base of this lesion is sessile, or broad and flat. A pedunculated lesion has a stemlike base similar to that of a mushroom stem. A lobule is a segment or lobe that is part of the whole. A bulla usually contains serous fluid and looks like a blister. REF: Vocabulary, Clinical Appearance of Soft Tissue Lesions, Fig. 1.5, A, pages 1, 3 OBJ: 1 28. This torus palatinus is correctly described as a. bullous. b. papular. c. lobulated. d. nodular. ANS: C

This torus palatinus is lobulated (i.e., lobes that are fused together). Bullous lesions contain serous fluid and resemble blisters. A papule is a soft tissue lesion that is elevated above the surface of normal surrounding tissue. A nodule is a palpable solid lesion found in soft tissue. REF: Vocabulary, Clinical Appearance of Soft Tissue Lesions, Fig. 1.1, pages 1, 2 OBJ: 1 29. Another term for an amalgam tattoo is a a. melanoma. b. focal argyrosis. c. nevus. d. multiple myeloma. ANS: B

Focal argyrosis is a synonym for amalgam tattoo. A melanoma is a malignant tumor. A nevus is a benign overgrowth of melanocytes. Multiple myeloma is a malignant proliferation of plasma cells. REF: Clinical Diagnosis, page 7

OBJ: 2

30. Which finding is not apparent in leukoedema? a. Intracellular edema in the spinous cells b. Acanthosis of the epithelium c. Generalized opalescence of the buccal mucosa d. A white diffuse material on the buccal mucosa that can be wiped off


ANS: D

A white diffuse material that can be wiped off is significant in diagnosing pseudomembranous candidiasis. Intracellular edema in the spinous cells is found in leukoedema. Acanthosis of the epithelium is found in leukoedema. Generalized opalescence of the buccal mucosa is seen in leukoedema. REF: Leukoedema, page 23

OBJ: 8

31. Which term best describes an elevated, 5-mm soft tissue lesion containing serous fluid? a. Macule b. Vesicle c. Lobule d. Pustule ANS: B

A vesicle is a small (less than 1 cm in diameter), elevated lesion that contains serous fluid. A macule is a flat area usually distinguished by a color different from the surrounding tissue. A lobule is described as a segment or lobe that is part of a whole. A pustule is a circumscribed elevation containing pus. REF: Vocabulary, Clinical Appearance of Soft Tissue Lesions, page 1 OBJ: 1 32. A lesion with a pedunculated base is best described as a. an ulcer. b. stemlike. c. pallor. d. broad and flat. ANS: B

A stemlike lesion is referred to as pedunculated. An ulcer is a break in the surface epithelium. Pallor is a paleness of the skin or mucosal tissues. Broad and flat describes the base of a sessile lesion. REF: Vocabulary, Clinical Appearance of Soft Tissue Lesions, page 1 OBJ: 1 33. The following lesions can be identified radiographically except one. Which one is the

exception? a. Root resorption b. Fordyce granules c. Interproximal dental caries d. Compound odontoma ANS: B

Fordyce granules are yellow clusters of ectopic sebaceous glands that are diagnosed through clinical appearance. Root resorption is identified radiographically when the apex of the tooth appears shortened or blunted. Interproximal dental caries are seen as radiographic radiolucencies. A compound odontoma is initially identified radiographically as a radiopaque area in which tooth structure can be identified.


REF: Clinical Diagnosis, page 7 | Fordyce Granules, page 20

OBJ: 3

34. These reddish-purple clusters observed on the ventral surface of the tongue and diagnosed

through clinical observation are referred to as which variants of normal conditions? a. Palatal cyst b. Mandibular tori c. Lingual varicosities d. Ranula ANS: C

Lingual varicosities are prominent lingual veins usually observed on the ventral and lateral surfaces of the tongue. A palatal cyst is radiolucent and not diagnosed through clinical appearance. Mandibular tori are outgrowths of hard dense bone and found on the lingual aspect of the mandible. Ranula is a term used to describe a mucocele-like lesion that forms unilaterally on the floor of the mouth. REF: Lingual Varicosities, page 22

OBJ: 4

35. An elevated serum alkaline phosphate level, significant in the diagnosis of Paget disease, is

determined from which diagnostic category? a. Laboratory b. Therapeutic c. Clinical d. Surgical ANS: A

Laboratory tests, including blood chemistries, can provide information that contributes to a diagnosis. Therapeutic testing applies the principle of diagnosis based on clinical and historical information with confirmation by the response of the condition to therapy. Clinical diagnosis suggests that the strength of the diagnosis comes from the clinical appearance of the lesion. Surgical diagnosis is based on information about a lesion gained during surgical intervention. REF: Laboratory Diagnosis, page 16

OBJ: 2

36. The following conditions most likely respond to therapeutic diagnosis except one. Which one

is the exception? a. Angular cheilitis b. Necrotizing ulcerative gingivitis c. Nutritional deficiencies d. Stafne bone cyst ANS: D

Stafne bone cyst, in which entrapped salivary gland tissue is identified, is diagnosed through surgical examination. Angular cheilitis commonly responds to antifungal therapy once nutritional deficiencies have been ruled out. Necrotizing ulcerative gingivitis responds to hydrogen peroxide rinses. Nutritional deficiencies are common conditions diagnosed by therapeutic means. REF: Surgical Diagnosis, page 18

OBJ: 3


37. The variant of normal coloration seen on the mandibular gingival surface is termed a. lingual varicosities. b. leukoedema. c. melanin pigmentation. d. linea alba. ANS: C

Melanin pigmentation is most commonly seen in dark-skinned individuals and gives color to the oral mucosa and gingiva. Lingual varicosities are prominent lingual veins observed on the ventral and lateral surfaces of the tongue. Leukoedema is a generalized opalescence imparted to the buccal mucosa. Linea alba is a white line that extends anteroposteriorly on the buccal mucosa along the occlusal plane of the teeth. REF: Melanin Pigmentation, Fig. 1.50, pages 21-22

OBJ: 6

38. A white lesion that cannot be rubbed off and cannot be diagnosed on the basis of clinical

characteristics alone is termed a. leukoplakia. b. dentinogenesis imperfecta. c. erythroplakia. d. squamous cell carcinoma. ANS: A

Leukoplakia is a clinical term for a white lesion that cannot be rubbed off and cannot be diagnosed on the basis of clinical characteristics alone. Dentinogenesis imperfecta is a genetic condition involving a defect in the development of dentin. Erythroplakia is a clinical term for a red lesion that cannot be diagnosed on the basis of clinical features alone. Squamous cell carcinoma is diagnosed by microscopic evaluation and does not refer to all white lesions that cannot be rubbed off. REF: Microscopic Diagnosis, page 18

OBJ: 9

39. The radiographic appearance of a simple radicular cyst is best described as a. coalescence. b. diffuse. c. multilocular. d. unilocular. ANS: D

Unilocular means having one compartment or unit that is well defined. This term is often used to describe a radicular cyst. Coalescence refers to the process by which parts of a whole join together, or fuse, to make one. Diffuse describes a lesion with ill-defined borders, making it impossible to detect the exact parameters of the lesion. A multilocular lesion has also been described as “soap bubble”–like; the lobes appear to fuse together to make up the lesion. REF: Vocabulary, Radiographic Terms Used to Describe Lesions in Bone, Fig. 1.13, pages 5, 6 OBJ: 1 40. A pathologic lesion found frequently in 30-year-old black women that requires a radiographic

image and historical data for diagnosis is termed a. verrucous vulgaris.


b. necrotizing ulcerative gingivitis. c. periapical cemento-osseous dysplasia. d. amalgam tattoo. ANS: C

Periapical cemento-osseous dysplasia requires a radiograph, specific patient history, and a pulp test to evaluate tooth vitality. It is frequently found in black women in the third decade of life. Verrucous vulgaris is diagnosed on the basis of its clinical and microscopic appearance and does not require a radiograph. Necrotizing ulcerative gingivitis requires clinical and historical data for diagnosis and does not require a radiograph. An amalgam tattoo relies on clinical and historical data for diagnosis and does not require a radiograph. REF: Historical Diagnosis, page 16

OBJ: 5

41. Leukoedema is a generalized opalescence appearing on the buccal mucosa; linea alba is a dark

pigmented line appearing on the buccal mucosa. a. Both statements are true. b. Both statements are false. c. The first statement is true, and the second statement is false. d. The first statement is false, and the second statement is true. ANS: C

The first statement is true, and the second statement is false. Leukoedema is a generalized opalescence appearing on the buccal mucosa, as stated, but linea alba is not a dark pigmented line. The statement that leukoedema is a generalized opalescence appearing on the buccal mucosa is true; the statement that linea alba is a dark pigmented line is false. (Linea alba is a white line.) Leukoedema is a generalized opalescence appearing on the buccal mucosa, but linea alba is not a dark pigmented line. REF: Leukoedema, Fig. 1.54, page 23 | Linea Alba, Fig. 1.53, page 23 OBJ: 8 42. Each condition is considered a variant of normal except one. Which is the exception? a. Melanin pigmentation b. Linea alba c. Geographic tongue d. Retrocuspid papilla ANS: C

Geographic tongue is characterized by diffuse areas devoid of filiform papillae. It is not a variant of normal. Melanin pigmentation is commonly seen in dark-skinned individuals and is considered a variant of normal. Linea alba is located on the buccal mucosa along the occlusal plane. It is most prominent in patients who have a clenching or bruxism habit. It is so common that it is considered a variant of normal. Retrocuspid papilla is a sessile nodule on the gingival margin of the lingual aspect of the mandibular cuspids and is considered a variant of normal. REF: Geographic Tongue, page 24 | Variants of Normal, pages 20-23 OBJ: 6 43. The term erythroplakia is best used in which context? a. Microscopic


b. Laboratory c. Surgical d. Clinical ANS: D

Erythroplakia is a descriptive clinical term for a red lesion. Microscopic diagnosis requires a biopsy. Laboratory diagnosis involves the use of clinical laboratory tests, including blood chemistries and urinalysis. Surgical diagnosis requires surgical intervention. REF: Vocabulary, Color of Lesion, page 2

OBJ: 9

44. The pathologic lesion seen on the dorsal surface of the tongue is characteristic of a. fissured tongue. b. hairy tongue. c. median rhomboid glossitis. d. migratory glossitis. ANS: C

Median rhomboid glossitis appears as a flat or slightly raised oval or rectangular erythematous area in the midline of the dorsal surface of the tongue. Fissured tongue is characterized by deep fissures or grooves. Hairy tongue has the appearance of white, elongated filiform papillae. Migratory glossitis is another name for geographic tongue and appears as patches on the lateral and dorsal surfaces of the tongue. REF: Median Rhomboid Glossitis, page 23 | Fig. 1.20, page 8

OBJ: 7

45. Each condition is considered benign and of unknown cause except one. Which one is the

exception? a. Hairy tongue b. Amelogenesis imperfecta c. Migratory glossitis d. Fissured tongue ANS: B

Amelogenesis imperfecta represents a group of inherited conditions affecting the enamel of teeth. Hairy tongue is a benign condition of unknown cause. Migratory glossitis is a benign condition of unknown cause. Fissured tongue is a benign condition of unknown cause. REF: Historical Diagnosis, page 15

OBJ: 7

46. A medical history of a patient prescribed a calcium channel blocker may reveal which

condition? a. Gingival enlargement b. Cementoma c. Paget disease d. Ulcerative colitis ANS: A

A patient taking a calcium channel blocker may exhibit gingival enlargement. A cementoma is not a result of a patient taking a calcium channel blocker. An elevated serum alkaline phosphatase level is significant for Paget disease but not for use of a calcium channel blocker. Ulcerative colitis may contribute to oral lesions but not gingival enlargement.


REF: Historical Diagnosis, Fig. 1.38, B, pages 16, 17

OBJ: 3

47. Urticaria is an example of a(n) a. genetic disorder. b. developmental disturbance. c. immediate response to an allergen. d. immunodeficiency response. ANS: C

Urticaria is an immediate response to an allergen. Urticaria is not the result of a genetic disorder. Urticaria is not the result of a developmental disturbance. Urticaria is not a result of immunodeficiency. REF: Historical Diagnosis, page 16

OBJ: 3

48. Which term describes a lesion in which parts of a whole are joined together, or fused, to make

one? a. Fissured b. Coalescence c. Diffuse d. Multilocular ANS: B

Coalescence describes the process by which parts of a whole join together, or fuse, to make one. Fissured describes a cleft or groove, normal or otherwise, showing prominent depth. Diffuse describes a lesion with borders that are not well defined. Multilocular lesions are described radiographically as “soap bubbles” (i.e., a lesion with many lobes beyond the confines of one distinct area). REF: Vocabulary, Radiographic Terms Used to Describe Lesions in Bone, page 3 OBJ: 1 49. While obtaining diagnostic information from a patient, the dental hygienist learns that the

appearance of this patient’s teeth is familial. The correct diagnosis is a. tetracycline staining. b. pulpal dysplasia. c. extrinsic staining. d. dentinogenesis imperfecta. ANS: D

Dentinogenesis imperfecta is a pathologic condition in which the family history plays a significant role in the diagnosis. Tetracycline staining is not a familial condition. Pulpal dysplasia is not a pathologic familial condition. Extrinsic staining is not a familial condition. REF: Historical Diagnosis, Fig. 1.37, pages 15, 16

OBJ: 3

50. The eight categories that provide information leading to a definitive diagnosis are as follows:

microscopic, clinical, laboratory, surgical, differential findings, radiographic, therapeutic, and a. etiology. b. historical. c. chief complaint.


d. treatment. ANS: B

Historical information reveals past experiences that may be relevant to the diagnosis. Etiology is not a component of the definitive or final diagnosis. The chief complaint may be helpful in understanding the patient’s perspective but is not considered a diagnostic tool. Treatment is decided after the definitive or final diagnosis is made, and it is not part of the information leading to the diagnosis. REF: Making a Diagnosis, page 6

OBJ: 3

51. What is the radiographic appearance of periapical cemento-osseous dysplasia in its earliest

stage? a. Radiolucent b. Radiopaque c. Radiolucent and radiopaque d. Cotton-wool radiolucencies ANS: A

In stage I of periapical cemento-osseous dysplasia, the appearance is radiolucent. Radiopacities are not seen in the early stage of perioapical cemento-osseous dysplasia. In stage II, the appearance is radiolucent and radiopaque. Cotton-wool radiopacities are seen in patients with Paget disease. REF: Vocabulary, Radiographic Terms Used to Describe Lesions in Bone, page 3 OBJ: 5 52. All conditions are associated with lingual varicosities except one. Which is the exception? a. Red to purple enlarged vessels b. May be associated with varicosities in the legs c. Age-related condition d. May swell during eating ANS: D

Lingual varicosities do not swell during the eating process. Red to purple enlarged vessels are seen with lingual varicosities. Lingual varicosities may be associated with varicosities in the legs. Lingual varicosities are considered an age-related condition, often seen in individuals over the age of 60. REF: Variants of Normal, pages 22-23

OBJ: 4

53. Various sizes of circumscribed elevations that contain pus are termed a. vesicles. b. pustules. c. papules d. lobules ANS: B

Pustules are variously sized circumscribed elevations containing pus. Vesicles are elevated lesions that contain serous fluid. Papules are elevated lesions that protrude above the surface of normal surrounding tissue. Lobules are segments or lobes that are part of the whole.


REF: Vocabulary, Clinical Appearance of Soft Tissue Lesions, page 1 OBJ: 1 54. Studies have linked the presence of a lingual thyroid in association with the following life

events except one. Which is the exception? a. Pregnancy b. Infancy c. Menopause d. Puberty ANS: B

Infancy is not associated with the presence of a lingual thyroid. The emergence of a lingual thyroid is linked to hormonal changes, such as pregnancy. The emergence of a lingual thyroid is linked to hormonal changes, such as menopause. The emergence of a lingual thyroid is linked to hormonal changes, such as puberty. REF: Benign Conditions of Unknown Cause, page 23

OBJ: 11

55. The lingual thyroid is most often found in which intraoral location? a. Posterior to the circumvallate papillae in the midline of the tongue b. Along the occlusal plane on the buccal mucosa c. At the junction of the hard and soft palate d. At the labial commissures ANS: A

The lingual thyroid is seen as a mass in the midline of the dorsal surface of the tongue, posterior to the circumvallate papillae. The lingual thyroid is not seen on the buccal mucosa. The lingual thyroid is not found at the junction of the hard and soft palate. The lingual thyroid is not found at the labial commissures. REF: Benign Conditions of Unknown Cause, page 23

OBJ: 11

56. Your patient is diagnosed with a lingual thyroid. What is the next step on the treatment plan? a. Biopsy b. Surgical removal c. Thyroid scan d. Radiographic evaluation ANS: C

A thyroid scan should be performed to determine if the thyroid gland is functioning normally. A biopsy is not needed for a lingual thyroid because it is composed of normal thyroid tissue. Surgical removal is not recommended since this may represent the patient’s functioning thyroid. Radiographic evaluation is not helpful in the confirmation of a lingual thyroid. REF: Benign Conditions of Unknown Cause, page 23

OBJ: 11

57. Because of its size and location, patients with a lingual thyroid may complain of a. dysphagia. b. gingival inflammation. c. intraoral ulcers. d. tooth erosion.


ANS: A

Clinical symptoms of lingual thyroid can include dysphagia, or difficulty swallowing, because of the mass located at the posterior dorsal surface of the tongue. Gingival inflammation is not associated with patients with a lingual thyroid. Intraoral ulcers are not associated with patients with a lingual thyroid. Tooth erosion is not associated with patients with a lingual thyroid. REF: Benign Conditions of Unknown Cause, page 23

OBJ: 11

58. Where is the common intraoral location for leukoedema? a. Lateral borders of the tongue b. Soft palate and uvula c. Floor of the mouth d. Buccal mucosa ANS: D

A generalized opalescence is imparted to the buccal mucosa in leukoedema. Leukoedema is not evident on the lateral borders of the tongue. The soft palate and uvula is not a location to find leukoedema. The floor of the mouth is not a location to see leukoedema. REF: Variants of Normal, page 23

OBJ: 8

59. The human papillomavirus (HPV) is associated with what condition? a. Thrush b. Tuberculosis c. Verruca vulgaris d. Herpangina ANS: C

Verruca vulgaris (the common wart) is caused by a low-risk type of the human papillomavirus. Thrush, or pseudomembranous candidiasis, is a superficial fungal infection seen in the oral cavity. Tuberculosis is a bacterial infection caused by inhalation of airborne droplets from someone with active disease. Herpangina is caused by a coxsackie virus and is characterized by fever, sore throat and oral lesions. REF: Microscopic Diagnosis, page 18

OBJ: 10

60. A “white” hairy tongue indicates an increase in the amount of a. keratin. b. medications. c. fluoride use. d. HPV circulating in the blood. ANS: A

The increased amount of keratin on the filiform papillae gives the tongue a white appearance. Medications may affect the surface of the tongue but generally do not produce a white color. Fluoride use does not cause a “white” hairy tongue. The presence of the HPV does not cause a “white” hairy tongue. REF: Benign Conditions of Unknown Cause, page 24

OBJ: 7

61. Benign migratory glossitis is a condition that does not remain static. What does this statement

mean?


a. b. c. d.

A variety of over-the-counter treatments are available for this condition. Surgical intervention may be required. Remission and changes in the surface of the tongue occur. An intraoral habit is responsible for this condition.

ANS: C

Benign migratory glossitis (geographic tongue) is a condition that does not stay the same over time; remission and changes in the surface of the tongue occur. A condition that does not remain static does not refer to treatment protocol. A condition that does not remain static does not refer to treatment protocol. Intraoral habits are not related to benign migratory glossitis. REF: Benign Conditions of Unknown Cause, page 24

OBJ: 7

62. Which statement about a palatal torus is true? a. A palatal torus is symptomatic. b. A palatal torus appears as a radiolucent mass on a radiograph. c. Palatal tori are typically bilateral, seen along the lingual side of the maxilla. d. Palatal tori are inherited. ANS: D

Palatal tori are inherited. Palatal tori are asymptomatic. A palatal torus appears as a radiopaque mass on a radiograph. Palatal tori can take on various shapes and sizes yet occur in the midline of the palate. REF: Variants of Normal, page 21

OBJ: 4

63. Serous fluid can typically be found in which lesion? a. Bulla b. Lobule c. Macule d. Papule ANS: A

Bullae are circumscribed, elevated lesions that usually contain serous fluid and resemble a blister. Lobules are segments or lobes that are part of a whole. Macules are areas distinguished by a different color than the surrounding tissue. Papules are small, circumscribed lesions that protrude above the surface. REF: Vocabulary, Clinical Appearance of Soft Tissue Lesions, page 1 OBJ: 1 64. Which term describes a lesion that displays a color different from surrounding tissue? a. Bulla b. Lobule c. Macule d. Papule ANS: C

Macules are areas distinguished by a different color than the surrounding tissue. Bullae are circumscribed, elevated lesions that usually contain serous fluid and resemble a blister. Lobules are segments or lobes that are part of a whole. Papules are small, circumscribed lesions that protrude above the surface.


REF: Vocabulary, Clinical Appearance of Soft Tissue Lesions, page 1 OBJ: 1 65. Trying to establish the correct number of primary and permanent teeth present in a 7-year-old

child may best be accomplished by which diagnostic component? a. Clinical b. Historical c. Radiographic d. Differential ANS: C

A radiographic examination including intraoral and extraoral images would provide sufficient information to establish the correct number of teeth present. The clinical diagnosis would be adequate to view only the erupted teeth, not the unerupted permanent teeth. Some historical data might be helpful in this case but would not be considered the best diagnostic component. Enough information should be present in the clinical and radiographic examinations that a differential diagnosis is not necessary. REF: Making a Diagnosis, page 9

OBJ: 3


Chapter 02: Inflammation and Repair Ibsen: Oral Pathology for the Dental Hygienist, 7th Edition MULTIPLE CHOICE 1. A decrease in the size and function of a cell, a tissue, an organ, or the body is referred to as a. emigration. b. atrophy. c. hyperplasia. d. phagocytosis. ANS: B

Atrophy is defined as a decrease in the size and function of a cell, a tissue, an organ, or the whole body. Emigration is the passage of white blood cells through the endothelium and wall of the microcirculation into the injured tissue. Hyperplasia is the enlargement of a tissue or organ resulting from an increase in the number of normal cells. Phagocytosis is the process of ingestion and digestion of particulate material by cells. REF: Reactive Tissue Response, page 43

OBJ: 8

2. The first response of the body to injury is a. anaphylaxis. b. erythema. c. fever. d. inflammation. ANS: D

The inflammatory response is the first reaction to injury, and it involves a series of microscopic events. Anaphylaxis is a severe type of hypersensitivity or allergic reaction in which there is an exaggerated immunologic reaction resulting from the release of vasoactive substances such as histamine. Erythema is redness of the skin or mucosa and is a local sign of inflammation. Fever is the elevation of the normal body temperature and is a systemic sign of inflammation. REF: Inflammation, page 34

OBJ: 1

3. Which type of inflammation occurs when the injury is minimal and brief and its source is

removed from the tissue? a. Acute b. Chronic c. Local d. Systemic ANS: A

Acute inflammation occurs when the injury is minimal and brief. Chronic inflammation occurs when the inflammatory response lasts for longer periods, even indefinitely. Local is a term used to describe a specific area of inflammation. Systemic factors such as fever, leukocytosis, and lymphadenopathy occur when the injury is extensive. REF: Inflammation, page 34

OBJ: 2


4. Which cell is the first to arrive at the site of injury and is the primary cell type involved in

acute inflammation? a. Macrophage b. Neutrophil c. Plasma cell d. Mast cell ANS: B

The neutrophil is the first cell to arrive at the site of injury and is the primary cell type involved in acute inflammation. The macrophage is the second cell type to participate in the inflammatory response. The plasma cell is involved in chronic inflammation. The mast cell participates in both the inflammatory and immune responses. REF: White Blood Cells in the Inflammatory Response, page 38 OBJ: 4 5. Which symptom is not a classic local sign of inflammation? a. Redness b. Swelling c. Leukocytosis d. Loss of normal tissue function ANS: C

Leukocytosis is an increase in the number of white blood cells and is a sign of systemic inflammation. Redness is a local clinical change at the site of injury and is one of the classic local signs of inflammation. Swelling is a local clinical change observed at the site of injury and is one of the classic local signs of inflammation. Loss of normal tissue function at the site of injury is a classic local sign of inflammation. REF: Leukocytosis, page 41

OBJ: 5

6. Healing of an injury with little tissue loss, such as a surgical incision, is referred to as healing

by which type of intention? a. Tertiary b. Keloid c. Secondary d. Primary ANS: D

Healing by primary intention occurs when there is very little loss of tissue. The clean edges of the surgical incision are joined with sutures, and very little granulation tissue forms. Healing by tertiary intention occurs when an infection develops at the site of a surgical incision that is healing by primary intention. Healing by secondary intention may ensue. Keloid formation is excessive scar tissue development that can occur in healing by secondary intention when there is a significant loss of tissue. Healing by secondary intention occurs when the injury involves significant loss of tissue and the edges of the injury cannot be joined during healing. A large clot forms, resulting in an increase in granulation tissue. REF: Healing by Primary Intention, page 45

OBJ: 11


7. The wearing away of tooth structure during mastication is termed a. attrition. b. erosion. c. abrasion. d. abfraction. ANS: A

Attrition is defined as the wearing away of tooth structure during mastication. Erosion is the loss of tooth structure from chemical action. Abrasion is a pathologic wearing of tooth structure resulting from a repetitive mechanical habit. Abfraction is the result of biomechanical forces on the teeth. REF: Attrition, page 46

OBJ: 14

8. The loss of tooth structure seen in bulimia is caused by a. anorexia. b. erosion. c. attrition. d. bruxism. ANS: B

Generalized erosion, especially on the lingual surfaces of maxillary anterior teeth, is caused by frequent vomiting in patients with bulimia. Patients with anorexia nervosa do not vomit after eating. Attrition is the wearing away of tooth structure during mastication. Bruxism occurs when there is nonfunctional grinding or clenching of the teeth. REF: Erosion, page 48

OBJ: 14

9. A patient comes to the office for an emergency visit complaining of a toothache in the left

posterior mandible. On clinical examination you notice a gray-to-white patch on the left posterior buccal mucosa. On questioning, the patient tells you that this area is also painful. After reviewing the patient’s medical history, you question the patient regarding his recent use of a. hydrogen peroxide. b. aspirin. c. antibiotics. d. mouthwash. ANS: B

This is a classic case of aspirin burn caused by the misuse of aspirin. The patient placed aspirin near the tooth that was aching; thus necrosis of the mucosa occurred, resulting in the painful white patch on the buccal mucosa. A chemical burn from the use of hydrogen peroxide would be more diffuse, probably bilateral, and not a white plaque. Antibiotics would be taken systemically and most likely swallowed. Commercial mouthwashes would not cause a localized lesion. REF: Aspirin Burn, page 49OBJ:

17

10. A white raised line observed on the buccal mucosa along the occlusal plane of the teeth is

most likely a. cheek biting.


b. linea alba. c. white sponge nevus. d. frictional keratosis. ANS: B

Linea alba is a raised white line on the buccal mucosa along the occlusal plane; it is considered a variant of normal. Cheek biting is usually seen bilaterally as a diffuse area causing sloughing of the buccal mucosa near the occlusal plane. White sponge nevus is a genetic autosomal-dominant condition. Clinically, it is characterized by a white, soft, folding (corrugation) of the buccal mucosa. A thick layer of keratin produces the whitening effect. Frictional keratosis is caused by a chronic rubbing or friction against the mucosa or alveolar ridge. Diagnosis is made by identifying the trauma causing the lesion and removing the cause. REF: Linea Alba, page 52

OBJ: 17

11. Which term describes white blood cells adhering to the walls of a blood vessel during

inflammation? a. Margination b. Pavementing c. Leukocytosis d. Emigration ANS: B

Pavementing is the adherence of white blood cells to the walls of a blood vessel during inflammation. Margination is a process during inflammation in which white blood cells move to the periphery of the blood vessel wall. Leukocytosis is a temporary increase in the number of white blood cells circulating in blood. Emigration is the passage of white blood cells through the endothelium and wall of the microcirculation into the injured tissue. REF: Microscopic Events of Inflammation and Clinical Signs, page 35 OBJ: 1 12. Which symptom is a systemic sign of inflammation? a. Redness b. Pain c. Loss of normal tissue function d. Fever ANS: D

Fever is a systemic sign of inflammation. Redness is a local sign of inflammation. Pain is a local sign of inflammation caused by pressure on nerves by exudate formation. Loss of normal tissue function is a local sign of inflammation associated with local swelling and pain. REF: Microscopic Events of Inflammation and Clinical Signs, Table 2-1, page 35 OBJ: 3 13. The enlargement of lymph nodes is termed a. atrophy. b. lymphadenopathy. c. hyperplasia. d. leukocytosis.


ANS: B

Lymphadenopathy occurs when lymph nodes become enlarged and palpable. Atrophy is a decrease in size and function of a cell, a tissue, an organ, or the whole body. Hyperplasia is an enlargement of a tissue or an organ resulting from an increase in the number of normal cells. Leukocytosis is a temporary increase in the number of white blood cells. REF: Lymphadenopathy, page 41

OBJ: 1

14. The first microscopic event in the inflammatory response is a. decreased blood flow. b. constriction of the microvasculature. c. phagocytosis. d. dilation of microvasculature. ANS: B

After injury to the tissue, the first microscopic event is constriction of the microvasculature. Decreased blood flow occurs after exudate formation. Phagocytosis occurs when the white blood cells remove foreign substances from the site by ingestion and digestion; these substances must be removed for the inflammation to resolve. Dilation of the microvasculature is the second microcirculation event to occur after injury. REF: Microscopic Events of Inflammation and Clinical Signs, page 34 OBJ: 4 15. Serous exudate is composed of which material(s)? a. Tissue debris and many white blood cells b. Suppuration c. Plasma fluids and proteins with a few white blood cells d. Plasma fluids and red blood cells ANS: C

Serous exudate is composed of plasma fluids and proteins with a few white blood cells. Purulent exudate contains tissue debris and many white blood cells. Suppuration is the formation and discharge of pus, as seen in purulent exudate. Serous describes the watery consistency of plasma. Red blood cells are not a component of serous fluid. REF: Microscopic Events of Inflammation and Clinical Signs, page 35 OBJ: 5 16. When formation of exudate is excessive, a drainage tract may develop through the injured

tissue. This channel is often termed a. a fistula. b. leukocytosis. c. erythema. d. emigration. ANS: A

A fistula is the channel through which excessive exudate passes to drain to the outside. Leukocytosis is a temporary increase in white blood cells. Erythema is redness of the skin or mucosa. Emigration occurs when white blood cells pass through the endothelium and wall of the microcirculation into the injured tissue.


REF: Microscopic Events of Inflammation and Clinical Signs, page 35 OBJ: 5 17. Neutrophils constitute what percentage of the entire white blood cell population? a. 5% b. 20% c. 65% d. 90% ANS: C

Neutrophils make up 60% to 70% of all white blood cells. There are significantly more than 5% of neutrophils in the entire white blood cell count. There are significantly more than 20% of neutrophils in the entire white blood cell count. There are fewer than 90% of neutrophils in the entire white blood cell count. REF: Neutrophils, page 38

OBJ: 6

18. All statements are true concerning the neutrophil except that the neutrophil is a. the first cell at the site of injury. b. the primary cell in acute inflammation. c. the primary cell in chronic inflammation. d. a phagocyte. ANS: C

In chronic inflammation, the primary cells are macrophages, lymphocytes, and plasma cells. Neutrophils are the first cells at the site of injury. Neutrophils are the primary cells in acute inflammation. The main function of the neutrophil is phagocytosis. REF: White Blood Cells in the Inflammatory Response, page 38 OBJ: 6 19. Which system mediates inflammation by causing increased dilation of the blood vessels at the

site of injury and increases the permeability of local blood vessels? a. Kinin system b. Clotting system c. Complement system d. Lysosomal enzymes ANS: A

The kinin system mediates inflammation by causing increased dilation of the blood vessels at the site of injury and increases the permeability of local blood vessels. The clotting mechanism functions primarily in the clotting of blood. The complement system involves the production of a sequential cascade of plasma proteins that function in inflammation and immunity. Lysosomal enzymes are released from granules in the white blood cells; they act as chemotactic factors and can cause damage to connective tissues and the clot that has formed at the site of injury. REF: Kinin System, page 40

OBJ: 4

20. Which medication is a steroidal anti-inflammatory drug? a. Aspirin b. Prednisone


c. Ibuprofen d. Motrin ANS: B

Prednisone is a steroidal anti-inflammatory drug. Aspirin is a nonsteroidal anti-inflammatory agent. Ibuprofen is a nonsteroidal anti-inflammatory agent. Motrin is ibuprofen, a nonsteroidal anti-inflammatory drug. REF: Anti-inflammatory Drugs, page 42

OBJ: 6

21. Which term is defined as an increase in the number of cells in a tissue or organ? a. Hypertrophy b. Atrophy c. Hyperplasia d. Repair ANS: C

Hyperplasia is an increase in the number of cells in a tissue or organ. Hypertrophy is an increase in the size of an organ or tissue but not in the number of cells. Atrophy is a decrease in the size and function of a cell, a tissue, or an organ. Repair is the restoration of damaged or diseased tissue. REF: Reactive Tissue Response, page 43

OBJ: 1

22. Excessive scarring in skin is termed a. a keloid. b. healing by primary intention. c. a hematoma. d. healing by tertiary intention. ANS: A

A keloid occurs when there is excessive scarring in the skin. In healing by primary intention, very little granulation tissue forms. A hematoma occurs when there is hemorrhage into the tissue. This may impair healing. Healing by tertiary intention occurs after the infection has been resolved and surgical tissue repair has been performed. REF: Healing by Secondary Intention, page 45

OBJ: 9

23. The first sign of attrition is a. open contacts. b. disappearance of mamelons on incisors. c. temporomandibular joint dysfunction. d. biomechanical forces on the teeth. ANS: B

The first sign of attrition is the disappearance of mamelons on incisors. Open contacts are associated with erosion. Temporomandibular joint dysfunction problems are more likely associated with bruxism. Excessive attrition, muscle pain, and wear facets are present in bruxism. Abfraction results from biomechanical forces on the teeth. REF: Attrition, page 46

OBJ: 14


24. Which habit is not a cause of abrasion? a. Pipe placement by smokers b. Playing wind instruments c. Holding needles or pins with the teeth d. Frequent sucking of lemons ANS: D

Erosion, not abrasion, can be caused by the frequent sucking of lemons. Abrasion can be caused by pipe placement by pipe smokers. Abrasion may be caused by playing wind instruments. Abrasion can be caused by needles and pins held between the teeth. REF: Erosion, pages 48-49

OBJ: 14

25. This type of erosion is classically associated with a. anorexia nervosa. b. bulimia. c. sucking lemons. d. abrasive toothpaste. ANS: B

Bulimia is an eating disorder characterized by food binges followed by self-induced vomiting that causes erosion to the lingual aspects of the teeth. Anorexia nervosa is an eating disorder, but it is not associated with erosion because vomiting after eating is not a component of the disorder. Sucking lemons would cause erosion to the facial aspects of the teeth. Abrasive toothpastes are more responsible for contributing to abrasion. REF: Erosion, Fig. 2.25, page 48

OBJ: 14

26. Aspirin burn on the oral mucosa a. is caused by ingestion of too many aspirin tablets. b. is caused by placing the aspirin on the tooth with the toothache, causing the

surrounding mucosa to become necrotic. c. is painless. d. requires a biopsy for diagnosis. ANS: B

Aspirin burn is caused when aspirin is placed on the tooth with the toothache, causing the surrounding mucosa to become necrotic. Aspirin burn is caused by a topical misuse of aspirin; it is not systemic. Aspirin burn is very painful and slow to heal. Aspirin burn is usually diagnosed by questioning the patient to reveal the cause of the lesion. REF: Aspirin Burn, page 49OBJ:

17

27. Electric burns in the oral area are usually seen in which patient group? a. Electricians b. Infants and young children c. Elderly d. Individuals involved in an electrical fire ANS: B


Electric burns in the oral area are most often seen in infants and young children who have bitten or chewed a live electrical cord. Electricians do not usually have electric burns in the oral area. The elderly do not usually have electric burns in the oral area. Individuals in an electrical fire do not usually have electric burns in the oral area. REF: Electric Burn, page 50

OBJ: 17

28. The diagnosis of a traumatic ulcer is usually based on a. history of the lesion. b. scalpel biopsy. c. therapeutic procedures. d. laboratory tests. ANS: A

Traumatic ulcers are usually diagnosed on the basis of the relationship of the history to the lesion. Scalpel biopsy is not used in the diagnosis of traumatic ulcers. However, if the trauma persists and the ulcer lasts 14 days, a biopsy may be performed. Therapeutic measures are not used to diagnose traumatic ulcers. Laboratory tests are not used to diagnose traumatic ulcers. REF: Traumatic Ulcer, page 51

OBJ: 17

29. The major cause of a mucocele is a. a sialolith. b. salivary duct obstruction. c. trauma to a minor duct. d. allergic reaction. ANS: C

The major cause of a mucocele is trauma to a minor duct. The mucous salivary gland secretion spills into the adjacent connective tissue. A sialolith is a salivary gland stone. Dilated salivary gland ducts are believed to develop as a result of salivary duct obstruction. A mucocele is not caused by an allergic reaction. REF: Mucous Retention Lesions, page 56

OBJ: 19

30. Necrotizing sialometaplasia is thought to result from a. lack of blood supply to the affected salivary gland. b. a sialolith. c. trauma to the floor of the mouth. d. pleomorphic adenoma. ANS: A

Necrotizing sialometaplasia results from lack of blood supply to the affected salivary gland. A sialolith is a salivary gland stone that causes an obstruction in the salivary gland. Necrotizing sialometaplasia occurs on the hard palate, not the floor of the mouth. Pleomorphic adenoma is a benign salivary gland tumor found unilaterally on the posterior palate. REF: Necrotizing Sialometaplasia, page 57 31. Which choice is most likely to result in frictional keratosis? a. High-fiber diet b. Chewing on an edentulous ridge

OBJ: 18


c. Malignancy d. Daily use of mouthwash ANS: B

Frictional keratosis results from chronic chewing on an edentulous ridge. A high-fiber diet does not cause frictional keratosis. Frictional keratosis is not associated with malignancy. Mouthwashes do not cause frictional keratosis. REF: Frictional Keratosis, page 52

OBJ: 17

32. This lesion on the palate is typically associated with heavy pipe and cigar smoking and is

termed a. tobacco pouch keratosis. b. necrotizing sialometaplasia. c. nicotine stomatitis. d. frictional keratosis. ANS: C

Nicotine stomatitis is a benign lesion of the hard palate typically associated with heavy pipe and cigar smoking. Tobacco pouch keratosis occurs in the mucobuccal fold and is caused by chewing/spitting tobacco. Necrotizing sialometaplasia is caused by lack of blood supply to a specific area of the palate. An ulcer is often present. Frictional keratosis results from chronic chewing on an edentulous alveolar ridge. REF: Nicotine Stomatitis, page 52

OBJ: 18

33. Traumatic neuroma is a lesion caused by injury to which structure? a. The epithelium b. A peripheral nerve c. A salivary gland d. Striated muscle ANS: B

The traumatic neuroma is a lesion caused by injury to a peripheral nerve. The mental foramen is the most common location. The traumatic neuroma does not result from epithelial injury. The traumatic neuroma does not result from injury to a salivary gland. The traumatic neuroma does not result from injury to striated muscle. REF: Traumatic Neuroma, page 52

OBJ: 18

34. Which lesion occurs on the gingiva or alveolar process and contains many multinucleated

giant cells, red blood cells, and chronic inflammatory cells? a. Ranula b. Central giant cell granuloma c. Fibroma d. Peripheral giant cell granuloma ANS: D


The peripheral giant cell granuloma occurs on the gingiva or alveolar process; originates from the periodontal ligament; is thought to be a response to injury; and histologically is characterized by many multinucleated giant cells, red blood cells, and inflammatory cells. The ranula is found on the floor of the mouth. The central giant cell granuloma is found within bone. The fibroma occurs most frequently on the buccal mucosa and is composed of dense scarlike connective tissue containing few blood vessels. REF: Peripheral Giant Cell Granuloma, page 58

OBJ: 18

35. Epulis fissuratum is caused by a. denture adhesive products. b. poor suction in the palatal vault. c. poor denture hygiene. d. an ill-fitting denture flange. ANS: D

Epulis fissuratum is caused by an ill-fitting denture flange. Denture adhesive products do not cause epulis fissuratum. Poor suction in the palatal vault causes papillary hyperplasia of the palate. Poor denture hygiene does not cause epulis fissuratum. REF: Denture-induced Fibrous Hyperplasia, page 60

OBJ: 18

36. This granular, erythematous papillary surface of the palatal vault was caused by a. poor oral hygiene. b. an ill-fitting suction area of a maxillary denture. c. the denture flange. d. soaking the denture in caustic rinses. ANS: B

Papillary hyperplasia is caused by the palatal suction of an ill-fitting maxillary denture. Poor oral hygiene does not cause papillary hyperplasia. It may contribute to the inflammatory response of the area. The ill-fitting denture flange causes epulis fissuratum. Soaking the denture in caustic rinses may contribute to inflammation but not papillary hyperplasia. REF: Inflammatory Papillary Hyperplasia of the Palate, page 61 OBJ: 17 37. The most common site for the development of a pulp polyp is a. the occlusal surface of a large open carious tooth. b. the apex of the tooth. c. the gingival margin of the tooth. d. deep in the pulp canal. ANS: A

The most common site for the pulp polyp is in the occlusal surface of large open carious teeth. It is seen as a red or pink nodule that fills the occlusal surface. It is an excessive proliferation of chronically inflamed dental pulp tissue. Pulp polyps are not seen at the apex of teeth. Pulp polyps are not seen on the gingival margin of teeth. Pulp polyps are not seen deep in the pulp canal. REF: Chronic Hyperplastic Pulpitis, page 62

OBJ: 18


38. Which does not cause gingival enlargement? a. Hormonal changes b. Calcium channel blockers c. Hereditary factors d. Nitroglycerin ANS: D

Nitroglycerin is prescribed for angina and does not cause gingival enlargement. Hormonal changes do contribute to gingival enlargement. Calcium channel blockers do cause gingival enlargement. Certain hereditary factors do cause gingival enlargement. REF: Gingival Enlargement, pages 61-62

OBJ: 8

39. Which inflammatory periapical lesion is most painful? a. Periapical abscess b. Periapical granuloma c. Radicular cyst d. Residual cyst ANS: A

The periapical abscess is associated with severe pain caused by the inflammation. Periapical granuloma is most often asymptomatic. The radicular cyst is often asymptomatic and discovered on radiographic examination. The residual cyst forms when the radicular cyst is incompletely removed and left behind at the extraction site. REF: Periapical Abscess, page 62

OBJ: 22

40. Resorption of tooth structure from outside the tooth is termed a. internal resorption. b. external resorption. c. idiopathic tooth resorption. d. condensing osteitis. ANS: B

External resorption begins outside the tooth. Internal resorption begins inside the pulpal area. Idiopathic tooth resorption can involve the crown or roots of impacted teeth, and the cause cannot be identified. Condensing osteitis is a change in the bone near the apices of teeth that is thought to be a reaction to a low-grade infection. The mandibular first molar is most commonly involved, and the area is seen radiographically as a radiopacity below the root apex of the involved tooth. REF: Tooth Resorption, page 64

OBJ: 13

41. A process during inflammation in which white blood cells move to the blood vessel wall is

termed a. chemotaxis. b. margination. c. leukocytosis. d. transudate. ANS: B


Margination is defined as a process during inflammation in which white blood cells move to the blood vessel wall. Chemotaxis is the directed movement of white blood cells to the area of injury by biochemical mediators. Leukocytosis is a temporary increase in the number of white blood cells circulating in blood. Transudate is the fluid component of blood that normally passes through the endothelial walls of the microvasculature. REF: Microscopic Events of Inflammation and Clinical Signs, page 35 OBJ: 1 42. An example of an irreversible cellular response that occurs during tissue injury is termed a. atrophy. b. hypertrophy. c. hyperplasia. d. necrosis. ANS: D

Necrosis is the pathologic death of one or more cells or a portion of the tissue or an organ that results from irreversible damage to cells. Atrophy is an example of a reversible cellular response. Hypertrophy is an example of a reversible cellular response. Hyperplasia is an example of a reversible cellular response. REF: Injury, page 34

OBJ: 2

43. The inflammatory response is a dynamic process, continually changing in response to injury

and repair. Repair of tissue occurs only if the persistent source of injury is removed. a. Both statements are true. b. Both statements are false. c. The first statement is true; the second is false. d. The first statement is false; the second is true. ANS: A

Both statements are true. REF: Inflammation, page 34

OBJ: 3

44. Hyperemia is responsible for which two clinical signs of inflammation? a. Emigration and pain b. Heat and erythema c. Transudation and redness d. Swelling and chemotaxis ANS: B

Heat and erythema are caused by hyperemia. Emigration is the process by which the white blood cells escape from the blood vessels and is not a clinical sign of inflammation. Transudation is the process of plasma cells passing between the endothelial cells and entering the tissue and is not a clinical sign of inflammation. Chemotaxis is the directed movement of white blood cells toward the site of the injury and is not a clinical sign of inflammation. REF: Microscopic Events of Inflammation and Clinical Signs, page 35 OBJ: 4 45. During inflammation, pain may be caused by which microscopic event?


a. b. c. d.

Phagocytosis Leukocytosis Exudate formation Anaphylaxis

ANS: C

Exudate presses on sensory nerves and may cause pain. Phagocytosis is the ingestion and digestion of foreign substances and is not a cause of pain. Leukocytosis is an increase in the number of white blood cells and is not a cause of pain. Anaphylaxis is a type of hypersensitivity or allergic reaction and is not a direct cause of pain. REF: Microscopic Events of Inflammation and Clinical Signs, page 35 OBJ: 5 46. During the acute inflammatory process, the second type of white blood cell to emigrate from

the blood vessel into the injured tissue is termed a. macrophage. b. neutrophil. c. plasma cell. d. lymphocyte. ANS: A

The macrophage is the second cell to participate in the inflammatory response. The neutrophil is the first cell to arrive at the site of injury and is the primary cell involved in acute inflammation. The plasma cell is involved in chronic inflammation. The lymphocyte is involved in chronic inflammation. REF: White Blood Cells in the Inflammatory Response, pages 38-39 OBJ: 6 47. Each statement regarding the atrophy of tissue cells is true except one. Which is the

exception? a. Atrophied cells are capable of returning to their normal size after stress is removed. b. Atrophy can occur with changes in cellular growth, malnutrition, ischemia, or hormonal changes. c. Atrophy can be present in the muscular wasting that occurs in some chronic diseases that do not allow for mobility. d. Atrophy occurs in the smooth muscles of the uterus and the mammary glands in response to pregnancy. ANS: D

Hypertrophy occurs in the smooth muscles of the uterus and the mammary glands in response to pregnancy. All other statements are true. REF: Reactive Tissue Response, page 43

OBJ: 8

48. If the source of injury has been completely removed, the inflammation and immune responses

in the tissues are completed in which time frame? a. One day after removal of injury b. Two days after removal of injury


c. Seven days after removal of injury d. Two weeks after removal of injury ANS: C

If the source of injury has been completely removed, the inflammation and immune responses in the tissues are completed in approximately 7 days. The day after injury removal, acute inflammation takes place in the area of future repair. Two days after removal of injury, fibroplasia, angiogenesis, the formation of granulation tissue, and epithelialization occur. Two weeks after removal of injury, matured fibrous connective tissue or scar tissue occurs. REF: Seven Days After Injury, page 44

OBJ: 9

49. Repair of bone injury is similar to the process that takes place in fibrous connective tissue

except that it involves the creation of bone tissue. The removal of osteoblast-producing tissues and excessive movement of the bone promote bone healing. a. Both statements are true. b. Both statements are false. c. The first statement is true; the second is false. d. The first statement is false; the second is true. ANS: C

The removal of osteoblast-producing tissues and excessive movement of the bone can interrupt healing. The first statement is true and the second is false. REF: Bone Tissue Repair, page 45

OBJ: 10

50. In cases of healing, if an infected injury is left open and the edges are not surgically joined

until the infection is controlled; this process is referred to as healing by _________ intention. a. primary b. secondary c. tertiary ANS: C

Healing by tertiary intention occurs when an infected injury is left open and the edges are not surgically joined until the infection is controlled. Healing by primary intention refers to the healing of an injury in which little loss of tissue takes place. Healing by secondary intention involves injury in which tissue is lost; thus the edges of the injury cannot be joined by healing. REF: Healing by Tertiary Intention, page 45

OBJ: 11

51. Each factor may impair healing except one. Which is the exception? a. Tobacco use b. Staphylococcus c. Nutritional supplements d. Renal failure ANS: C

The use of nutritional supplements has not been shown to impair healing. Tobacco use has been shown to impair healing. Staphylococcus infection has been shown to impair healing. Renal failure has been shown to impair healing. REF: Factors that Impair Healing, page 46

OBJ: 12


52. A tooth must be extracted if internal root resorption is present and a perforation occurs. a. Both the statement and reason are correct and related. b. Both the statement and reason are correct but not related. c. The statement is correct, but the reason is not. d. The statement is not correct, but the reason is correct. e. Neither the statement nor the reason is correct. ANS: A

A tooth must be extracted if internal root resorption is present and a perforation occurs. Both the statement and reason are correct and related. REF: Tooth Resorption, page 64

OBJ: 13

53. A wedge-shaped defect at the cervical area of a tooth, the cause of which is related to

microfracture of the tooth structure in areas of concentration of stress, is termed a. attrition. b. erosion. c. abrasion. d. abfraction. ANS: D

Abfraction is the result of biomechanical forces on the teeth. Attrition is defined as the wearing away of tooth structure during mastication. Erosion is the loss of tooth structure from chemical action. Abrasion is a pathologic wearing of tooth structure resulting from a repetitive mechanical habit. REF: Abfraction, page 47

OBJ: 14

54. Which statement is characteristic of erosion? a. It is a pathologic wearing away of tooth structure that results from a repetitive

mechanical habit. b. It is caused by local factors such as occlusal interferences in combination with

stress and tension. c. If tooth structure is lost around a restoration, the restoration will appear raised

from the surrounding demineralized tooth structure. d. Its first clinical sign is the disappearance of mamelons on the anterior teeth and the

flattening of occlusal cusps on the molar teeth. ANS: C

This phenomenon is seen in erosion and is not seen in abrasion, bruxism, or attrition. This is the definition of abrasion. This describes bruxism. This describes attrition. REF: Erosion, pages 48-49

OBJ: 15

55. Aspirin burn to the oral mucosa appears as a. white. b. pigmented. c. bulbous. d. papillary. ANS: A


Aspirin burn causes the tissue to become necrotic and appears white. Aspirin burn does not appear pigmented. Aspirin burn does not appear bulbous. Aspirin burn does not appear papillary. REF: Aspirin Burn, page 49OBJ:

17

56. The most likely cause of a ranula is a. inflammation of gland tissue. b. blockage of blood supply. c. trauma to a minor duct. d. salivary duct obstruction. ANS: D

Salivary gland obstruction is the most likely cause of a ranula. Inflammation of salivary gland tissue is referred to as sialadenitis. Necrotizing sialometaplasia results from blockage of the blood supply. The major cause of a mucocele is trauma to a minor duct. REF: Mucous Retention Lesions, page 56

OBJ: 19

57. Each habit is most likely to result in frictional keratosis except one. Which is the exception? a. Chronic cheek biting b. Chewing on an edentulous ridge c. Cigarette smoking d. Tongue chewing ANS: C

Frictional keratosis is not associated with cigarette smoking. Chronic cheek biting can result in frictional keratosis. Frictional keratosis results from chronic chewing on an edentulous ridge. Tongue chewing can result in frictional keratosis. REF: Frictional Keratosis, page 52

OBJ: 17

58. This sessile-based lesion is on the gingiva of a 13-year-old female. It is soft to palpation and

bleeds easily. The accurate diagnosis for this lesion is a. peripheral giant cell granuloma. b. pyogenic granuloma. c. traumatic neuroma. d. irritation fibroma. ANS: B

Pyogenic granulomas are commonly found in teenagers and exhibit a lesion that is soft to palpation and bleeds easily. Peripheral giant cell granuloma does not exhibit these characteristics. Traumatic neuroma does not exhibit these characteristics. Irritation fibroma does not exhibit these characteristics. REF: Pyogenic Granuloma, page 58

OBJ: 19

59. These elongated folds of tissue are a result of irritation from an ill-fitting denture. The

accurate diagnosis for this lesion is a. palatal papillomatosis. b. gingival hyperplasia. c. chronic hyperplastic pulpitis.


d. epulis fissuratum. ANS: D

Epulis fissuratum (denture-induced fibrous hyperplasia) consists of elongated folds of tissue as a result of irritation from an ill-fitting denture. Palatal papillomatosis is seen on the palate. Gingival hyperplasia is an enlargement of the gingiva. Chronic hyperplastic pulpitis is an excessive proliferation of chronically inflamed dental pulp tissue. REF: Denture-induced Fibrous Hyperplasia, page 60

OBJ: 18

60. The decrease in size and function of a cell, tissue, organ or body describes a. atrophy. b. hypertrophy. c. hyperemia. d. regeneration. ANS: A

Atrophy is defined as the decrease in size and function of a cell, tissue, organ, or whole body. Hypertrophy is the enlargement of a tissue resulting from an increase in the size of its individual cells. Hyperemia is an excess of blood within blood vessels in a part of the body. Regeneration is the process whereby injured tissue is replaced with tissue identical to that present before the injury. REF: Vocabulary, page 32

OBJ: 8

61. Innate or natural defenses used to protect the body include the following except one. Which is

the exception? a. Intact skin b. Components of saliva c. Drainage of an abscess d. Stomach acid ANS: C

Drainage of an intraoral abscess is not an innate or natural defense. Intact skin is an inborn defense that acts as a physical barrier. Components of saliva have antimicrobial activity. Stomach acid kills most of the microorganisms that are taken into the body. REF: Innate Defenses, page 34

OBJ: 4

62. Hyperemia is responsible for which two clinical signs of inflammation? a. Erythema and swelling b. Erythema and heat c. Swelling and pain d. Pain and loss of normal tissue function ANS: B

The increased blood flow of hyperemia causes the redness (erythema) and heat production. Swelling is resulting from the permeability of the microcirculation leading to exudate formation in the tissues. Swelling is resulting from exudate formation, which may cause pressure on the nerves (pain). Loss of tissue function is associated with swelling and pain. REF: Microscopic Events of Inflammation and Clinical Signs, page 35


OBJ: 5 63. An increase in blood viscosity reveals a. blood moving quickly through the vessels. b. decreased amounts of red blood cells. c. increase in plasma fluid. d. thicker blood unable to flow as normal. ANS: D

Viscosity is increased when the blood is thicker and cannot flow as easily. Viscosity refers to the thickness of the blood; an increase in viscosity does not move blood quicker through the vessels. Viscosity is not related to a decrease in red blood cells. Viscosity is not related to an increase in plasma fluid. REF: Microscopic Events of Inflammation and Clinical Signs, page 35 OBJ: 5 64. The fluid with a high protein content that leaves the microcirculation during the inflammatory

process is termed a. transudate. b. margination. c. exudate. d. pavementing. ANS: C

Exudate is the fluid with a high protein content that leaves the microcirculation during an inflammatory response. Transudate is the fluid component of blood that passes through the endothelial cell walls of the microcirculation. Margination is the movement of the white blood cells to the periphery of the blood vessel. Pavementing is the adherence of white blood cells to blood vessel walls during inflammation. REF: Microscopic Events of Inflammation and Clinical Signs, page 35 OBJ: 4 65. All of the statements refer to chronic inflammation except one. Which is the exception? a. Duration may last weeks, months or years b. Tissue returns to its original state c. Heals less readily d. Associated with functional deficiencies ANS: B

The tissue may return to its original state with acute inflammation. Duration may last weeks, months, or indefinitely with chronic inflammation. Because of its prolonged duration, the tissue heals less readily with chronic inflammation. Chronic inflammation is also associated with more serious functional deficiencies. REF: Inflammation, page 34

OBJ: 7

66. The fluid component of blood in which the blood cells are suspended and is composed mainly

of water and proteins is termed a. plasma. b. serous fluid.


c. transudate. d. exudate. ANS: A

Plasma is the fluid component of blood in which blood cells are suspended. Exchange of oxygen and nutrients occurs as plasma passes between the endothelium lining. Serous fluid is a secretion with a watery consistency. Transudate is the fluid component of blood that passes through the endothelial cell walls of the microcirculation. Exudate is the fluid with a high protein content that leaves the microcirculation during an inflammatory response. REF: Microscopic Events of Inflammation and Clinical Signs, page 34 OBJ: 4 67. Matured fibrous tissue seen 2 weeks after an injury appears whiter or paler at the surface of

the repaired injury for what two reasons? 1. Increased granulation tissue 2. Increased numbers of collagen fibers 3. Enlarged lymphocyte population 4. Decreased vascularity a. 1 and 2 b. 1 and 3 c. 2 and 3 d. 2 and 4 ANS: D

Increased numbers of collagen fibers and decreased vascularity is seen 2 weeks after an injury when the scar tissue has formed. Granulation tissue has more capillaries and fibroblasts and clinically appears as a vivid pink or red, not whiter or paler. Granulation tissue has more capillaries and fibroblasts and clinically appears as a vivid pink or red, not whiter or paler. Enlarged lymphocyte population has no bearing on the injury site 2 weeks later. REF: Microscopic Events During Repair, page 44

OBJ: 9

68. Local factors that impair healing include the following except one. Which is the exception? a. Hemorrhage into the tissues b. Excessive moving of the injured tissue c. Increased scar tissue d. Poor blood supply ANS: C

Increased scar tissue is not a local factor that affects healing. Hemorrhage into the tissues may cause a hematoma that can impair healing. Excessive moving of the injured tissue can impair healing. Poor blood supply is a local factor that can impair healing. REF: Types of Repair, page 45 69. Bruxism is a form of a. abrasion. b. attrition. c. erosion.

OBJ: 12


d. abfraction. ANS: B

Attrition is the wearing away of tooth structure during mastication, and bruxism is the grinding of teeth together. Abrasion is the pathologic wearing of tooth structure from a mechanical habit. Erosion is the loss of tooth structure from chemical action. Abfraction is a wedge-shaped defect seen at the cervical area of teeth. REF: Traumatic Injuries to Teeth, page 46

OBJ: 14

70. Which permanent tooth would most likely demonstrate abfraction? a. #7 b. #13 c. #18 d. #27 ANS: B

Abfraction is most likely seen on premolar teeth. REF: Traumatic Injuries to Teeth, page 47

OBJ: 14

71. Your patient presents with tooth structure that has been lost around occlusal restorations. The

amalgam restorations appear raised from the surrounding demineralized tooth structure. Identify the traumatic injury to the teeth that has occurred: a. Attrition b. Abrasion c. Abfraction d. Erosion ANS: D

If erosion occurs in a restored tooth, the tooth structure is lost around the restoration, making the restoration appear raised. With attrition, the restoration would also be worn down with the tooth surface. With abrasion, the restoration would also be worn down with the tooth surface. With abfraction, the defect is seen in the cervical area of the teeth. REF: Traumatic Injuries to Teeth, page 48

OBJ: 14

72. Frequent vomiting seen in patients with bulimia can result in generalized erosion of which

tooth surface? a. Buccal b. Lingual c. Mesial d. Distal ANS: B

The frequent vomiting in a patient with bulimia results in generalized erosion of the lingual surfaces of teeth. Erosion is not typically seen on buccal surfaces. Erosion is not typically seen on mesial surfaces. Erosion is not typically seen on distal surfaces. REF: Traumatic Injuries to Teeth, page 48 73. What is the treatment for a patient with linea alba?

OBJ: 14


a. b. c. d.

Brush the affected area; recommend good oral hygiene Antibiotic therapy Gentle irrigation No treatment required

ANS: D

No treatment is indicated for linea alba. Brushing the area and oral hygiene do not affect linea alba. Antibiotic therapy is not recommended for linea alba. Gentle irrigation is not protocol for linea alba. REF: Lesions From Self-induced Injuries, page 52

OBJ: 17

74. Chronic hyperplastic pulpitis presents with all symptoms except one. Which is the exception? a. Pain b. Can occur in primary and permanent teeth c. Seen as a pink nodule protruding from the tooth d. Occurs in teeth with large carious lesions ANS: A

Since hyperplastic tissue contains few nerves, chronic hyperplastic pulpitis is usually asymptomatic. Chronic hyperplastic pulpitis can occur in both dentitions. Chronic hyperplastic pulpitis appears as a red to pink nodule protruding from the affected tooth. Chronic hyperplastic pulpitis does occur in teeth with large open carious lesions. REF: Reactive Connective Tissue Hyperplasia, page 62

OBJ: 18


Chapter 03: Immunity and Immunologic Oral Lesions Ibsen: Oral Pathology for the Dental Hygienist, 7th Edition MULTIPLE CHOICE 1. The breakdown of cellular adhesion between epithelial cells is termed a. cell-mediated immunity. b. acantholysis. c. Nikolsky sign. d. anaphylaxis. ANS: B

Acantholysis is the dissolution of the intracellular bridges of the prickle cell layer of epithelium. Cell-mediated immunity is immunity in which the predominant role is played by T-lymphocytes. Nikolsky sign occurs when the superficial epithelium separates easily from the basal layer on exertion of firm, sliding, manual pressure. Anaphylaxis is a severe type of systemic hypersensitivity reaction. REF: Pemphigus, page 101

OBJ: 1

2. Which protein molecule is produced by plasma cells and is also termed an immunoglobulin? a. An autoimmune disease b. A natural killer (NK) cell c. Rheumatoid factor d. An antibody ANS: D

An antibody is a protein molecule, also called an immunoglobulin, that is produced by plasma cells and reacts with a specific antigen. An autoimmune disease is a disease characterized by tissue injury caused by a humoral or cell-mediated response against constituents of the body’s own tissues. An NK cell is a lymphocyte that is part of the body’s innate immunity. Rheumatoid factor is a protein, immunoglobulin M (IgM), found in serum and detected in laboratory tests. It is associated with rheumatoid arthritis and other autoimmune diseases. REF: Vocabulary, page 79 | B-cell Lymphocyte, page 81

OBJ: 1

3. Which lymphocyte matures without passing through the thymus and later can develop into a

plasma cell that produces antibodies? a. T-lymphocyte b. B-lymphocyte c. Macrophage d. LE cell ANS: B

The B-lymphocyte matures without passing through the thymus and later can develop into a plasma cell that produces antibodies. The T-lymphocyte matures in the thymus before migrating to tissues. A macrophage is a large tissue-bound mononuclear phagocyte derived from monocytes circulating in the blood. The LE cell is an atypical mature neutrophil characteristic of lupus erythematosus and other autoimmune diseases.


REF: Vocabulary, page 79 | B-cell Lymphocyte, page 81

OBJ: 3

4. In which type of immune response do B-lymphocytes and antibodies play the predominant

role? a. Cell-mediated immunity b. Immune complex c. Humoral immunity d. Passive immunity ANS: C

In humoral immunity the B-lymphocytes and antibodies play the predominant role. In cellmediated immunity, the T-lymphocyte plays the predominant role. The immune complex is a combination of antigen and antibody. Passive immunity occurs when antibodies produced by another person are used to protect an individual from an infectious disease. REF: Major Divisions of the Immune Response, page 84

OBJ: 2

5. Which is not a specific component of immunity? a. A humoral response b. A cell-mediated response c. Memory d. Pavementing ANS: D

Pavementing is the adherence of white blood cells to the walls of a blood vessel during an inflammatory response. Humoral immunity involves the production of antibodies. The Blymphocyte is the predominant cell. Cell-mediated immunity involves the T-lymphocyte. Memory is an important function of the immune system. Certain lymphocytes retain the memory of an antigen after an initial encounter, allowing for faster immune responses. REF: Major Divisions of the Immune Response; Memory and Immunity, pages 80, 84 OBJ: 6 6. An example of natural passive immunity occurs when a. antibodies from a mother pass through the placenta to the developing fetus. b. a microorganism causes a disease. c. a person receives a vaccination. d. a person is immunized. ANS: A

Natural passive immunity occurs when antibodies from a mother pass through the placenta to the developing fetus. Active immunity occurs naturally when a microorganism causes a disease. Active acquired immunity occurs as a result of vaccination. After vaccination, the immune system produces a stronger response and prevents the development of the disease. This production of acquired immunity is called immunization. REF: Types of Immunity, page 85

OBJ: 7

7. Which is an example of type I hypersensitivity? a. Immune complexes formed between microorganisms and antibody in the

circulating blood b. Asthma


c. Autoimmune hemolytic anemia d. Serum sickness ANS: B

Asthma is an example of type I hypersensitivity. In type III hypersensitivity, immune complexes are formed between microorganisms and antibody in the circulating blood. Autoimmune hemolytic anemia is a type II hypersensitivity reaction. Serum sickness is a classic example of a type III hypersensitivity reaction. REF: Type I Hypersensitivity, page 85

OBJ: 8

8. Which route of drug administration causes the most significant hypersensitivity reaction? a. Topical b. Patch c. Parenteral d. Oral ANS: C

When the reaction occurs after parenteral administration (injection), it may be more severe because the allergen can be carried quickly to many parts of the body through the bloodstream. It takes more time for topically applied drugs to enter the bloodstream compared with parenterally applied (injected) drugs. It takes more time for a drug to enter the bloodstream via a patch compared with parenteral administration. It takes more time for oral medications to enter the bloodstream than parenterally applied drugs. REF: Hypersensitivity to Drugs, page 86 OBJ: 8 9. Which is a condition that involves a deficiency in number, function, or interrelationships of

the involved white blood cells and their products? a. Autoimmune disease b. Leukocytosis c. Immunodeficiency d. Anaphylaxis ANS: C

Immunodeficiency is a type of immunopathologic condition that involves a deficiency in number, function, or interrelationships of the involved white blood cells and their products. Autoimmune disease is characterized by tissue injury caused by a humoral or cell-mediated immune response against constituents of the body’s own tissues. Leukocytosis is an increase in the circulating white blood cells. It is a systemic sign of an inflammatory response. Anaphylaxis is a severe type of hypersensitivity reaction. REF: Immunodeficiency, page 87

OBJ: 8

10. What is the most common precipitating factor in the development of aphthous ulcers? a. Allergy b. Trauma c. Systemic disease d. Citrus foods ANS: B


Trauma is the most common precipitating factor in the development of aphthous ulcers. Allergies do not cause aphthous ulcers. Aphthous ulcers do occur in association with certain systemic diseases, but they are not the most common precipitating factor. Citrus foods can trigger episodes of aphthous ulcers, but they are not the most common precipitating factor. REF: Aphthous Ulcers, Recurrent Aphthous Ulcers, page 87

OBJ: 11

11. Certain recurrent ulcers appear on movable mucosa. These ulcers are discrete, round-to-oval,

3 to 5 mm in diameter and exhibit a yellowish-white fibrin center surrounded by an erythematous halo. These ulcers are termed a. major aphthous ulcers. b. Sutton disease. c. herpetiform aphthous ulcers. d. minor aphthous ulcers. ANS: D

Minor aphthous ulcers are recurrent and appear on movable mucosa. They are 3 to 5 mm in diameter and have a yellowish-white fibrin center surrounded by an erythematous halo. Major aphthous ulcers are deeper in the tissue, are larger (5 to 10 mm), take much longer to heal, and often heal with scarring. Sutton disease is the same as major aphthous ulcers. Herpetiform aphthous ulcers are very tiny (1 to 2 mm). The size of this type of aphthous ulcer is similar to the size of ulcers caused by the herpes simplex virus. REF: Types of Aphthous Ulcers, page 87

OBJ: 11

12. Which ulcer may require a biopsy to make the diagnosis? a. Minor aphthous b. Major aphthous c. Herpetiform aphthous d. Behçet syndrome ANS: B

Major aphthous ulcers sometimes require a biopsy to rule out other causes of ulceration such as squamous cell carcinoma or deep fungal infections. Minor aphthous ulcers are diagnosed through clinical appearance, location, and complete patient history. Herpetiform aphthous ulcers resemble those associated with the herpes simplex virus, but there are no systemic signs or symptoms as in primary herpes simplex infection. Behçet syndrome is a multisystem disorder characterized by numerous clinical manifestations. Although there are oral ulcerations, the diagnosis requires the presence of other features such as genital, ocular, and skin lesions. REF: Types of Aphthous Ulcers, page 87

OBJ: 11

13. Antihistaminic drugs are the principal method of treatment for a. urticaria. b. minor aphthous ulcers. c. herpetiform aphthous ulcers. d. contact mucositis. ANS: A


Urticaria, also called hives, appears as well-demarcated areas of swelling on the skin accompanied by pruritus. The release of the chemical mediator histamine causes the increased vascular permeability. Antihistaminic drugs are used to treat urticaria. Minor aphthous ulcers heal spontaneously in 7 to 10 days. They may be treated with topical corticosteroid or nonsteroidal antiinflammatory medications. Herpetiform aphthous ulcers do not respond to antihistaminic drugs. They may respond to corticosteroid therapy or topical liquid tetracycline. Contact mucositis results from direct contact of an allergen with the mucosa. Treatment begins with removal of the cause. REF: Urticaria and Angioedema, Treatment and Prognosis, page 90 OBJ: 13 14. A prodromal period begins a. 1 week before the onset of ulcers. b. 1 to 2 days before the onset of ulcers. c. the day the ulcers erupt. d. 7 to 10 days before ulcers erupt. ANS: B

The prodromal period occurs 1 to 2 days before ulcers appear. There are a burning sensation and soreness where the ulcers will appear. The prodromal period before ulceration is only 1 to 2 days. Prodromal symptoms should not occur 1 week before ulceration. The prodromal period does not occur on the day of ulceration. The prodromal period precedes ulceration by only 1 to 2 days. Prodromal symptoms should not occur 7 to 10 days before ulceration. REF: Types of Aphthous Ulcers, page 87

OBJ: 11

15. A condition thought to be a hypersensitivity reaction is characterized by skin and mucous

membrane lesions. The skin lesions are referred to as target, iris, or bull’s eye lesions. The lip and mucosal lesions are more extensive and painful and often appear bloody and crusted. The intraoral lesions are extensive superficial ulcerations. The onset is explosive. On the basis of these clinical features, what condition is suspected? a. Erythema multiforme b. Fixed drug reaction c. Behçet syndrome d. Angioedema ANS: A

Erythema multiforme is a hypersensitivity reaction that affects the skin and mucous membranes. Bloody crusted lips are a common presentation. The onset is explosive, and the skin lesions are referred to as target, iris, or bull’s eye lesions. Fixed drug eruptions are lesions that appear in the same site each time a drug is introduced. There may be a single red patch or macule on the skin. Mucous membranes are rarely involved. Behçet syndrome is a chronic recurrent autoimmune disease consisting primarily of oral ulcers, genital ulcers, and ocular inflammation. Angioedema appears as a diffuse swelling of tissue caused by permeability of deeper blood vessels. REF: Erythema Multiforme, page 91

OBJ: 14

16. Wickham striae is a term used to describe the oral mucosal lesions of a. linea alba.


b. frictional keratosis. c. lichen planus. d. erythema multiforme. ANS: C

Wickham striae describe the slender white lines seen in lichen planus. Linea alba is the white line on the buccal mucosa that extends anteroposteriorly along the occlusal plane. Frictional keratosis occurs when there is chronic cheek and tongue chewing and chewing on edentulous ridges. Erythema multiforme is an acute self-limited disease that affects the skin and mucous membranes. The skin lesions are described as iris, target, or bull’s eye lesions. The mucosal lesions are superficial ulcerations. REF: Lichen Planus, page 93

OBJ: 15

17. The diagnosis of lichen planus is made on the basis of a. clinical and radiographic appearance. b. clinical and histologic appearance. c. history of the condition. d. therapeutic medication. ANS: B

Diagnosis of lichen planus is made on the basis of clinical appearance and histologic findings of biopsy tissue. Radiographic findings do not contribute to the diagnosis of lichen planus. History of the condition is not definitive enough for the diagnosis of lichen planus. Topical and systemic corticosteroids have been used in the treatment of lichen planus but not to establish the diagnosis. REF: Types of Lichen Planus, Diagnosis, page 94

OBJ: 15

18. Which condition is not a component of Reiter syndrome? a. Arthritis b. Geographic tongue–like lesions c. Urethritis d. Wickham striae ANS: D

Wickham striae are associated with lichen planus. Arthritis is a component of Reiter syndrome. Oral mucosal lesions that resemble geographic tongue have been described in patients with Reiter syndrome. Urethritis is associated with Reiter syndrome. REF: Reactive Arthritis (Reiter Syndrome), page 95

OBJ: 16

19. An antigenic marker termed HLA-B27 is present in most patients with a. lichen planus. b. Reiter syndrome. c. erythema multiforme. d. Stevens–Johnson syndrome. ANS: B


An antigenic marker called HLA-B27 is present in most patients with Reiter syndrome. Lichen planus is diagnosed through the clinical and histologic appearance of biopsy tissue. Erythema multiforme is thought to be a hypersensitivity reaction involving the skin and mucous membranes. Stevens–Johnson syndrome is the most severe form of erythema multiforme. REF: Reactive Arthritis (Reiter Syndrome), page 95

OBJ: 16

20. The acute disseminated form of Langerhans cell disease is termed a. eosinophilic granuloma. b. Hand–Schüller–Christian disease. c. Letterer–Siwe disease. d. Reiter syndrome. ANS: C

Letterer–Siwe disease is the acute disseminated form of Langerhans cell disease. Eosinophilic granuloma is a solitary or chronic localized form of Langerhans cell disease. Hand–Schüller– Christian disease is the chronic disseminated or multifocal form of Langerhans cell disease. Reiter syndrome is not a form of Langerhans cell disease. REF: Langerhans Cell Histiocytosis (Langerhans Cell Disease), page 96 OBJ: 17 21. Which form of Langerhans cell disease resembles lymphoma, affects children younger than

age 3, and has a rapidly fatal course? a. Letterer–Siwe disease b. Hand–Schüller–Christian disease c. Eosinophilic granuloma d. Chronic localized form ANS: A

Letterer–Siwe resembles lymphoma, affects children under 3 years, and has a rapidly fatal course. It is the most severe form of Langerhans cell disease. Hand–Schüller–Christian disease is the multifocal form of Langerhans cell disease. It occurs in children younger than 5 years. A classic triad is seen in 25% of patients. Eosinophilic granuloma affects older children and young adults. The chronic localized form is the same as eosinophilic granuloma. REF: Langerhans Cell Histiocytosis (Langerhans Cell Disease), page 96 OBJ: 17 22. The classic triad of symptoms seen in 25% of patients with Hand–Schüller–Christian disease

includes all conditions except a. well-defined or punched-out radiolucent areas in the skull. b. exophthalmos. c. cyclic neutropenia. d. diabetes insipidus. ANS: C


Cyclic neutropenia is an inherited disorder characterized by a cyclic decrease in the number of circulating neutrophils. Aphthous ulcers, gingival recession, and alveolar bone loss can be seen in association with this systemic condition. Well-defined or punched-out radiolucent areas in the skull are part of the triad of symptoms in chronic disseminated Hand–Schüller– Christian disease. Exophthalmos is a part of the triad of symptoms in chronic disseminated Hand–Schüller–Christian disease. Diabetes insipidus is part of the triad of symptoms in chronic disseminated Hand–Schüller–Christian disease. REF: Langerhans Cell Histiocytosis (Langerhans Cell Disease), page 96 OBJ: 12 23. Sjögren syndrome is a(n) a. autoimmune disease that affects the salivary and lacrimal glands. b. allergic reaction. c. form of aphthous ulcers. d. type of Langerhans cell disease. ANS: A

Sjögren syndrome is an autoimmune disease that affects the salivary and lacrimal glands. Sjögren syndrome is not an allergic reaction. Sjögren syndrome is not a form of aphthous ulcers. Sjögren syndrome is not a type of Langerhans cell disease. REF: Sjögren Syndrome, page 97

OBJ: 18

24. The most common oral manifestation of Sjögren syndrome is a. xerostomia. b. xerophthalmia. c. epimyoepithelial islands. d. aphthous ulcers. ANS: A

Xerostomia, or dry mouth, is the most common oral manifestation of Sjögren syndrome. Xerophthalmia is dry eyes caused by lack of lacrimal flow. Epimyoepithelial islands are seen histologically in Sjögren syndrome, but are not the most common oral manifestation. Aphthous ulcers are not associated with Sjögren syndrome, but are seen more commonly in Behçet syndrome, another autoimmune disease. REF: Sjögren Syndrome, page 97

OBJ: 18

25. Primary Sjögren syndrome occurs when a. another autoimmune disease accompanies salivary and lacrimal gland

involvement. b. only salivary and lacrimal gland involvement occurs without the presence of

another autoimmune disease. c. rheumatoid arthritis is present. d. Raynaud phenomenon is present. ANS: B


Primary Sjögren syndrome occurs when there is only salivary and lacrimal gland involvement without the presence of another autoimmune disease. Secondary Sjögren syndrome occurs when another autoimmune condition accompanies the salivary and lacrimal gland involvement. Rheumatoid arthritis is an autoimmune disease that can be a component of secondary Sjögren syndrome. Twenty percent of patients with Sjögren syndrome have Raynaud phenomenon. It is characterized by pallor of the skin that results from vasoconstriction and reduced blood flow. The toes and fingers are affected. REF: Sjögren Syndrome, page 97

OBJ: 18

26. The eye damage that occurs in Sjögren syndrome is termed a. xerophthalmia. b. keratoconjunctivitis sicca. c. conjunctivitis. d. burning eyes. ANS: B

Keratoconjunctivitis sicca is damage to the eye in Sjögren syndrome. It is confirmed by eye examination. Xerophthalmia, or dry eyes, may be present in Sjögren syndrome. Conjunctivitis is inflammation of the conjunctiva. Burning eyes may also be a component of Sjögren syndrome. REF: Sjögren Syndrome, page 97

OBJ: 18

27. Diagnosis of systemic lupus erythematosus (SLE) is made on the basis of a. skin lesions. b. multiorgan involvement and the presence of antinuclear antibodies in the serum. c. arthritis and arthralgia. d. shortness of breath. ANS: B

Diagnosis of systemic SLE is made on the basis of multiorgan involvement and the presence of antinuclear antibodies in the serum. Skin lesions occur in 85% of individuals, but the diagnosis is not made on the basis of skin lesions. Arthritis and arthralgia may be components of SLE, but are not sufficient to make the diagnosis. Shortness of breath may be a component of SLE, but is not significant to the diagnosis. REF: Systemic Lupus Erythematosus, Diagnosis, page 99

OBJ: 18

28. The most characteristic skin lesion in SLE is described as a. a blister. b. urticaria. c. a butterfly rash. d. a bulla. ANS: C

The classic skin lesion in SLE is described as a butterfly rash. A blister is not the classic skin lesion in SLE. Urticaria is associated with a hypersensitivity reaction. A bulla may occur in SLE, but it is not the most characteristic skin lesion. REF: Systemic Lupus Erythematosus, Diagnosis, page 99

OBJ: 18


29. The histologic appearance of lesions in SLE may resemble a. an ulcer. b. lichen planus. c. pemphigus vulgaris. d. erythema multiforme. ANS: B

The histologic appearance of lesions in SLE may resemble lichen planus. An ulcer appears as a break in the epithelium and is histologically nonspecific for SLE or lichen planus. Pemphigus vulgaris is a severe progressive autoimmune disease characterized by intraepithelial blister formation that results from breakdown of the cellular adhesion between epithelial cells. In erythema multiforme, the microscopic appearance is nonspecific. REF: Systemic Lupus Erythematosus, Diagnosis, page 99

OBJ: 18

30. Which one is not a characteristic feature of pemphigus vulgaris? a. Epithelial acantholysis b. Bullae c. Tzanck cells d. Occurrence most commonly in females ANS: D

No sex predilection exists in pemphigus vulgaris. Epithelial acantholysis occurs in pemphigus vulgaris. Bullae do appear in pemphigus vulgaris. Tzanck cells, or acantholytic cells, appear in pemphigus vulgaris as rounded epithelial cells and are present in the area of separation of epithelial cells. REF: Pemphigus Vulgaris, pages 101-102

OBJ: 18

31. Treatment for pemphigus vulgaris involves a. high doses of systemic corticosteroids. b. antihistamines. c. antibiotics. d. antiviral drugs. ANS: A

High doses of systemic corticosteroids are used in the treatment of pemphigus vulgaris. Antihistamines are not used to treat pemphigus vulgaris. Antibiotics are not used to treat pemphigus vulgaris. Antiviral drugs are not used to treat pemphigus vulgaris. REF: Pemphigus Vulgaris, Treatment and Prognosis, page 102

OBJ: 18

32. The most common oral site for cicatricial pemphigoid is the a. floor of the mouth. b. gingiva. c. palate. d. tongue. ANS: B

The gingiva is the most common site for cicatricial pemphigoid. The floor of the mouth is not a common site for cicatricial pemphigoid. The palate is not a common site for cicatricial pemphigoid. The tongue is not a common site for cicatricial pemphigoid.


REF: Mucous Membrane Pemphigoid, page 102

OBJ: 18

33. The diagnosis of cicatricial pemphigoid is made on the basis of a. the clinical appearance of the lesions. b. Nikolsky sign. c. biopsy and histologic examination. d. response to corticosteroids. ANS: C

Biopsy and histologic examination are used to diagnose cicatricial pemphigoid. The clinical appearance of lesions is not sufficient for the diagnosis of cicatricial pemphigoid. The Nikolsky sign is not sufficient to determine the diagnosis of cicatricial pemphigoid. Although corticosteroids are the recommended treatment for cicatricial pemphigoid, diagnosis of the condition is not made through therapeutic diagnosis. REF: Mucous Membrane Pemphigoid, Diagnosis, page 102

OBJ: 18

34. Pemphigus vulgaris differs histologically from cicatricial pemphigoid in that pemphigus

vulgaris involves a. degeneration of the epithelium. b. connective tissue fragmentation. c. significant numbers of inflammatory cells. d. scarring. ANS: A

The lesions of pemphigus vulgaris histologically exhibit acantholysis, which is degeneration of the epithelium. In pemphigus vulgaris, the basal cells of the epithelium and the basement membrane remain attached to the underlying connective tissue. Inflammatory cells are present in both cicatricial pemphigoid and pemphigus vulgaris. Scarring occurs in lesions associated with cicatricial pemphigoid. REF: Pemphigus Vulgaris, page 101

OBJ: 18

35. The diagnosis of Behçet syndrome requires which considerations? a. Two of the three principal manifestations b. The presence of all three manifestations c. A biopsy d. A history of pemphigus vulgaris ANS: A

Behçet syndrome is a chronic recurrent autoimmune disease consisting primarily of oral ulcers, genital ulcers, and ocular inflammation. Two of these three principal manifestations must be present for the diagnosis. Only two principal manifestations must be present for the diagnosis of Behçet syndrome. A biopsy is not necessary for the diagnosis of Behçet syndrome. Pemphigus vulgaris is another severe progressive autoimmune disease. Pemphigus vulgaris is not associated with Behçet syndrome. REF: Behçet Syndrome, Diagnosis, page 103

OBJ: 18

36. The classic appearance of what condition shown below is a pattern of interconnecting slender

lines referred to as Wickham striae?


a. b. c. d.

Lichen planus Linea alba Erythema multiforme Reiter syndrome

ANS: A

The classic clinical appearance of lichen planus is a pattern of slender white interconnecting lines referred to as Wickham striae. Linea alba is the white line that develops on the buccal mucosa along the occlusal plane; it is more prominent in patients with clenching or bruxism habits. Erythema multiforme is an acute self-limiting disease affecting the skin and mucous membranes; it is thought to be a hypersensitivity reaction. The oral lesions in Reiter syndrome are characterized by aphthous-like ulcerations, erythematous lesions, and geographic tongue– like lesions. REF: Lichen Planus, page 93

OBJ: 15

37. In which condition is the Langerhans cell not the proliferating cell? a. Lichen planus b. Letterer–Siwe disease c. Hand–Schüller–Christian disease d. Eosinophilic granuloma ANS: A

In lichen planus, there is degeneration of the basal cell layer, sawtooth rete ridges, and a broad band of lymphocytes in the connective tissue immediately subjacent to the epithelium. Letterer–Siwe disease is an acute disseminated form of Langerhans cell disease. Hand– Schüller–Christian disease is a chronic disseminated or multifocal form of Langerhans cell disease. Eosinophilic granuloma is the solitary or chronic localized form of Langerhans cell disease. REF: Langerhans Cell Histiocytosis (Langerhans Cell Disease), page 96 OBJ: 15 38. The onset of this condition is explosive. It is characterized by skin and mucosal lesions. The

skin lesions are referred to as target, iris, or bull’s eye lesions. The mucosal lesions affect the buccal mucosa, lips, and tongue and appear as superficial ulcerations. The lip lesions are ulcerated, encrusted, and bloody. On the basis of this description, which condition is suspected? a. Lichen planus b. Erythema multiforme c. Contact dermatitis d. SLE ANS: B

The skin lesions of erythema multiform are described as target, iris, or bull’s eye lesions. The mucosal lesions are superficial ulcerations. The lip lesions are bloody and encrusted. In lichen planus, skin lesions are 2- to 4-mm papules. In contact dermatitis, skin lesions result from direct contact with the allergen. The lesions may be erythematous with swelling and vesicles. In SLE, the most classic skin lesion appears on the face and is described as a butterfly rash. The erythematous rash involves the cheeks and the bridge of the nose.


REF: Erythema Multiforme, page 91

OBJ: 15

39. This patient has Sjögren syndrome and severe xerostomia. Which clinical feature most

appropriately describes the changes on the tongue in this patient? a. Dryness b. Candidiasis c. Loss of filiform and fungiform papillae d. Fissures ANS: C

Loss of filiform and fungiform papillae is the main clinical feature that is responsible for the appearance of this patient’s tongue. The entire mouth is dry, but this is not the main clinical feature involving changes in the tongue. Candidiasis may be present, but it does not describe the changes seen in this patient’s tongue. Fissures can occur as a result of the dryness, but they are not the main clinical feature involving the tongue. REF: Sjögren Syndrome, page 97

OBJ: 18

40. A cell product produced by the cells involved in the immune response is a(n) a. attenuation. b. cytokine. c. dendritic cell. d. Langerhans cell. ANS: B

A cytokine is a cell product produced by cells involved in the immune response. Attenuation describes the reduction in the severity of a disease or the virulence of a pathogenic agent, as is done in the development of certain vaccines. A dendritic cell is a white blood cell that acts as an antigen-processing cell in the skin and mucosa. A Langerhans cell is a specialized dendritic cell, found in skin and mucosa, that is involved in the immune response. REF: Cellular Involvement in the Immune Response, page 81

OBJ: 1

41. The immune response differs from the inflammatory response because it has the capacity for

memory and responds more quickly to a foreign substance if encountered again. a. Both the statement and reason are correct and related. b. Both the statement and reason are correct but not related. c. The statement is correct but the reason is not. d. Neither the statement nor the reason is correct. ANS: A

The immune response differs from the inflammatory response because it has the capacity for memory and responds more quickly to a foreign substance if encountered again. Both the statement and reason are correct and related. REF: Acquired Immune Response, pages 80-81

OBJ: 2

42. The three main types of lymphocytes active during an immune response include the following

except one. Which one is the exception? a. T-cell b. NK cell


c. B-cell d. C-cell ANS: D

The C-cell is not a type of lymphocyte active during an immune response. The T-cell is a type of lymphocyte active during an immune response. The NK cell is a type of lymphocyte active during an immune response. The B-cell is a type of lymphocyte active during an immune response REF: Cellular Involvement in the Immune Response, page 81

OBJ: 3

43. The various types of T-cell lymphocytes include the T-helper cell, the T-suppressor cell, the

cytotoxic cell, and what other type of cell? a. T-enhancement cell b. Virulence cell c. Memory cell d. Oral cell ANS: C

The memory cell is a type of T-cell lymphocyte. The T-enhancement cell is not a type of T-cell lymphocyte. The virulence cell is a not a type of T-cell lymphocyte. The oral cell is not a type of T-cell lymphocyte. REF: T-cell Lymphocyte, page 83

OBJ: 4

44. Along with phagocytosis, the macrophage acts to process antigen material and present it on its

surface to the T-helper cell. This stimulates both types of lymphocytes to travel from the lymphoid tissue or surrounding blood vessels to the injury site. a. Both statements are true. b. Both statements are false. c. The first statement is true; the second is false. d. The first statement is false; the second is true. ANS: A

Both statements are true. REF: Macrophage, page 83

OBJ: 5

45. The major divisions of the immune response are the cell-mediated and humoral responses.

Both of these responses originate from what type of cell? a. Plasma cell b. Stem cell c. T-lymphocyte d. B-lymphocyte ANS: B

Both the cell-mediated response and the humoral response originate from the stem cell. The plasma cell is part of the humoral response, but not the cell-mediated response. The Tlymphocyte is part of the cell-mediated response, but not the humoral response. The Blymphocyte is part of the humoral response, but not the cell-mediated response. REF: Major Divisions of the Immune Response, Fig. 3.6, page 84


OBJ: 6 46. Antibodies pass through the placenta to a developing fetus and protect the newborn infant

from disease; this is termed ____________ immunity. a. natural active b. acquired passive c. acquired active d. natural passive ANS: D

Natural passive immunity is acquired when antibodies pass through the placenta to a developing fetus and protect the newborn infant from disease. Natural active immunity is acquired when a pathogenic microorganism causes the disease. Acquired passive immunity occurs through an injection of antibodies. Acquired active immunity occurs through immunization. REF: Types of Immunity, page 85

OBJ: 7

47. Anaphylaxis occurs as a result of what type of hypersensitivity? a. I b. II c. III d. IV ANS: A

Type I, or anaphylactic type, hypersensitivity is a reaction that occurs immediately, within minutes of exposure to a previously encountered antigen or allergen. Type II, or cytotoxic type, hypersensitivity occurs when an antibody combines with an antigen that is bound to the surface of tissue cells. Type III, or immune complex type, hypersensitivity is marked by the formation of immune complexes between microorganisms and antibody in the circulating blood. Type IV, or cell-mediated type, hypersensitivity involves a cell-mediated immune response rather than a humoral response that produces antibodies. REF: Type I Hypersensitivity, page 86

OBJ: 8

48. An example of an immunopathologic condition in which one’s own body cells are no longer

tolerated is a. contact mucositis. b. angioedema. c. autoimmune disease. d. contact dermatitis. ANS: C

Autoimmune disease occurs when one’s own body cells are no longer tolerated. Contact mucositis results from direct contact of an allergen with the oral mucosa or skin. Angioedema is a skin and oral mucosal hypersensitivity. Contact dermatitis occurs as a result of direct contact of an allergen with the oral mucosa or skin. REF: Autoimmune Diseases, page 86

OBJ: 9


49. Each condition is an example of an oral immunologic disorder except one. Which one is the

exception? a. Aphthous ulcers b. Erythema multiforme c. Fixed drug eruptions d. Osteomyelitis ANS: D

Osteomyelitis is an example of an infectious disease. Aphthous ulcers are an example of an oral immunologic disorder. Erythema multiforme is an example of an oral immunologic disorder. Fixed drug eruptions are an example of an oral immunologic disorder. REF: Aphthous Ulcers, page 87 | Erythema Multiforme, page 91 | Drug Eruption, page 91 OBJ: 10 50. The diagnosis of the type of aphthous ulcers is made on the basis of their distinctive clinical

appearance, the location of the lesion, and a a. patient history. b. therapeutic result. c. radiographic examination. d. laboratory test. ANS: A

A complete patient history is part of the final diagnosis of aphthous ulcers. Therapeutic results do not aid in the diagnosis of aphthous ulcers. Radiographic examinations are not diagnostic for aphthous ulcers. Laboratory results are not specific for any form of aphthous ulcer. REF: Types of Ulcers, Diagnosis, pages 87, 89

OBJ: 10

51. Which is a systemic disease associated with aphthous ulcers? a. Stafne bone cyst b. Necrotizing sialometaplasia c. Behçet syndrome d. Erythema multiforme ANS: C

Behçet syndrome is a disease characterized by aphthous ulcers, genital ulcers, ocular inflammation, and arthritis. Stafne bone cyst is a developmental disorder and is not considered a systemic disease. Necrotizing sialometaplasia is an inflammatory condition of the salivary glands. Erythema multiforme is not a systemic disease. REF: Behçet Syndrome, page 103

OBJ: 12

52. A patient presents with a diffuse swelling of the upper lip caused by permeability of deeper

blood vessels. The skin covering the swelling appears normal and not accompanied by itching. This is characteristic of what pathologic condition? a. Minor aphthous ulcer b. Lichen planus c. Angioedema d. Sjögren syndrome ANS: C


This description is indicative of angioedema. Minor aphthous ulcers are round-to-oval ulcers that exhibit a yellowish-white fibrin surface surrounded by a halo of erythema. Lichen planus appears as interconnecting white lines and circles on the oral mucosa or skin. Sjögren syndrome is characterized by a decrease in salivary gland secretion. REF: Urticaria and Angioedema, page 89

OBJ: 13

53. Each statement about lichen planus is true except one. Which statement is false? a. The tongue is most commonly affected. b. It is an idiopathic condition. c. It is a benign condition. d. It is a chronic condition. ANS: A

Lichen planus occurs on the skin and oral mucosa; in the latter case, the buccal mucosa is most frequently affected. Many factors have been implicated in lichen planus; however, the cause remains unknown. Lichen planus is considered benign. Lichen planus is a chronic disease. REF: Lichen Planus, pages 92-93

OBJ: 15

54. Which conditions are not appropriately paired? a. Reactive arthritis—Reiter syndrome b. Aphthous ulcer—canker sore c. Urticaria—hives d. Lichen planus—Sjögren syndrome ANS: D

No association exists between lichen planus and Sjögren syndrome. Reactive arthritis is another name for Reiter syndrome. An aphthous ulcer is also called a canker sore. Urticaria is also referred to as hives. REF: Lichen Planus, pages 92-93 | Sjögren Syndrome, page 97 OBJ: 16 55. Which form of Langerhans cell disease occurs in children younger than 5 years, is multifocal,

and a classic triad is seen in 25% of patients? a. Letterer–Siwe disease b. Hand–Schüller–Christian disease c. Eosinophilic granuloma d. Chronic localized form ANS: B

Hand–Schüller–Christian disease is the multifocal form of Langerhans cell disease. It occurs in children younger than 5 years. A classic triad is seen in 25% of patients. Letterer–Siwe disease resembles lymphoma, affects children younger than 3 years, and has a rapidly fatal course. It is the most severe form of Langerhans cell disease. Eosinophilic granuloma affects older children and young adults. The chronic localized form is the same as eosinophilic granuloma. REF: Langerhans Cell Histiocytosis, page 96

OBJ: 17


56. In the mildest form of this autoimmune disease, oral lesions appear as erythematous plaques

or erosions. White striae radiating from the center of the lesion are also commonly present. These oral lesions are indicative of what autoimmune disease? a. Mucous membrane pemphigoid b. Pemphigus vulgaris c. Lupus erythematosus d. Behçet syndrome ANS: C

In the mildest form of lupus erythematosus, oral lesions appear as erythematous plaques or erosions. White striae radiating from the center of the lesion are also commonly present. Oral lesions in mucous membrane pemphigoid are clinically described as desquamative gingivitis. The oral lesions in pemphigus vulgaris range from shallow ulcers to fragile vesicles or bullae. The oral ulcers that occur in Behçet syndrome are very similar in appearance to aphthous ulcers. REF: Systemic Lupus Erythematosus, page 99

OBJ: 18

57. This patient presents with an intraepithelial bulla from breakdown of the cellular adhesion

between epithelial cells. The bulla rupture, resulting in a painful itching ulcer. The correct diagnosis for this progressive autoimmune disease is a. pemphigus vulgaris. b. Reiter syndrome. c. aphthous ulcers. d. Langerhans cell disease. ANS: A

Acantholysis (breakdown of the cellular adhesion between epithelial cells) is characteristic of pemphigus vulgaris. Reiter syndrome does not present with a bulla that ruptures, resulting in an ulcer. Aphthous ulcers are not a progressive autoimmune disease. Oral manifestations of Langerhans cell disease include loss of bone support, sore mouth, halitosis, gingivitis, and sore teeth. REF: Pemphigus Vulgaris, page 101

OBJ: 18

58. Behçet syndrome is an acute, nonrecurring autoimmune disease consisting mainly of oral

ulcers, genital ulcers, ocular inflammation, and skin lesions. Systemic and topical corticosteroids, as well as immunosuppressive drugs, are used in the management of Behçet syndrome. a. Both statements are true. b. Both statements are false. c. The first statement is true; the second is false. d. The first statement is false; the second is true. ANS: D

Behçet syndrome is chronic and recurring; and systemic and topical corticosteroids, as well as immunosuppressive drugs, are used to treat it. The first statement is false: Behçet syndrome is not acute and it does recur. The second statement is true: systemic and topical corticosteroids, as well as immunosuppressive drugs, are used to treat it. The first statement is false; the second is true.


REF: Behçet Syndrome, page 103

OBJ: 20

59. Decreased salivary flow that results in xerostomia is also known as a. hyposalivation. b. mucositis. c. pruritus. d. anaphylaxis. ANS: A

Hyposalivation is the decreased salivary flow that may result in dry mouth, or xerostomia. Mucositis refers to mucosal inflammation because of a disease process. Pruritus is another term for itching. Anaphylaxis is a severe or immediate type of hypersensitivity in which an exaggerated immunologic reaction occurs. REF: Vocabulary, page 79

OBJ: 1

60. NK cells have the ability to destroy foreign cells. NK cells are active against a variety of cells

except one. Which is the exception? a. Cancer cells b. HIV and AIDS c. Viruses d. NK cells are active against A, B, and C ANS: B

NK cell function is abnormal in HIV-related acquired immunodeficiency syndrome. NK cells are active against cancer cells. NK cells are active against most viruses. NK cells are active against most viruses and cancer cells, but not HIV-related AIDS. REF: Cellular Involvement in the Immune Response, page 83

OBJ: 3

61. Which statement is false about the T-cell lymphocyte? a. The T-cell lymphocyte develops from a bone marrow stem cell. b. The T-cell lymphocyte travels to the liver and is processed into a mature cell. c. The T-cell lymphocyte has a special receptor on its surface. d. The T-cell lymphocyte increases the functioning of the B-cell. ANS: B

The T-cell lymphocyte does not travel to the liver to be processed into a mature cell; the cell travels to the thymus. The T-cell lymphocyte develops from a bone marrow stem cell. The Tcell lymphocyte has a special receptor on its surface. The T-cell lymphocyte increases the functioning of the B-cell. REF: Cellular Involvement in the Immune Response, pages 82-83 OBJ: 3 62. In addition to a clinical diagnosis, which additional category is used in diagnosing aphthous

ulcers? a. Laboratory b. Differential c. Historical d. Therapeutic


ANS: C

The diagnosis of aphthous ulcers is made on the basis of their clinical appearance and a complete patient history. Laboratory results are not specific for any form of aphthous ulcer. The thoroughness and data collection of a differential diagnosis is not necessary to diagnose aphthous ulcers. A therapeutic diagnosis is not necessary to diagnose aphthous ulcers. REF: Oral Immunologic Lesions and Diseases, page 89

OBJ: 12

63. What is the common name for urticaria? a. Hives b. Dermatitis c. Dry socket d. Jaundice ANS: A

Urticaria is also known as hives. Dermatitis (eczema) is the inflammation of the skin. Dry socket (alveolar osteitis) is a postoperative complication of tooth extraction. Jaundice (icterus) is an abnormal accumulation of bilirubin in the blood and manifested by a yellowish discoloration of the skin. REF: Oral Immunologic Lesions and Diseases, page 89

OBJ: 14

64. What is the group of medications used to treat patients with urticaria and angioedema? a. Barbiturates b. Antihistamines c. Antidepressants d. Corticosteroids ANS: B

Antihistamines (e.g., Benadryl) are the standard drugs used to treat urticaria and angioedema. Barbiturates are not useful when treating urticaria and angioedema. Antidepressants are not needed when treating patients with urticaria or angioedema. Corticosteroids are not the drugs of choice when treating urticaria or angioedema. REF: Oral Immunologic Lesions and Diseases, page 90

OBJ: 14

65. Skin lesions, called target or bull’s eye lesions, are characteristic of persons with erythema

multiforme. If the oral cavity is also involved, which conditions are typically seen? a. Gingival inflammation b. Periodontal disease c. Oral ulcers on the tongue and lips d. Xerostomia ANS: C

Oral lesions are usually ulcers that frequently form on the lateral borders of the tongue. Crusting and bleeding lips are also often seen. Gingival involvement with erythema multiforme is rare. Since gingival involvement is rare, periodontal issues are not common with erythema multiforme. Xerostomia is not an oral manifestation of erythema multiforme. REF: Oral Immunologic Lesions and Diseases, page 91

OBJ: 15

66. What is a clinical difference between lichen planus and linea alba?


a. Lichen planus may be removed with a gauze square; linea alba cannot be removed. b. Lichen planus appears more bluish; linea alba appears white. c. Lichen planus may appear on the buccal mucosa as well as the tongue, lips, and

floor of the mouth; linea alba is confined to the buccal mucosa. d. Lichen planus is asymptomatic; linea alba may cause erosions, bullae, or ulcers. ANS: C

Lichen planus can occur in numerous regions of the oral cavity, whereas linea alba is best seen on the buccal mucosa. Lichen planus is not removed with a gauze square. Lichen planus does not carry a blue appearance. Lichen planus may be associated with erosions, bullae, or ulcers. Linea alba is asymptomatic. REF: Oral Immunologic Lesions and Diseases, page 92

OBJ: 16

67. Patients with Sjögren syndrome who experience xerostomia are at a high risk of the following

conditions except one. Which is the exception? a. Melanotic macules b. Dental caries c. Oral candidiasis d. Periodontal disease ANS: A

Patients with Sjögren syndrome are not prone to the development of melanotic macules. Patients with xerostomia are at a high risk for the development of dental caries. Patients with xerostomia are at a high risk for the development of oral candidiasis. Patients with xerostomia are at a high risk for the development of periodontal disease. REF: Autoimmune Disease With Oral Manifestations, page 97

OBJ: 19

68. Some patients with Sjögren syndrome will also demonstrate the Raynaud phenomenon, which

is a disorder that affects the a. joints. b. fingers and toes. c. lacrimal glands. d. eyelids. ANS: B

Raynaud phenomenon is a disorder affecting the fingers and toes, usually triggered by cold and emotional stress. Raynaud phenomenon does not affect the joints of the patient. The lacrimal glands are not affected by Raynaud phenomenon. The eyelids are not affected by Raynaud phenomenon. REF: Autoimmune Disease With Oral Manifestations, page 98

OBJ: 19

69. A person with SLE may have oral manifestations similar to which other condition? a. Lichen planus b. Angioedema c. Minor aphthous ulcers d. Mucous membrane pemphigoid ANS: A


Oral lesions appear as erythematous plaques or erosions with white striae radiating from the center, resembling lichen planus. Angioedema appears as a diffuse swelling of tissue caused by permeability of deeper blood vessels. Minor aphthous ulcers are discrete, punched-out, round to oval ulcers surrounded by a halo of erythema. Mucous membrane pemphigoid demonstrates desquamative gingivitis, which is a range of ulceration and erythema involving the marginal and attached gingiva. REF: Autoimmune Disease With Oral Manifestations, page 99

OBJ: 19

70. Oral lesions accompanying skin lesions may occur in the following diseases except one.

Which is the exception? a. SLE b. Pemphigus vulgaris c. Lichen planus d. Aphthous ulcers ANS: D

Aphthous ulcers are not seen on skin but may develop in the oral cavity. Oral lesions accompany skin lesions in about 25% of patients with SLE. Pemphigus vulgaris is a severe, progressive autoimmune disease that affects both the skin and mucous membranes. Several forms of lichen planus have been described. In addition to oral lesions, skin lesions of itching and pruritus have been noted. REF: Autoimmune Disease With Oral Manifestations, page 87

OBJ: 19

71. What is the most common precipitating factor seen in the development of aphthous ulcers? a. Tobacco use b. Trauma c. Emotional stress d. Infection ANS: B

Trauma is the most commonly reported precipitating factor in the development of aphthous ulcers (e.g., toothbrush trauma, x-ray film or sensor placement, injection site of local anesthetics). Tobacco use is not considered a precipitating factor in the development of aphthous ulcers. Nonsmokers report aphthous ulcer development. Emotional stress has been suggested as a contributing factor of the development of aphthous ulcers. Infection is not typically associated with the development of aphthous ulcers. REF: Oral Immunologic Lesions and Diseases, page 87

OBJ: 12

72. Acantholysis, or the dissolution of the intercellular bridges of the prickle cell layer of the

epithelium, is an oral manifestation of which disease? a. HIV b. Urticaria c. Pemphigus vulgaris d. Behçet syndrome ANS: C


Pemphigus vulgaris is characterized by intraepithelial vesicle formation that results from breakdown of the cellular adhesion between epithelial cells, termed acantholysis. While HIV disease presents with numerous oral manifestations, acantholysis is not typically listed. Urticaria appears as multiple areas of well-demarcated edema and erythema of the skin. Behçet syndrome is an autoimmune disease that presents with oral ulcers that are painful and recurrent. REF: Autoimmune Disease With Oral Manifestations, page 101 OBJ: 19 73. Persons with Behçet syndrome suffer from the following symptoms except one. Which is the

exception? a. Nikolsky sign b. Oral ulcers c. Ocular inflammation d. Genital lesions ANS: A

The Nikolsky sign is associated with finger pressure causing a cleavage in the intraoral epithelium, resulting in the formation of a bulla. This finding is associated with pemphigus vulgaris. Oral ulcers are seen in persons with Behçet syndrome. Ocular inflammation usually begins with photophobia and develops into conjunctivitis in persons with Behçet syndrome. Genital lesions are usually small yet painful and recurrent in persons with Behçet syndrome. REF: Autoimmune Disease With Oral Manifestations, page 103 OBJ: 19


Chapter 04: Infectious Diseases Ibsen: Oral Pathology for the Dental Hygienist, 7th Edition MULTIPLE CHOICE 1. Which condition does not occur in impetigo? a. Itching b. Pruritus c. Regional lymphadenopathy d. Fever ANS: D

Fever and malaise do not occur in impetigo. Itching can occur in impetigo. Pruritus (itching) can occur in impetigo. Regional lymphadenopathy can occur in impetigo. REF: Impetigo, page 117

OBJ: 4

2. Which agent is used to treat impetigo? a. Antiviral medications b. Topical or systemic antibiotics c. Corticosteroids d. Nonsteroidal antiinflammatory agents ANS: B

Impetigo is caused by a bacterial infection; therefore, topical or systemic antibiotics are used to treat it. Antiviral medications are not used to treat impetigo. Corticosteroids are not used to treat impetigo. Nonsteroidal antiinflammatory agents are not used to treat impetigo. REF: Impetigo, page 117

OBJ: 4

3. Tonsillitis and pharyngitis are caused by group A -hemolytic streptococci. These conditions

are significant because of their relationship to scarlet fever and rheumatic fever. Which condition may be related to heart valve damage? a. Rheumatic fever b. Strawberry tongue c. Scarlet fever d. Geographic tongue ANS: A

Rheumatic fever can result in heart valve damage and the development of bacterial endocarditis. Strawberry tongue is associated with scarlet fever, but not specifically with heart valve damage. Heart valve damage is not a specific component of scarlet fever. Geographic tongue is not associated with heart valve damage. REF: Tonsillitis and Pharyngitis, page 117

OBJ: 5

4. Which antigen is injected into the skin to determine exposure and infection with

Mycobacterium tuberculosis? a. HLA-B27 b. PPD


c. VDRL d. IgE ANS: B

Purified protein derivative (PPD) is injected into the skin to determine whether the person has been exposed to and infected with Mycobacterium tuberculosis. HLA-B27 is an antigenic marker present in most patients with Reiter syndrome. VDRL is a serologic blood test used to confirm the diagnosis of syphilis. IgE in the immune response causes mast cells to release their granules containing histamine. REF: Diagnosis, page 118

OBJ: 4

5. Which disease is caused by the spirochete Treponema pallidum? a. Tuberculosis b. Actinomycosis c. Syphilis d. Necrotizing ulcerative gingivitis (NUG) ANS: C

Syphilis is caused by the spirochete Treponema pallidum. Tuberculosis is caused by Mycobacterium tuberculosis. Actinomycosis is an infection caused by a filamentous bacterium called Actinomyces israelii. NUG is caused by both a fusiform bacillus and a spirochete, Borrelia vincentii. REF: Syphilis, page 119

OBJ: 4

6. Involvement and enlargement of the submandibular and cervical lymph nodes in tuberculosis

is called a. miliary tuberculosis. b. lymphadenopathy. c. scrofula. d. angioedema. ANS: C

Scrofula is the name for the involvement and enlargement of the submandibular and cervical lymph nodes in tuberculosis. Miliary tuberculosis is when the bacteria are carried throughout the body, causing involvement of organs such as the kidney and liver. Lymphadenopathy is enlargement of lymph nodes associated with many illnesses and conditions. Angioedema is a diffuse swelling of tissue caused by vascular permeability of the deeper blood vessels. REF: Tuberculosis, page 118

OBJ: 4

7. Which condition is associated with scarlet fever? a. Black hairy tongue b. Median rhomboid glossitis c. Scrofula d. Strawberry tongue ANS: D


Strawberry tongue is associated with scarlet fever. Black hairy tongue occurs when elongation and black discoloration of the filiform papillae is present, often caused by tobacco, alcohol, hydrogen peroxide, or chemical rinses. Median rhomboid glossitis is thought to be associated with Candida organisms. Scrofula is involvement and enlargement of the submandibular and cervical lymph nodes in tuberculosis. REF: Tonsillitis and Pharyngitis, page 117

OBJ: 5

8. The most common site(s) for oral lesions in tuberculosis is(are) the a. tongue and palate. b. gingiva. c. floor of the mouth and mucobuccal folds. d. uvula and fauces. ANS: A

The tongue and palate are the most common sites for oral lesions in tuberculosis. The gingiva is not a common site for oral lesions in tuberculosis. The floor of the mouth and mucobuccal folds are not common sites for oral lesions in tuberculosis. The uvula and fauces are not common sites for oral lesions in tuberculosis. REF: Tuberculosis, page 118

OBJ: 4

9. The most characteristic form of which disease is the formation of abscesses that tend to drain

from the mandible to the skin by the formation of sinus tracts, with sulfur granules in the pus draining from the sinus tracts? a. Actinomycosis b. Tuberculosis c. Syphilis d. Impetigo ANS: A

Actinomycosis is characterized by the formation of abscesses that tend to drain from the mandible to the skin by the formation of sinus tracts, with sulfur granules identified in the pus draining from the sinus tracts. Tuberculosis is a primary infection of the lung. The signs and symptoms of this lung infection include fever, chills, fatigue, weight loss, and persistent cough. Syphilis is not characterized by abscesses and sulfur granules. Impetigo is a bacterial skin infection. The lesions are either vesicles or bullae. REF: Actinomycosis, page 118

OBJ: 4

10. In primary syphilis, the oral lesion is termed a. mucous patch. b. chancre. c. gumma. d. ulcer. ANS: B

A chancre is the lesion in stage I (primary) syphilis. A mucous patch is the lesion in stage II (secondary) syphilis. A gumma is the lesion in stage III (tertiary) syphilis. An ulcer is a general descriptive term and is not the specific term used to describe the lesions of syphilis.


REF: Syphilis, page 119

OBJ: 4

11. Which statement is not true about syphilis? a. Dark-field examination may be used to diagnose syphilitic lesions on the skin. b. Syphilis is generally treated with penicillin. c. The VDRL and fluorescent treponemal antibody absorption (FTA-ABS) tests are

commonly used to confirm the diagnosis of syphilis. d. The antibody titer increases if treatment has been successful. ANS: D

The antibody titer decreases if the treatment has been successful. Dark-field examination is used to diagnose skin lesions. Syphilis is generally treated with penicillin. The VDRL and FTA-ABS tests are two serologic blood tests used to confirm the diagnosis of syphilis. REF: Diagnosis and Treatment, page 120

OBJ: 4

12. Pericoronitis is most often associated with a. maxillary canines. b. mandibular third molars. c. maxillary second molars. d. mandibular first molars. ANS: B

Pericoronitis most often involves the mucosal tissue associated with partially erupted mandibular third molars. Pericoronitis is not seen on maxillary canines. Pericoronitis is not seen on maxillary second molars. Pericoronitis is not seen on mandibular first molars. REF: Pericoronitis, page 120

OBJ: 4

13. Which is the name of the soft tissue flap that covers the distal-occlusal part of an incompletely

erupted third molar? a. Operculum b. Gingival flap c. Pericoronitis d. Partial eruption flap ANS: A

The operculum is the soft tissue flap that covers the distal-occlusal part of an incompletely erupted third molar. The gingival flap is not an operculum. Pericoronitis is the inflammation of the mucosa around the crown of a partially erupted or impacted tooth. The term partial eruption flap does not exist. REF: Pericoronitis, page 120

OBJ: 4

14. Which statement is true concerning a positive reaction to the PPD skin test? a. The patient has active tuberculosis. b. The patient is contagious. c. The patient has been infected previously with Mycobacterium tuberculosis. d. The patient is in need of hospitalization. ANS: C


A positive skin reaction to PPD means that the patient was previously infected with the bacteria. A positive skin reaction to PPD does not mean that the patient has active tuberculosis. A positive skin reaction to PPD does not mean that the patient is contagious. A positive skin reaction to PPD does not require any hospitalization. REF: Diagnosis, page 118

OBJ: 4

15. The gingivae shown are painful and erythematous. The interdental papillae appear as

punched-out, necrotic, cratering areas. The overall sloughing of the necrotic tissue appears as a pseudomembrane over the tissues. The patient experiences a foul odor and metallic taste. On the basis of these features, which condition is suspected? a. Acute marginal gingivitis b. Primary herpes simplex infection c. NUG d. Pseudomembranous candidiasis ANS: C

In NUG the gingiva are painful and erythematous, the interdental papillae are punched out and necrotic, and the patient experiences a foul odor and metallic taste. In acute marginal gingivitis there is no cratering and necrosis of the papillae. In primary herpes simplex infection there is no cratering and necrosis of the papillae. In pseudomembranous candidiasis a white curdlike material is present on the mucosal surfaces. The underlying mucosa appears erythematous. REF: Necrotizing Ulcerative Gingivitis, page 120

OBJ: 4

16. Acute osteomyelitis of the jaws may commonly result from which condition? a. Extension of a periapical abscess b. Surgery c. Fracture of the jaw d. Paget disease ANS: A

Acute osteomyelitis of the jaws commonly results from extension of a periapical abscess. Acute osteomyelitis can result from surgery, but this is not a common reaction. Acute osteomyelitis can result from fracture of the jaw, but again this is not a common reaction. Paget disease has been associated with chronic osteomyelitis. REF: Acute Osteomyelitis, page 121

OBJ: 4

17. Candidiasis is the result of an overgrowth of a a. yeastlike fungus. b. spirochete. c. filamentous bacterium. d. fusiform bacillus. ANS: A

Candidiasis is the result of an overgrowth of a yeastlike fungus, Candida albicans. A spirochete, Borrelia vincentii, causes NUG. A filamentous bacterium called Actinomyces israelii causes actinomycosis. Fusiform bacilli are involved in the etiology of NUG.


REF: Candidiasis, page 121

OBJ: 6

18. Which condition does not contribute to the overgrowth of Candida albicans? a. Antibiotic therapy b. Dentures c. Cancer chemotherapy d. Angioedema ANS: D

Angioedema is a type I hypersensitivity reaction and has nothing to do with Candida albicans. Antibiotic therapy can cause an overgrowth of Candida albicans. Dentures can contribute to an overgrowth of Candida albicans. Cancer chemotherapy can contribute to an overgrowth of Candida albicans. REF: Candidiasis, page 121

OBJ: 6

19. The most common type of candidiasis affecting the oral mucosa is __________________. a. pseudomembranous b. erythematous c. chronic atrophic d. chronic hyperplastic ANS: C

Chronic atrophic candidiasis, or denture stomatitis, is the most common type of candidiasis affecting the oral mucosa. Pseudomembranous candidiasis is a white curdlike material present on the mucosal tissues. The underlying mucosa is erythematous in appearance. It is not the most common type of oral mucosal candidiasis. Erythematous candidiasis is often painful and may be localized or generalized. It is not the most common type of oral mucosal candidiasis. Chronic hyperplastic candidiasis appears as a white lesion that does not wipe off. If it does not respond to antifungal therapy, a biopsy should be considered. It is not the most common type of candidiasis. REF: Denture Stomatitis, page 122

OBJ: 6

20. This lesion in the commissure is most likely caused by Candida albicans or a nutritional

deficiency and is termed a. pseudomembranous candidiasis. b. angular cheilitis. c. chronic mucocutaneous candidiasis. d. chronic atrophic candidiasis. ANS: B

Angular cheilitis is a lesion seen in the commissure of the lips and is most likely caused by Candida albicans or a nutritional deficiency. Pseudomembranous candidiasis appears as a white curdlike material on the mucosal surface. The underlying mucosa is erythematous. Chronic mucocutaneous candidiasis is a severe form of candidiasis that occurs in patients who are severely immunocompromised. Chronic atrophic candidiasis, or denture stomatitis, is the most common type of oral candidiasis affecting the oral mucosa. REF: Angular Cheilitis, page 123

OBJ: 6


21. Deep fungal infections include all except a. histoplasmosis. b. coccidioidomycosis. c. blastomycosis. d. actinomycosis. ANS: D

Actinomycosis is not a deep fungal infection. It is caused by a filamentous bacterium. Histoplasmosis is a deep fungal infection widespread in the Midwestern United States. Coccidioidomycosis is a deep fungal infection more prevalent in the Western United States. Blastomycosis is a deep fungal infection most common in the Ohio–Mississippi River basin area. REF: Deep Fungal Infections, page 124

OBJ: 6

22. The initial infection and signs and symptoms of deep fungal infections, including

histoplasmosis, coccidioidomycosis, and blastomycosis, are related to diseases in which location? a. Lung b. Kidney c. Lymph nodes d. Nasal cavity ANS: A

Deep fungal infections are most commonly primary infections of the lung. The kidney is not the primary site for deep fungal infection. The lymph nodes are not the primary site for deep fungal infection. The nasal cavity may be a primary site for a rare fungal infection called mucormycosis. REF: Deep Fungal Infections, page 124

OBJ: 6

23. Another name for verruca vulgaris is a. condyloma acuminatum. b. the common wart. c. Heck disease. d. focal epithelial hyperplasia. ANS: B

Another name for verruca vulgaris is the common wart. Condyloma acuminatum is a benign papillary lesion sexually transmitted and caused by a human papillomavirus (HPV), which is different from the papillomavirus that causes verruca vulgaris. Heck disease is caused by an HPV that is different from the types of HPV that cause verruca vulgaris and condyloma acuminatum. The disease presents with multiple mucosal nodules and resolves spontaneously. Focal epithelial hyperplasia is another name for Heck disease. REF: Verruca Vulgaris, page 125

OBJ: 6

24. The image shows an oral condyloma acuminatum presenting in a child. The presence of

condyloma acuminatum in a child suggests a. hypersensitivity. b. genetic predisposition.


c. sexual abuse. d. autoimmune disease. ANS: C

The HPV that causes condyloma acuminatum is almost always transmitted by sexual contact. The presence of condyloma acuminatum in a child suggests sexual abuse. Hypersensitivity is not the cause of condyloma acuminatum. Genetic predisposition has nothing to do with the cause of condyloma acuminatum. Condyloma acuminatum is not an autoimmune disease. REF: Condyloma Acuminatum, Fig. 4.18, page 126

OBJ: 4

25. The most common form of recurrent herpes simplex infection is termed a. herpes zoster. b. herpetic whitlow. c. herpes labialis. d. type 2 herpes. ANS: C

Herpes labialis is the most common form of recurrent herpes simplex virus infection. Herpes zoster is caused by the varicella-zoster virus and is characterized by unilateral vesicles along a sensory nerve. Herpetic whitlow occurs when herpes simplex infection involves the fingers. Type 2 herpes simplex is most commonly associated with genital infections. Genital herpes simplex infections may be primary or recurrent. REF: Recurrent Herpes Simplex Infection, page 127

OBJ: 8

26. The varicella-zoster virus causes a highly contagious disease in children termed a. shingles. b. chickenpox. c. mumps. d. measles. ANS: B

Chickenpox is a highly contagious disease in children caused by the varicella-zoster virus. Shingles is a recurrent form of the varicella-zoster virus. It is most commonly seen in older adults and in immunocompromised individuals. Mumps is caused by a paramyxovirus and is a viral infection of the salivary glands. Measles is a highly contagious disease caused by a paramyxovirus and is seen most commonly in children. REF: Chickenpox, page 130

OBJ: 9

27. The intraoral lesion seen in patients with measles is termed a. minor salivary gland swelling. b. generalized painful gingivitis. c. Koplik spots. d. candidiasis. ANS: C


Koplik spots are erythematous macules with white necrotic centers that may occur in the oral cavity in patients with measles. Minor salivary gland swelling is not associated with measles. Generalized painful gingivitis occurs in primary herpes simplex infection. Candidiasis is one of the most common oral manifestations associated with patients who have human immunodeficiency virus (HIV). REF: Other Viral Infections That May Have Oral Manifestations, page 132 OBJ: 12 28. Which virus is associated with infectious mononucleosis? a. Herpes simplex virus b. HPV c. Epstein-Barr virus d. Varicella-zoster virus ANS: C

Infectious mononucleosis is caused by the Epstein-Barr virus. Herpes simplex virus causes primary herpetic gingivostomatitis and recurrent herpes simplex virus infections. HPV is responsible for several oral viral conditions. Varicella-zoster virus causes chickenpox and shingles. REF: Epstein-Barr Virus Infection, page 131

OBJ: 11

29. Herpangina is caused by which virus? a. Herpes simplex virus b. Epstein-Barr virus c. Varicella-zoster virus d. Coxsackievirus ANS: D

Herpangina is caused by the coxsackievirus. Herpes simplex virus causes primary herpetic gingivostomatitis and recurrent herpes simplex virus infections. Epstein-Barr virus causes mononucleosis. Varicella-zoster virus causes chickenpox and shingles. REF: Herpangina, page 132

OBJ: 12

30. In testing for HIV infection, which antibody test is performed initially before the confirmatory

Western blot test? a. VDRL b. ELISA (EIA) c. FTA-ABS d. CBC ANS: B

The ELISA (enzyme-linked immunosorbent assay [also called EIA]) is the routine initial test used to identify HIV infection. When positive, this test is repeated, and positive results are confirmed with another laboratory test called the Western blot. VDRL is a blood test performed to diagnose syphilis. FTA-ABS is the fluorescent treponemal antibody absorption test performed to diagnose syphilis. CBC is a complete blood count and not specific for the diagnosis of HIV. REF: Human Immunodeficiency Virus Testing, page 133

OBJ: 13


31. In HIV infection, which test identifies the viral titer rather than circulating antibody? a. VDRL b. ELISA c. Western blot d. PCR ANS: D

In evaluating patients with HIV infection, the PCR (polymerase chain reaction) identifies virus circulating in the blood and not circulating antibody. VDRL tests for syphilis. The ELISA tests for antibodies to HIV. The Western blot test follows two positive ELISA tests for antibodies to HIV. REF: Human Immunodeficiency Virus Testing, page 133

OBJ: 13

32. The most reliable method of diagnosis for this condition of the tongue is the isolation of the

Epstein-Barr virus. This condition is a. white hairy tongue. b. hairy leukoplakia. c. mononucleosis. d. Kaposi sarcoma. ANS: B

Hairy leukoplakia is caused by the Epstein-Barr virus. White hairy tongue is a condition in which there is elongation of the filiform papillae; it is commonly caused by tobacco, hydrogen peroxide, chemical rinses, alcohol, or certain foods. Mononucleosis is caused by the EpsteinBarr virus, but is not a condition of the tongue. Kaposi sarcoma is a vascular neoplasm that occurs in some patients with HIV infection. REF: Hairy Leukoplakia, page 136

OBJ: 11

33. Which does not occur as a manifestation of the immune deficiency caused by HIV infection? a. Candidiasis b. Hairy leukoplakia c. Kaposi sarcoma d. Bilateral parotid atrophy ANS: D

Bilateral parotid enlargement, not atrophy, has been reported in patients who are HIV positive. Candidiasis is seen commonly in patients who are HIV positive. Hairy leukoplakia is often seen in patients who are HIV positive. Kaposi sarcoma is a neoplasm seen in HIV-positive patients. REF: Salivary Gland Disease, page 138

OBJ: 14

34. The patient is HIV seropositive. The most common intraoral locations for this lesion are the

gingiva and palate. When diagnosed, this vascular lesion meets the criteria for the diagnosis of acquired immune deficiency syndrome (AIDS). What is the name of the vascular lesion seen in this image? a. Non-Hodgkin lymphoma b. Kaposi sarcoma


c. Thrombocytopenia d. Hemangioma ANS: B

Kaposi sarcoma is a vascular neoplasm seen on the palate or gingiva of an HIV-infected patient. Oral Kaposi sarcoma is an indication of severe immune deficiency. Non-Hodgkin lymphoma is a malignant tumor that occurs in association with HIV infection. Thrombocytopenia is a bleeding disorder associated with many systemic conditions. Hemangioma is a benign developmental proliferation of capillaries. REF: Kaposi Sarcoma, page 136

OBJ: 14

35. The lesions seen in this illustration are not commonly found in HIV-infected patients. Which

is the likely diagnosis? a. Candidiasis b. Hairy leukoplakia c. Herpes simplex d. Heck disease ANS: D

Heck disease (focal epithelial hyperplasia) is caused by a HPV and is most commonly seen in children. Rare cases have also been described in HIV-seropositive patients. Candidiasis is commonly seen in patients who are HIV positive. Hairy leukoplakia is a common oral manifestation of HIV infection. Herpes simplex is a common component of HIV infection. REF: Multifocal Epithelial Hyperplasia, pages 136-137

OBJ: 14

36. Verruca vulgaris closely resembles a papillary benign tumor of squamous epithelium termed a. fibroma. b. papilloma. c. lipoma. d. Heck disease. ANS: B

Verruca vulgaris resembles the papilloma, which is a focal papillary benign tumor of squamous epithelium. A fibroma is a reactive lesion that is composed primarily of dense fibrous connective tissue surfaced by epithelium. A lipoma is a benign tumor of fat cells. Heck disease is caused by a HPV and is characterized by the presence of multiple white-to-pink nodules distributed throughout the mucosa. REF: Verruca Vulgaris, page 125

OBJ: 4

37. In herpes labialis, the amount of virus present is highest in the __________ stage. a. crusted b. prodromal c. vesicle d. healing ANS: C


In herpes labialis, the amount of virus present is greatest in the vesicle stage. The crusted stage occurs after the vesicles break. Few viral particles remain at this stage. The prodromal stage occurs a few days before eruption of the vesicles. Viral particles are released at the epithelium but are fewer in number than at the vesicle stage. The healing stage occurs after the vesicles have broken. Less virus is present at this stage compared with the vesicle stage. REF: Recurrent Herpes Simplex Infection, page 128

OBJ: 7

38. Which type of medication is used for the treatment of herpes zoster? a. Antibiotics b. Antiviral medications c. Supportive therapy only, no drug therapy d. Nonsteroidal antiinflammatory drugs ANS: B

Antiviral medications are used to treat herpes zoster. Antibiotics should be used only to treat secondary infection. Supportive therapy is not sufficient to treat herpes zoster. Nonsteroidal antiinflammatory drugs generally are not used to treat herpes zoster. Corticosteroids have been used to attempt to prevent the pain of postherpetic neuralgia. REF: Herpes Zoster, page 130

OBJ: 9

39. The stage between the infection of an individual by a pathogen and the manifestation of the

disease it causes is a. opportunistic infection. b. incubation period. c. subclinical infection. d. paresthesia. ANS: B

The incubation period is the stage between the infection of an individual by a pathogen and the manifestation of the disease it causes. Opportunistic infection is disease caused by a microorganism that does not ordinarily cause disease but becomes pathogenic under certain circumstances. Subclinical infection is disease not detectable by the usual clinical signs. Paresthesia is an abnormal sensation such as tingling or prickling. REF: Vocabulary, page 116

OBJ: 1

40. Opportunistic infections can occur because the oral microflora is affected by changes such as

a decrease in salivary flow, immune system alterations, or antibiotic administration. a. Both the statement and reason are correct and related. b. Both the statement and reason are correct but not related. c. The statement is correct, but the reason is not. d. The statement is not correct, but the reason is correct. e. Neither the statement nor the reason is correct. ANS: A

Opportunistic infections can occur because the oral microflora is affected by changes such as a decrease in salivary flow, immune system alterations, or antibiotic administration. Both the statement and reason are correct and related.


REF: Introduction, page 117

OBJ: 2

41. Tonsillitis and pharyngitis have a significant relationship with scarlet fever and rheumatic

fever when caused by certain bacteria. What is the name of these bacteria? a. Group A -hemolytic streptococci b. Mycobacterium tuberculosis c. Actinomyces israelii d. Treponema pallidum ANS: A

Tonsillitis and pharyngitis caused by group A -hemolytic streptococci are significant because of their relationship to scarlet fever and rheumatic fever. Mycobacterium tuberculosis causes tuberculosis. Actinomyces israelii causes actinomycosis. Treponema pallidum causes syphilis. REF: Tonsillitis and Pharyngitis, page 117

OBJ: 5

42. The type of candidiasis that appears as a white lesion that does not wipe off and does not

respond to antifungal therapy is a. pseudomembranous. b. erythematous. c. chronic atrophic. d. chronic hyperplastic. ANS: D

Chronic hyperplastic candidiasis appears as a white lesion that does not wipe off. If it does not respond to antifungal therapy, a biopsy should be considered. Pseudomembranous candidiasis is a white curdlike material present on the mucosal tissues. The underlying mucosa is erythematous in appearance. Erythematous candidiasis is an erythematous, often painful mucosa that may be localized or generalized. Chronic atrophic candidiasis or denture stomatitis presents as erythematous mucosa with lesions that vary from petechiae-like to more generalized and granular. REF: Candidiasis, page 122

OBJ: 6

43. Histoplasmosis, coccidioidomycosis, blastomycosis, and cryptococcosis are examples of what

type of infection? a. Viral b. Fungal c. Developmental d. Bacterial ANS: B

Histoplasmosis, coccidioidomycosis, blastomycosis, and cryptococcosis are all examples of deep fungal infections. Viral infections do not include histoplasmosis, coccidioidomycosis, blastomycosis, and cryptococcosis. Infections are not referred to as developmental. Bacterial infections do not include histoplasmosis, coccidioidomycosis, blastomycosis, and cryptococcosis. REF: Deep Fungal Infections, page 124

OBJ: 6

44. Each benign lesion is caused by the HPV except one. Which one is the exception?


a. b. c. d.

Focal epithelial hyperplasia Condyloma acuminatum NUG Verruca vulgaris

ANS: C

NUG is not caused by the HPV. Focal epithelial hyperplasia is a benign lesion caused by the HPV. Condyloma acuminatum is a benign lesion caused by the HPV. Verruca vulgaris is a benign lesion caused by the HPV. REF: Human Papillomavirus Infection, pages 125-126

OBJ: 4

45. Focal epithelial hyperplasia, characterized by the presence of multiple whitish-to-pale pink

nodules distributed throughout the oral mucosa, is termed a. osteomyelitis. b. lymphadenopathy. c. Heck disease. d. angioedema. ANS: C

Another name for focal epithelial hyperplasia is Heck disease. Osteomyelitis is inflammation of the bone and bone marrow spaces and is not another name for focal epithelial hyperplasia. Lymphadenopathy is enlargement of lymph nodes associated with many illnesses and conditions and is not another name for focal epithelial hyperplasia. Angioedema is a diffuse swelling of tissue caused by vascular permeability of deeper blood vessels and is not another name for focal epithelial hyperplasia. REF: Multifocal Epithelial Hyperplasia, page 126

OBJ: 4

46. The vesicle cluster visible on the corner of this patient’s vermilion border is also referred to as

a cold sore or fever blister. The clinical diagnosis is a. herpes labialis. b. primary herpes. c. aphthous ulcer. d. herpetic whitlow. ANS: A

The clinical diagnosis for the patient in these photographs is herpes labialis. Primary herpes is not referred to as a cold sore or fever blister, and vesicle clusters are more prominent. An aphthous ulcer does not appear as a vesicle and is not referred to as a cold sore or fever blister. Herpetic whitlow occurs on the finger and is not referred to as a cold sore or fever blister. REF: Recurrent Herpes Simplex Infection, page 127 47. A vaccine is available to prevent which infection? a. Herpes labialis b. Epstein-Barr c. Varicella-zoster d. Coxsackievirus ANS: C

OBJ: 7


A vaccine is available to prevent varicella-zoster infections. The vaccine is given to children to prevent chickenpox and to older adults to prevent the recurrence of the infection as herpes zoster. Herpes labialis, the most common type of recurrent oral herpes simplex infection, does not have a vaccine. The Epstein-Barr virus, found in infectious mononucleosis, does not have a vaccine. The coxsackievirus infection does not have a vaccine. REF: Varicella-Zoster Virus, page 130

OBJ: 12

48. The unilateral eruption of vesicles along the distribution of a sensory nerve seen in this adult

patient is characteristic of a. recurrent intraoral herpes simplex. b. minor aphthous ulcers. c. varicella-zoster or chickenpox. d. herpes zoster or shingles. ANS: D

Herpes zoster or shingles appears as a unilateral eruption of vesicles along the distribution of a sensory nerve. Recurrent intraoral herpes simplex occurs in the oral cavity. Minor aphthous ulcers appear in the oral cavity and not along the distribution of a sensory nerve. Varicellazoster or chickenpox appears as vesicular and pustular eruptions of the skin and mucous membranes. REF: Herpes Zoster, page 130

OBJ: 9

49. The type of cell of the immune system that HIV infects that is important in cell-mediated

immunity and in regulating the immune response is which cell? a. VDRL b. CD4+ T-helper lymphocytes c. Group A -hemolytic streptococci d. Tzanck cells ANS: B

CD4+ T-helper lymphocytes are important immune system cells infected by HIV. VDRL is a test for syphilis. Group A -hemolytic streptococci cause tonsillitis and pharyngitis. Tzanck cells are altered epithelial cells that are seen on a cytologic smear in herpes simplex infections. REF: The Spectrum of Human Immunodeficiency Virus, page 133 OBJ: 13 50. The lesions seen in this illustration most commonly appear in patients with HIV. What is the

clinical diagnosis? a. Candidiasis b. Lymphoma c. Kaposi sarcoma d. Aphthous ulcer ANS: A

Candidiasis is characterized by white, plaquelike lesions and is commonly seen in patients who are HIV positive. Lymphoma occasionally occurs as an oral manifestation of HIV infection. Kaposi sarcoma appears as a reddish purple, flat, or raised lesion in the oral cavity. Aphthous ulcers appear as deep, persistent, and painful ulcers.


REF: Oral Candidiasis, Fig. 4.33, pages 134-135

OBJ: 14

51. Why are newborn babies susceptible to an infection with Candida albicans? a. Newborns are always susceptible to numerous infections. b. The organism can cross the placenta. c. Babies do not have a fully developed immune system. d. The infection can be passed during breast-feeding. ANS: C

Newborns are particularly susceptible to this fungal infection because an established oral microflora or a fully developed immune system has not been recognized at this age. Newborns are not susceptible to many infections. The organism does not cross the placenta. This fungal infection is not passed through breast-feeding. REF: Fungal Infections, page 121

OBJ: 6

52. Which form of candidiasis is most asymptomatic? a. Pseudomembranous b. Erythematous c. Chronic atrophic candidiasis d. Angular cheilitis ANS: C

Chronic atrophic candidiasis is also known as denture stomatitis. It is asymptomatic and usually discovered during a routine oral examination. Pseudomembranous candidiasis may cause a burning sensation, accompanied by a metallic taste. Erythematous candidiasis is often a painful condition. Angular cheilitis appears reddened with fissuring in the corners of the mouth and may be painful upon opening. REF: Fungal Infections, page 122

OBJ: 6

53. Conditions associated with overgrowth of Candida albicans include the following except one.

Which is the exception? a. Denture wearer b. Xerostomia c. Diabetic patient d. Patients with NUG ANS: D

Patients with NUG are typically not at risk for developing an infection with Candida albicans. Denture wearers are at risk of developing an infection with Candida albicans. Xerostomia poses a risk for the development of an infection with Candida albicans. Persons with diabetes are at risk for developing an infection with Candida albicans. REF: Fungal Infections, page 121

OBJ: 6

54. Which tooth is most often associated with pericoronitis? a. #3 b. #15 c. #17 d. #30


ANS: C

The mandibular third molar (#17 or #32) is the most common location for pericoronitis. REF: Bacterial Infections, page 120

OBJ: 4

55. What are the most common intraoral sites for the development of a gumma? a. Tongue and buccal mucosa b. Tongue and palate c. Palate and floor of the mouth d. Floor of the mouth and labial mucosa ANS: B

A gumma can occur in the oral cavity, and the most common sites are the tongue and palate. REF: Bacterial Infections, page 119

OBJ: 4

56. A PPD test has been administered to check if your patient has been exposed and infected with

the Mycobacterium tuberculosis infection. The results of the skin test were positive. What is the next step? a. Begin treatment immediately with isoniazid b. Expose a chest radiograph c. Send the patient to the physician for blood and urine samples d. Confine the patient to bed for 7–10 days ANS: B

Once a positive PPD test is read, chest radiographs are exposed to determine whether or not active tuberculosis is present. Combinations of various medications, including isoniazid, are used to treat tuberculosis once it is confirmed that active disease is present. Blood and urine samples are not necessary procedures for a positive PPD test. Bed rest for 7–10 days is not a necessary procedure for a positive PPD test. REF: Bacterial Infections, page 118

OBJ: 4

57. What is an oral manifestation of a patient with scarlet fever? a. Strawberry tongue b. Formation of draining abscesses c. White, curdlike material present on the mucosa d. Intraoral ulcers ANS: A

Oral manifestations of scarlet fever include a strawberry tongue: the fungiform papillae are red and prominent, and the dorsal surface exhibits a white or red appearance. Formation of draining abscesses is seen in persons with actinomycosis. White, curdlike material on the mucosa describes pseudomembranous candidiasis. Intraoral ulcers may be seen with the herpes simplex virus. REF: Bacterial Infections, page 117

OBJ: 4

58. An erythematous, often rhombus-shaped, flat-to-raised area on the midline of the posterior

dorsal tongue describes a. median rhomboid glossitis.


b. geographic tongue. c. recurrent herpes simplex infection. d. hairy leukoplakia. ANS: A

Median rhomboid glossitis appears as a rhombus-shaped, flat-to-raised area on the midline of the posterior dorsal tongue. It is sometimes associated with an infection from Candida albicans. Geographic tongue displays diffuse areas devoid of filiform papillae with erythematous patches surrounded by a yellow or white perimeter. Recurrent herpes simplex infection does not involve the tongue. Hairy leukoplakia is an irregular white patch seen on the lateral borders of the tongue. REF: Fungal Infections, page 123

OBJ: 6

59. Which virus causes herpes zoster? a. Herpes simplex virus b. Epstein-Barr virus c. Coxsackie virus d. Varicella-zoster virus ANS: D

The varicella-zoster virus causes chickenpox and herpes zoster (shingles). The herpes simplex virus does not cause herpes zoster; it causes primary herpetic gingivostomatitis. The EpsteinBarr virus causes mononucleosis. The coxsackievirus causes herpangina and hand-foot-andmouth disease. REF: Viral Infections, page 129

OBJ: 9

60. What is a common route of infection for infectious mononucleosis? a. Secretions from skin lesions b. Saliva contact during kissing c. Epidemics in children younger than age 5 d. Oral-genital contact ANS: B

Infectious mononucleosis is transmitted by close contact, and contact with saliva during kissing is a frequent route of transmission. Secretions from skin lesions is an infection route for the varicella-zoster virus. Hand-foot-and-mouth disease is commonly transmitted in children younger than age 5, often in preschools or toddler care zones. Oral-genital contact may be a route of infection of the human papillomaviruses. REF: Viral Infections, page 131

OBJ: 11

61. Why is hairy leukoplakia a significant clinical finding in persons with HIV disease? a. It is considered a predictor of the development of AIDS b. Its presence suggests poor oral hygiene c. Hairy leukoplakia may affect the function of the taste buds d. It may represent a malignancy of the tongue ANS: A


Studies have shown hairy leukoplakia to be predictive of the development of AIDS in HIVinfected persons. Hairy leukoplakia is not associated with poor oral hygiene. Hairy leukoplakia does not affect taste bud function. Microscopically, hairy leukoplakia reveals hyperkeratosis and epithelial hyperplasia; no sign of malignant cells are present. REF: Human Immunodeficiency Virus and Acquired Immunodeficiency Syndrome, page 136 OBJ: 14 62. Intraoral lesions of Kaposi sarcoma are often found in which two sites? a. Labial mucosa and interdental papilla b. Gingiva and floor of the mouth c. Palate and gingiva d. Uvula and posterior 1/3 of the tongue ANS: C

The most common locations for the appearance of Kaposi sarcoma in a person with HIV infection are the palate and gingiva. REF: Human Immunodeficiency Virus and Acquired Immunodeficiency Syndrome, page 136 OBJ: 14 63. The common characteristic features of linear gingival erythema seen in a person with HIV

disease include the following except one. Which is the exception? a. Punched-out papilla b. A band-like erythema on the gingiva that does not respond to therapy c. Spontaneous bleeding d. Punctate lesions on the attached gingiva and alveolar mucosa ANS: A

Punched-out papilla is not a characteristic feature of linear gingival erythema. Since linear gingival erythema occurs independently of oral hygiene status, it does not respond to therapy and maintains its bandlike erythema. Spontaneous bleeding is a common sign of linear gingival erythema. Punctate or petechiae-like lesions on the attached gingiva and alveolar mucosa are commonly seen with linear gingival erythema. REF: Human Immunodeficiency Virus and Acquired Immunodeficiency Syndrome, page 136 OBJ: 14 64. HIV infects cells of the immune system. What is the most important cell that HIV infects? a. Neutrophil b. Macrophage c. Polymorphonuclear leukocytes d. CD4 T-helper lymphocyte ANS: D

The CD4 T-helper lymphocyte is the most important of the cells of the immune system that HIV infects. It is important in regulating the immune response. The neutrophil is the first white blood cell to arrive at a site of injury. The macrophage is the second white blood cell to arrive at a site of injury. Another name for the neutrophil is the polymorphonuclear leukocyte because this cell possesses a multilobed nucleus. REF: Human Immunodeficiency Virus and Acquired Immunodeficiency Syndrome, page 133


OBJ: 13 MATCHING

For each clinical appearance listed below, select the most closely linked disorder from the list provided. a. Impetigo b. Tuberculosis c. Actinomycosis d. Syphilis e. Primary herpetic gingivostomatitis f. Verrucous vulgaris g. Condyloma acuminatum 1. 2. 3. 4. 5. 6. 7.

Pink, papillary lesion(s), more diffuse than papilloma Draining abscesses, “sulfur granules” Vesicles or bullae, more often on skin of face and extremities Primary (chancre), secondary (mucous patch), tertiary (gumma) Multiple tiny vesicles that progress to form painful ulcers Granulomatous lesions with primary infection to the lungs White, papillary exophytic lesion resembling a papilloma

1. 2. 3. 4. 5. 6. 7.

ANS: G ANS: C ANS: A ANS: D ANS: E ANS: B ANS: F

REF: REF: REF: REF: REF: REF: REF:

Viral Infections, page 126 Bacterial Infections, page 118 Bacterial Infections, page 117 Bacterial Infections, page 119 Viral Infections, page 127 Bacterial Infections, page 118 Viral Infections, page 125

OBJ: OBJ: OBJ: OBJ: OBJ: OBJ: OBJ:

4 4 4 4 4 4 4

For each disorder listed below, select the most closely linked clinical appearance from the list provided. a. Herpangina b. Measles c. Acute lymphonodular pharyngitis d. Hairy leukoplakia e. Hand-foot-and-mouth disease f. Mumps 8. Irregular corrugated white lesion most commonly occurring on the lateral border of the tongue 9. Vesicles on the soft palate along with fever, malaise, sore throat, dysphagia, and erythematous 10. 11. 12. 13.

pharyngitis Painful vesicles and ulcers that can occur anywhere in the mouth; present in epidemic form in children younger than 5 years Hyperplastic lymphoid tissue of the soft palate or tonsillar pillars appears as yellowish or dark pink nodules Koplik spots in the oral cavity and skin rash that results from a paramyxovirus Epidemic parotitis characterized by bilateral swelling of the parotid glands


8. ANS: D OBJ: 4 9. ANS: A OBJ: 4 10. ANS: E OBJ: 4 11. ANS: C OBJ: 4 12. ANS: B OBJ: 4 13. ANS: F OBJ: 4

REF: Epstein-Barr Virus Infection, page 131 REF: Coxsackievirus Infections, page 132 REF: Coxsackievirus Infections, page 132 REF: Coxsackievirus Infections, page 132 REF: Other Viral Infections That May Have Oral Manifestations, page 132 REF: Other Viral Infections That May Have Oral Manifestations, page 132


Chapter 05: Developmental Disorders Ibsen: Oral Pathology for the Dental Hygienist, 7th Edition MULTIPLE CHOICE 1. Which term describes a disorder present at and existing from the time of birth? a. Anomaly b. Inherited c. Congenital d. Developmental ANS: C

A congenital disorder is present at and existing from the time of birth. An anomaly is a marked deviation from normal that can be the result of congenital or hereditary defects. Inherited disorders are caused by abnormalities in the genetic makeup transmitted from parent to offspring. Developmental disorders occur when failure or disturbances occur during the complex series of cell division, multiplication, or differentiation. REF: Vocabulary, page 148

OBJ: 1

2. Which term describes partial anodontia or the lack of one or more teeth? a. Anodontia b. Ankylosed c. Hypodontia d. Gemination ANS: C

Hypodontia defines partial anodontia or the lack of one or more teeth. Anodontia is the congenital lack of teeth. Ankylosed teeth are those fused to alveolar bone, usually retained deciduous teeth. Gemination occurs when a single tooth germ attempts to divide, resulting in the incomplete formation of two teeth. REF: Vocabulary, page 148

OBJ: 13

3. Which epithelium-lined tract is a developmental anomaly located in the corners of the mouth? a. Commissural lip pit b. Angular cheilitis c. Fistula d. Congenital lip pit ANS: A

Commissural lip pits are epithelium-lined blind tracts located in the corners of the mouth. Angular cheilitis is often caused by Candida organisms. It appears as erythema or fissuring at the labial commissures. A fistula is a drainage tract from an area of infection. A congenital lip pit occurs near the midline of the vermilion border of the lip, and it appears as a depression. REF: Commisural Lip Pits, page 151 4. The formation of dentin is termed a. amelogenesis.

OBJ: 5


b. dentinogenesis. c. dens in dente. d. odontogenesis. ANS: B

Dentinogenesis is the formation of dentin. Amelogenesis is the formation of enamel. Dens in dente is a developmental anomaly called a tooth within a tooth. Odontogenesis is tooth development in the human embryo. REF: Teeth, page 151

OBJ: 4

5. The first branchial arch divides into two maxillary processes and the _____ process. a. mandibular b. frontal c. median nasal d. globular ANS: A

The first branchial arch divides into two maxillary processes and the mandibular process. The frontal process is a structure above the first branchial arch. The median nasal process develops from the frontal process. The globular process develops from the median nasal process. REF: Face, page 149

OBJ: 4

6. The body of the tongue develops from the a. frontal process. b. first branchial arch. c. second branchial arch. d. third branchial arch. ANS: B

The body of the tongue develops from the first branchial arch. The frontal process is above the first branchial arch. The second and third branchial arches form the base of the tongue. REF: Face, page 149

OBJ: 4

7. Which term defines the joining of two adjacent teeth by cementum only? a. Twinning b. Concrescence c. Cementogenesis d. Fusion ANS: B

Concrescence is the joining of two or more adjacent teeth by cementum. Twinning, or gemination, occurs when a single tooth germ begins to divide, resulting in the incomplete formation of two teeth. Cementogenesis is the formation of cementum. Fusion is the union of two adjacent tooth germs. REF: Vocabulary, page 148

OBJ: 5

8. Odontogenesis in the human embryo occurs at a. 3 weeks.


b. 5 weeks. c. 5 months. d. 1 month. ANS: B

Odontogenesis in the human embryo occurs at 5 weeks. The face begins proliferation and differentiation at 3 weeks. Formation of hard dental tissues begins at 5 months. There is no initial odontogenesis at 1 month in utero. REF: Teeth, page 150

OBJ: 4

9. This patient exhibits an extensive adhesion of the tongue to the floor of the mouth caused by

the short lingual frenum. What condition is suspected? a. Ankyloglossia b. Frenectomy c. Lingual thyroid d. Total ankyloglossia ANS: A

Ankyloglossia is an extensive adhesion of the tongue to the floor of the mouth caused by a short lingual frenum. A frenectomy is a surgical procedure performed to remove a portion of the lingual frenum in the treatment of ankyloglossia. Lingual thyroid is a smooth nodular mass at the base of the tongue posterior to the circumvallate papillae and near the midline. Total ankyloglossia rarely occurs. REF: Ankyloglossia, page 151

OBJ: 6

10. Clinically, the lingual thyroid nodule appears as a smooth nodular mass a. at the base of the tongue posterior to the circumvallate papillae. b. on the anterior ventral tongue. c. on the lateral borders of the middle third of the tongue. d. anterior to the circumvallate papillae. ANS: A

Clinically the lingual thyroid nodule appears as a smooth nodular mass at the base of the tongue posterior to the circumvallate papillae. The lingual thyroid nodule is not found on the anterior ventral tongue. The lingual thyroid nodule is not found on the lateral borders of the middle third of the tongue. The lingual thyroid nodule is not found anterior to the circumvallate papillae. REF: Lingual Thyroid, page 152

OBJ: 6

11. The most common cyst observed in the oral cavity is caused by pulpal inflammation and is

called a(n) _____ cyst. a. dentigerous b. eruption c. radicular d. primordial ANS: C


The radicular cyst is the most common cyst observed in the oral cavity. It is caused by pulpal inflammation. A dentigerous cyst forms around the crown of an unerupted or developing tooth. An eruption cyst is found in the soft tissue around the crown of an erupting tooth. A primordial cyst develops in place of a tooth, usually the third molar or posterior to an erupted third molar. REF: Developmental Cysts, page 152

OBJ: 9

12. This unilocular radiolucency around the crown of an unerupted second premolar is most

likely a a. normal developmental sac. b. dentigerous cyst. c. primordial cyst. d. lateral periodontal cyst. ANS: B

A dentigerous cyst is a well-defined unilocular radiolucency around the crown of an unerupted tooth. A normal developmental sac has a much smaller radiolucency around the crown. A primordial cyst develops in place of a tooth. The lateral periodontal cyst is most often seen in the mandibular cuspid and premolar region. REF: Odontogenic Cysts, page 153

OBJ: 8

13. The _____ is characterized by its unique histologic appearance and its frequent recurrence

rate. a. radicular cyst b. residual cyst c. dentigerous cyst d. odontogenic keratocyst ANS: D

The odontogenic keratocyst is characterized by its unique histologic appearance and its frequent recurrence rate. The radicular cyst is caused by pulpal inflammation. The residual cyst remains after extraction of the tooth with the radicular cyst. The radicular cyst is left behind and not removed. The dentigerous cyst is treated by complete removal of the cyst and the tooth involved. REF: Odontogenic Keratocyst (Keratocystic Odontogenic Tumor), page 153 OBJ: 8 14. The lateral periodontal cyst occurs most often on the lateral aspect of a tooth root, which is

usually the a. mandibular third molar. b. maxillary premolars. c. mandibular cuspid/premolars. d. maxillary anteriors. ANS: C


The mandibular cuspid/premolar area is the most common site for the lateral periodontal cyst. The mandibular third molar is not the site for a lateral periodontal cyst. Maxillary premolars are not the site for a lateral periodontal cyst. Maxillary anteriors are not the site for a lateral periodontal cyst. REF: Lateral Periodontal Cyst, Gingival Cyst, and Botryoid Odontogenic Cyst, page 156 OBJ: 10 15. Radiographically, this radiolucent cyst is often heart shaped, caused by the anatomic Y shape

of the area. It is called the _____ cyst. a. nasopalatine canal b. median palatine c. nasolabial d. globulomaxillary ANS: A

The nasopalatine canal cyst is often heart shaped. The median palatine cyst appears as a welldefined unilocular radiolucency in the midline of the palate. The nasolabial cyst is a soft tissue cyst with no alveolar bone involvement. The globulomaxillary cyst is a well-defined pearshaped radiolucency found between the roots of the maxillary lateral and cuspid. REF: Nasopalatine Canal Cyst, page 157 OBJ: 9 16. The _____ cyst has a strong predilection for females. a. lateral periodontal b. nasopalatine canal c. nasolabial d. gingival ANS: C

The nasolabial cyst has a strong predilection for females. The lateral periodontal cyst is most often found in males. The nasopalatine canal cyst has a predilection for males. The gingival cyst has no sex predilection. REF: Nasolabial Cyst, page 157

OBJ: 10

17. Which is not true about the thyroglossal tract cyst? a. It is found in individuals younger than 20 years. b. No sex predilection exists. c. Clinically, it is located below the hyoid bone. d. Conservative nonsurgical treatment is sufficient. ANS: D

Treatment of the thyroglossal tract cyst requires complete excision of the cyst and tract, usually including part of the hyoid bone and muscle within the tract. The thyroglossal tract cyst is found in individuals younger than 20 years. The thyroglossal tract cyst has no sex predilection. Clinically, the thyroglossal tract cyst is located below the hyoid bone. REF: Thyroglossal Tract Cyst, pages 159-160 18. Which is not considered a pseudocyst? a. Thyroglossal tract cyst

OBJ: 11


b. Static bone cyst c. Simple bone cyst d. Aneurysmal bone cyst ANS: A

The thyroglossal tract cyst can be lined by various types of epithelia. The static bone cyst is not lined with epithelium. The simple bone cyst is not lined with epithelium. An aneurysmal bone cyst is a pseudocyst that contains blood-filled spaces surrounded by multinucleated giant cells and fibrous connective tissue. REF: Pseudocysts, page 161

OBJ: 11

19. What is the pseudocyst filled with salivary gland tissue that may be an extension of the

sublingual gland? a. Ranula b. Static bone cyst c. Lymphoepithelial cyst d. Traumatic bone cyst ANS: B

The static bone cyst is a pseudocyst filled with salivary gland tissue that may be an extension of the sublingual gland. The ranula histologically is a mucocele or a mucous cyst. It occurs unilaterally on the floor of the mouth and is caused by obstruction of the duct. Lymphoepithelial cysts are not pseudocysts. They are commonly found in major salivary glands. Traumatic bone cyst is a pseudocyst. Surgical intervention reveals a void within the bone. REF: Stafne Defect, page 161

OBJ: 8

20. Total anodontia is often associated with a hereditary disturbance termed a. taurodontism. b. amelogenesis imperfecta. c. ectodermal dysplasia. d. cleidocranial dysplasia. ANS: C

Total anodontia may be associated with a hereditary disturbance called ectodermal dysplasia. Taurodontism is a genetic heterogeneous condition characterized by very large, pyramidshaped molars with large pulp chambers. Amelogenesis imperfecta is a group of inherited conditions affecting the enamel of teeth. In cleidocranial dysplasia, the patient has numerous supernumerary teeth. REF: Anodontia, page 162

OBJ: 13

21. The most common supernumerary tooth is termed a. distomolar. b. mesiodens. c. mulberry molar. d. Turner tooth. ANS: B


The mesiodens is the most common supernumerary tooth. The second most common supernumerary tooth is the fourth molar or distomolar. The mulberry molar is seen in congenital syphilis. A Turner tooth is a permanent tooth exhibiting enamel hypoplasia, the result of infection of the deciduous tooth. REF: Supernumerary Teeth, page 163

OBJ: 13

22. The supernumerary tooth in this illustration is a. a mesiodens. b. a dilaceration. c. the result of twinning. d. the result of gemination. ANS: A

A mesiodens is a supernumerary tooth found between the maxillary central incisors. Dilaceration is a sharp bend or curve in the root of a tooth. Twinning is when a single tooth germ attempts to divide. Gemination is the same as twinning (i.e., a single tooth germ attempts to divide). REF: Supernumerary Teeth, page 163

OBJ: 13

23. Nonerupted supernumerary teeth should be extracted because of which risk? a. Malignant tumor development b. Cysts around the crowns c. Internal resorption d. Condensing osteitis ANS: B

Nonerupted supernumerary teeth should be extracted because of the risk of developing cysts around the crowns. Supernumerary teeth do not develop into malignant tumors. Internal resorption is an inflammatory reaction in an erupted tooth. Condensing osteitis appears radiographically as a radiopaque area near the apices of teeth and is thought to be a reaction to low-grade infection. REF: Supernumerary Teeth, page 164

OBJ: 13

24. For which condition would pulp vitality be nonvital? a. Radicular cyst b. Median mandibular cyst c. Median palatal cyst d. Periapical cemento-osseous dysplasia ANS: A

The radicular cyst occurs at the root of a nonvital tooth. Teeth surrounding a median mandibular cyst would be vital. Teeth surrounding a median palatal cyst would be vital. In periapical cemento-osseous dysplasia, all teeth are vital. REF: Developmental Cysts, page 152

OBJ: 9

25. Dens in dente is a developmental anomaly often seen with a. extra cusps. b. a periapical lesion.


c. tuberculated premolars. d. supernumerary roots. ANS: B

Dens in dente is a developmental anomaly often seen with a periapical lesion. Dens evaginatus is an accessory occlusal cusp found on mandibular premolars. Tuberculated premolars occur when the mandibular premolars are affected with dens evaginatus. Dens in dente does not exhibit evidence of supernumerary roots. REF: Dens Invaginatus, page 169

OBJ: 15

26. Which tooth is most commonly affected by dens in dente? a. Maxillary central b. Mandibular lateral c. Maxillary lateral d. A supernumerary tooth ANS: C

The maxillary lateral is the tooth most commonly affected by dens in dente. The maxillary central is not the most common tooth seen with dens in dente. The mandibular lateral is not the most common tooth seen with dens in dente. A supernumerary tooth is not seen with dens in dente. REF: Dens Invaginatus, page 169

OBJ: 15

27. Another name for dens invaginatus is a. taurodontism. b. dens in dente. c. dens evaginatus. d. enamel pearl. ANS: B

Dens in dente is another name for dens invaginatus. Taurodontism is a developmental anomaly in which teeth exhibit elongated large pulp chambers and short roots. Dens evaginatus is a rare developmental anomaly in which an enamel cusp is found on the occlusal surface of mandibular premolars. Enamel pearl or enameloma is a projection of enamel found on the furcation area of maxillary molars. REF: Dens Invaginatus, page 169

OBJ: 15

28. The developmental anomaly seen in this radiographic image is a. taurodontism. b. mulberry molar. c. supernumerary roots on the mandibular premolars. d. dilaceration. ANS: C

This radiographic image shows supernumerary roots on the mandibular premolars. Taurodontic teeth, or bull’s teeth, show large pulp chambers and short roots, not seen in this radiograph. Mulberry molars result from congenital syphilis. Small globules of enamel make up the occlusal surface of the first molar. Dilaceration is a sharp bend or curve in the root.


REF: Supernumerary Roots, page 170

OBJ: 15

29. This radiographic image clearly shows which developmental anomaly? a. Dens in dente b. Periapical pathology (PAP) c. Caries d. Open contacts ANS: A

The radiograph shows dens in dente in a maxillary lateral incisor. PAP is associated with dens in dente in this radiographic image, but it is not a developmental anomaly. Caries is not a developmental anomaly. Open contacts are the result of the peg-shaped crown and do not represent a developmental anomaly. REF: Dens Invaginatus, Fig. 5.32, B, page 169

OBJ: 15

30. Enamel hypoplasia is the result of a disturbance of or damage to ameloblasts during enamel

matrix formation. Which is not be a factor? a. Genetics b. Ingestion of high concentrations of fluoride during tooth development c. Vitamin deficiency during tooth development d. Shingles ANS: D

Shingles is caused by the herpes zoster virus and is seen in adults. Genetic problems do cause enamel hypoplasia. High fluoride intake during tooth development does cause enamel hypoplasia. Vitamin deficiency during tooth development does cause enamel hypoplasia. REF: Abnormalities of Tooth Structure, pages 170-171

OBJ: 6

31. Pitting is the most common type of enamel hypoplasia seen in patients who have which

condition during tooth development? a. Febrile illness b. Drinking water with 2.4 ppm of fluoride during tooth development c. Congenital syphilis d. Herpes simplex ANS: A

Febrile illnesses such as measles and chickenpox cause enamel hypoplasia showing pitting of the enamel. Drinking water with twice the recommended fluoride content causes white flecks or chalky areas of the enamel. Congenital syphilis causes mulberry molars and Hutchinson incisors. Herpes simplex is characterized by oral ulcers involving the soft tissues and not enamel hypoplasia. REF: Enamel Hypoplasia, page 170

OBJ: 6

32. Ingesting water with four times the amount of fluoride causes a. brown-to-black staining. b. cusp fractures. c. white spots on the middle third of smooth crowns. d. increased dental caries.


ANS: A

Ingesting water with four times the amount of fluoride causes brown-to-black staining. The amount of fluoride ingested does not cause cusp fractures. White spots on the middle third of smooth crowns are enamel hypocalcification. Teeth affected by fluorosis are generally decay resistant. REF: Enamel Hypoplasia Resulting From Fluoride Ingestion, page 171 OBJ: 6 33. Which defines a disturbance of the maturation of the enamel matrix? a. Turner tooth b. Mulberry molar c. Premature birth d. Enamel hypocalcification ANS: D

Enamel hypocalcification is a disturbance of the maturation of the enamel matrix. Turner tooth results from enamel hypoplasia. Mulberry molar results from enamel hypoplasia associated with congenital syphilis. Premature birth can contribute to enamel hypoplasia. REF: Enamel Hypocalcification, page 172

OBJ: 6

34. The projection of white material seen at the furcation area in this maxillary molar is a

developmental anomaly. Which condition is suspected? a. Dens evaginatus b. Enamel pearl c. Supernumerary cusp d. Calculus ANS: B

The enamel pearl is a projection of enamel caused by abnormal displacement of ameloblasts during tooth formation. It is found near the furcation in maxillary molars. Dens evaginatus is an accessory enamel cusp found on the occlusal surfaces of mandibular premolars. A supernumerary cusp would be on or near the occlusal surface. Calculus is not a developmental anomaly. REF: Enamel Pearl, page 167

OBJ: 6

35. Regional odontodysplasia is a. a decrease in radiodensity seen on one or more unerupted teeth in a quadrant. b. a genetic condition. c. caused by systemic illness. d. most often seen in the mandible. ANS: A

Regional odontodysplasia involves a decrease in radiodensity seen in one or more unerupted teeth in a quadrant. Regional odontodysplasia is not a genetic condition. Regional odontodysplasia is not caused by systemic illness. Regional odontodysplasia is more often seen in the anterior maxilla. REF: Regional Odontodysplasia, page 172

OBJ: 6


36. Impacted teeth cannot erupt because of a. lack of eruptive force. b. physical obstruction. c. ankylosis. d. bone pathology. ANS: B

Impacted teeth cannot erupt because of physical obstruction. Lack of eruptive force does not play a role in eruption of impacted teeth. A tooth is ankylosed if it is fused to bone. This condition is especially common with retained deciduous teeth. Bone pathology can affect the eruption of teeth, but it is not the main reason that impacted teeth do not erupt. REF: Impacted and Embedded Teeth, page 172

OBJ: 16

37. Regional odontodysplasia is also referred to as a. hypodontia. b. ghost teeth. c. taurodontism. d. supernumerary teeth. ANS: B

Regional odontodysplasia is also referred to as ghost teeth. Hypodontia is the lack of one or more teeth. Taurodontism is a term used to describe a developmental anomaly in which teeth exhibit elongated large pulp chambers and short roots. Supernumerary teeth are extra teeth (more than the normal number) found in the dental arches. REF: Regional Odontodysplasia, page 172

OBJ: 6

38. The pseudocyst seen in this radiographic image is surrounded by salivary gland tissue.

It is a(n) _____ bone cyst. a. simple b. Stafne c. traumatic d. aneurysmal ANS: B

A Stafne bone cyst is a pseudocyst surrounded by salivary gland tissue. A simple bone cyst is the same as a traumatic bone cyst and is characterized by a radiolucent lesion that scallops around the roots of teeth. A traumatic bone cyst is the same as a simple bone cyst. An aneurysmal bone cyst is a pseudocyst that consists of blood-filled spaces surrounded by multinucleated giant cells and fibrous connective tissue. REF: Stafne Defect, page 161

OBJ: 12

39. The pear-shaped radiolucency observed in this radiographic image is most likely a _____ cyst. a. radicular b. globulomaxillary c. lateral periodontal d. nasopalatine canal ANS: B


The globulomaxillary cyst is a pear-shaped radiolucency found between the roots of a maxillary lateral and cuspid. The radicular cyst is a root end cyst found at the apex of a tooth that is usually involved with caries. A lateral periodontal cyst is usually found between the roots of the mandibular cuspid and premolar. The nasopalatine canal cyst is usually heart shaped and found near the apices of the maxillary centrals, lingual aspect. REF: Globulomaxillary Cyst, page 157

OBJ: 11

40. Multiple supernumerary teeth may be a component of which condition? a. Cleidocranial dysplasia b. Dermoid cyst c. Syphilis d. Static bone cyst ANS: A

Multiple supernumerary teeth may be a component of cleidocranial dysplasia or Gardner syndrome, both described in Chapter 6. The dermoid cyst does not have teeth in the cyst wall. Children with congenital syphilis have mulberry molars and Hutchinson incisors but not supernumerary teeth. Static bone cyst has nothing to do with supernumerary teeth. REF: Supernumerary Teeth, page 164

OBJ: 13

41. Which term best describes a disorder caused by abnormalities in the genetic makeup

transmitted from parent to offspring? a. Anomaly b. Inherited c. Congenital d. Developmental ANS: B

Inherited disorders are caused by abnormalities in the genetic makeup transmitted from parent to offspring. An anomaly is a marked deviation from normal that can be the result of congenital or hereditary defects. A congenital disorder is present at and existing from the time of birth. Developmental disorders occur when failure or disturbances occur during the complex series of cell division, multiplication, or differentiation. REF: Introduction, page 149

OBJ: 2

42. Proliferation is defined as a. congenital lack of teeth. b. formation of dentin. c. multiplication of cells. d. disposition in favor of something. ANS: C

Proliferation is the multiplication of cells. Anodontia is the congenital lack of teeth. Dentinogenesis is the formation of dentin. Predilection is a disposition in favor of something; preference. REF: Vocabulary, page 149

OBJ: 1

43. Odontogenic keratocysts are a clinical component of


a. b. c. d.

nevoid basal cell carcinoma syndrome. neurofibromatosis of von Recklinghausen. cherubism. Gardner syndrome.

ANS: A

Odontogenic keratocysts are a clinical component of nevoid basal cell carcinoma syndrome. REF: Odontogenic Cysts, page 153

OBJ: 8

44. Which tumor frequently arises from a dentigerous cyst? a. Sarcoma b. Ameloblastoma c. Odontoma d. Dens in dente ANS: B

An ameloblastoma frequently arises from a dentigerous cyst. REF: Dentigerous Cyst, page 153

OBJ: 9

45. During embryonic development of the face, the frontal process divides into three parts. These

three parts include the median nasal process, the right lateral nasal process, and the left lateral nasal process. a. Both statements are true. b. Both statements are false. c. The first statement is true; the second is false. d. The first statement is false; the second is true. ANS: A

During embryonic development of the face, the frontal process divides into three parts. These three parts include the median nasal process, the right lateral nasal process, and the left lateral nasal process. Both statements are true. REF: Face, page 149

OBJ: 4

46. A small elevated mass of thyroid tissue located near the foramen cecum or posterior lateral

borders of the tongue, which forms as a result of failure of the embryonic thyroid tissue to migrate to its proper position, is called a(n) a. ameloblastic fibroma. b. hemangioma. c. lingual thyroid nodule. d. thyroglossal duct cyst. ANS: C

A small elevated mass of thyroid tissue located near the foramen cecum or posterior lateral borders of the tongue, which forms as a result of failure of the embryonic thyroid tissue to migrate to its proper position, is a lingual thyroid nodule. An ameloblastic fibroma is a mixed odontogenic tumor. A hemangioma is a benign proliferation of capillaries. A thyroglossal duct cyst is located below the hyoid bone. REF: Lingual Thyroid, page 152

OBJ: 6


47. The deformity seen here with a bend in root apices is characteristic of a. dilaceration. b. gemination. c. fusion. d. concrescence. ANS: A

Dilaceration refers to an abnormal curve or angle in the root. Gemination is when a single enamel organ (tooth germ) divides partially. Fusion is the union of two normally separated adjacent tooth germs. Concrescence is the union of two independently formed teeth by cementum. REF: Dilaceration, page 167

OBJ: 15

48. The cyst that appears in the bone in this radiographic image surrounds the fully formed crown

of an unerupted premolar. The dental hygienist should refer to this as a(n) _____ cyst. a. eruption b. follicular c. lateral periodontal d. primordial ANS: B

A follicular cyst, also called a dentigerous cyst, appears in the bone in this radiograph surrounding a fully formed crown of an unerupted premolar. An eruption cyst is similar to a follicular cyst but is found in the soft tissue around the crown of an erupting tooth. A lateral periodontal cyst is seen most often in the mandibular cuspid and premolar area. It presents as an asymptomatic, unilocular or multilocular radiolucent lesion located on the lateral aspect of a tooth root. A primordial cyst develops in place of a tooth. REF: Odontogenic Cysts, page 153

OBJ: 6

49. When two or more teeth are joined by cementum, as shown in this picture, it is termed a. concrescence. b. dilaceration. c. enamel pearl. d. gemination. ANS: A

Concrescence occurs when two adjacent teeth are united by cementum. Dilaceration refers to an abnormal curve or angle in the root. Enamel pearl is a small spherical enamel projection located on a root surface. Gemination occurs when a single enamel organ (tooth germ) divides partially. REF: Concrescence, page 166

OBJ: 15

50. Periapical radiographic examination reveals a well-defined unilocular radiolucency located in

the midline of the hard palate.


The diagnosis is _____ cyst. a. nasolabial b. globulomaxillary c. branchial cleft d. median palatine ANS: D

A median palatine cyst is a well-defined unilocular radiolucency located in the midline of the hard palate. A nasolabial cyst is a soft tissue cyst with no alveolar bone involvement. A globulomaxillary cyst is a well-defined, pear-shaped radiolucency found between the roots of the maxillary lateral incisor and cuspid. A branchial cleft cyst is located on the lateral neck at the anterior border of the sternocleidomastoid muscle. REF: Median Palatine Cyst, page 157

OBJ: 11

51. This patient is healthy with no history of local or systemic infection or disease. The patient’s

teeth are caries free, as are all of the teeth of all of the patients who exhibit this defect. This is characteristic of a. fluorosis. b. Hutchinson incisors. c. a Turner tooth. d. attrition. ANS: A

Fluorosis occurs from ingestion of a high concentration of fluoride during tooth development. The teeth affected by fluorosis are generally decay resistant. Hutchinson incisors are a result of congenital syphilis. A Turner tooth is the result of infection from a deciduous tooth. Attrition is the result of the wearing away of tooth structure during mastication. REF: Enamel Hypoplasia Resulting From Fluoride Ingestion, page 171 OBJ: 3 52. When counting the maxillary anterior teeth of this adolescent patient, it appears that five are

present clinically, if the large tooth is counted as one. A radiographic image reveals that this large central tooth has two roots. This tooth demonstrates a. geminism. b. concrescence. c. dilaceration. d. fusion. ANS: D

Fusion is the union of two normally separated adjacent tooth germs. Gemination is when a single enamel organ (tooth germ) divides partially. Concrescence occurs when two adjacent teeth are united by cementum. Dilaceration refers to an abnormal curve or angle in the root. REF: Fusion, page 165

OBJ: 15


53. This enlargement on the lateral neck of this patient has been present for months and is slowly

increasing in size. It is painless and feels soft. Histologic examination shows this to be an epithelium-lined sac filled with clear, yellow fluid. It is a _____ cyst. a. thyroglossal duct b. branchial c. median palatine d. globulomaxillary ANS: A

A thyroglossal duct cyst appears on the lateral neck and slowly increases in size. It is painless and feels soft. Histologic examination shows an epithelium-lined sac filled with clear, yellow fluid. A branchial cleft cyst is located on the lateral neck at the anterior border of the sternocleidomastoid muscle. A median palatine cyst is a well-defined unilocular radiolucency located in the midline of the hard palate. A globulomaxillary cyst is a well-defined, pearshaped radiolucency found between the roots of a maxillary lateral incisor and cuspid. REF: Thyroglossal Tract Cyst, pages 159-160

OBJ: 11

54. This patient exhibits an accessory cusp located in the cingulum of the maxillary right lateral

permanent incisor. This can be diagnosed as a a. talon cusp. b. dens in dente. c. taurodontism. d. dens evaginatus. ANS: A

A talon cusp is an accessory cusp located in the cingulum of a maxillary or mandibular permanent incisor. A dens in dente is a developmental anomaly that results when the enamel organ invaginates into the crown of a tooth before mineralization. Taurodontism is a term used to describe a developmental dental anomaly in which the teeth exhibit elongated, large pulp chambers and short roots. Dens evaginatus is an accessory enamel cusp found on the occlusal tooth surface. REF: Talon Cusp, page 168

OBJ: 15

55. The radiographic image of this patient exhibits a biloculated, well-defined radiolucency lateral

to the tooth root. It is asymptomatic. This is a _____ cyst. a. residual b. follicular c. lateral periodontal d. primordial ANS: C

A lateral periodontal cyst is named for its location. It presents as an asymptomatic radiolucency located on the lateral aspect of a tooth root. A residual cyst is a radicular cyst that remains after extraction of the offending tooth. A follicular cyst forms around the crown of an unerupted or developing tooth. A primordial cyst develops in place of a tooth. REF: Lateral Periodontal Cyst, Gingival Cyst, and Botryoid Odontogenic Cyst, page 156 OBJ: 12


56. Microdontia most commonly occurs in a. maxillary laterals and third molars. b. maxillary canine. c. mandibular molars. d. mandibular incisors and molars. ANS: A

Microdontia most commonly occurs in maxillary laterals and third molars. Microdontia does not commonly occur in the maxillary canine. Microdontia does not commonly occur in the mandibular molars. Microdontia does not commonly occur in the mandibular incisors and molars. REF: Microdontia, page 164

OBJ: 14

57. A solitary hypoplastic defect in this dentition is located on the facial surface of a permanent

maxillary central incisor. The most likely cause of this defect is a. a dietary deficiency during tooth formation. b. absence of the primary mandibular central incisor. c. physical injury of the primary maxillary central incisor. d. neonatal hypoplasia of the primary anterior teeth. ANS: C

Hypoplastic defects in a patient’s dentition are indicative of physical injury of the primary teeth. A dietary deficiency during tooth formation would not typically appear in only one tooth. This description is not indicative of an absence of the primary mandibular central incisor. This description is not characteristic of neonatal hypoplasia of the primary anterior tooth. REF: Enamel Hypoplasia Resulting From Local Infection or Trauma, pages 170-171 OBJ: 3 58. During tooth development, ectoderm and ectomesenchymal cells give rise to each of the

following except one. Which one is the exception? a. Periodontal ligament b. Ameloblasts c. Odontoblasts d. Cementoblasts ANS: A

The dental sac that surrounds the developing tooth germ provides cells that form the periodontal ligament. Ectoderm and ectomesenchymal cells give rise to ameloblasts. Ectoderm and ectomesenchymal cells give rise to odontoblasts. Ectoderm and ectomesenchymal cells give rise to cementoblasts. REF: Teeth, pages 150-151

OBJ: 4

59. Deciduous teeth in which bone has fused to cementum and dentin, preventing exfoliation of

the deciduous tooth and eruption of the underlying permanent tooth are termed a. embedded. b. ankylosed. c. impacted.


d. erupted. ANS: B

A tooth is ankylosed if it is fused to bone. This condition is especially common with retained deciduous teeth. Embedded teeth do not erupt because of a lack of eruptive force. Impacted teeth cannot erupt because of physical obstruction. Erupted teeth are not fused to cementum and dentin. REF: Ankylosed Teeth, page 173

OBJ: 16

60. Odontogenesis in the human embryo takes place at approximately a. 5 weeks. b. 2 months. c. 3 months. d. at birth. ANS: A

Odontogenesis in the human embryo takes place at approximately 5 weeks. Odontogenesis in the human embryo takes place before 2 months. Odontogenesis in the human embryo takes place before 3 months. Odontogenesis in the human embryo takes place before birth. REF: Teeth, page 150

OBJ: 4

61. What is a condition likely to reveal ankylosed teeth? a. Presence of a dentigerous cyst b. Existence of supernumerary teeth c. Orthodontic appliances d. Retained deciduous teeth ANS: D

Ankylosed teeth are fused to the alveolar bone, a condition especially common with deciduous teeth. A dentigerous cyst surrounds the crown of an unerupted or impacted tooth. Supernumerary teeth are not associated with ankylosed teeth. Orthodontic appliances are not associated with ankylosed teeth. REF: Vocabulary, page 148

OBJ: 1

62. With concrescence, which tissue unites two adjacent teeth? a. Enamel b. Dentin c. Cementum d. Pulp ANS: C

The condition in which two adjacent teeth become united by cementum is termed concrescence. Enamel is not involved with concrescence. Dentin is not involved with concrescence. Pulp is not involved with concrescence. REF: Vocabulary, page 148

OBJ: 1

63. After the tooth erupts into the oral cavity, how long is it before the root length is complete? a. 6 months


b. 1 year c. 1–4 years d. 2–6 years ANS: C

Root length is not completed until 1–4 years after the tooth erupts into the oral cavity. REF: Teeth, page 151

OBJ: 4

64. Which cyst develops from a preexisting periapical granuloma found at the apex of a nonvital

tooth? a. Radicular b. Follicular c. Eruption d. Calcifying odontogenic ANS: A

The radicular (or periapical) cyst is always associated with a nonvital tooth; it develops from a preexisting periapical granuloma found at the apex of a nonvital tooth. The follicular (dentigerous) cyst forms around the crown of an unerupted or developing tooth. The eruption cyst is found in the soft tissue around the crown of an erupting tooth. The calcifying odontogenic cyst resembles the epithelium of the ameloblastoma. REF: Developmental Cysts, page 152

OBJ: 6

65. What is the radiographic feature of a cyst found within soft tissue? a. Unilocular b. Multilocular c. Diffuse d. No radiographic features are evident. ANS: D

No radiographic features are seen when a cyst is found within soft tissue. Unilocular refers to a single rounded compartment. Multilocular describes multiple rounded compartments that may appear “soap bubble–like.” Diffuse denotes a border that is not well defined, and the parameters of the lesion are unknown. REF: Developmental Cysts, page 152

OBJ: 6

66. An asymptomatic, well-defined unilocular radiolucency was discovered in the region of tooth

#32 on a panoramic image of a young adult patient. This tooth had never formed and therefore was never extracted. Identify this cyst: a. Primordial b. Odontogenic keratocyst c. Periapical d. Static bone cyst ANS: A


The primordial cyst develops in place of a tooth and is most commonly found in place of the third molar. The odontogenic keratocyst is often seen in the mandibular third molar region, yet often appears as a multilocular radiolucency that can move teeth and resorb tooth structure. The periapical cyst is always associated with a nonvital tooth. The static bone cyst is a welldefined radiolucency seen in the posterior mandible; clinically, an anatomic depression may be felt in this area on the lingual side of the mandible. REF: Primordial Cyst, page 153

OBJ: 6

67. The following groups of teeth are most often missing with hypodontia except one. Which is

the exception? a. Third molars b. Mandibular canines c. Maxillary lateral incisors d. Mandibular second premolars ANS: B

The mandibular canines are not typically missing in a case of hypodontia. The teeth most often missing with hypodontia include the maxillary and mandibular third molars. The teeth most often missing with hypodontia include the maxillary lateral incisors. The teeth most often missing with hypodontia include the mandibular second premolars. REF: Development Abnormalities of Teeth, page 162

OBJ: 13

68. Which term is unlike the others? a. Distomolar b. Supernumerary c. Hypodontia d. Mesiodens ANS: C

Hypodontia refers to a lack of one or more teeth. A distomolar is an extra tooth, also known as a maxillary fourth molar. Supernumerary is a term to describe extra teeth found in the dental arches. A mesiodens is a supernumerary tooth located between the maxillary central incisors at the midline. REF: Development Abnormalities of Teeth, pages 162-164

OBJ: 13

69. Which teeth are most commonly affected by microdontia? a. #7 and #16 b. #10 and #17 c. #17 and #24 d. #25 and #31 ANS: A

The maxillary lateral incisor and maxillary third molar are the teeth most commonly affected by microdontia. REF: Development Abnormalities of Teeth, page 164 70. Which statement about macrodontia is true? a. A common developmental anomaly.

OBJ: 14


b. Commonly affects a single tooth. c. Seen in cases of pituitary gigantism. d. Treatment involves extraction and prosthetic replacement. ANS: C

Macrodontia is seen occasionally in cases of pituitary gigantism. Macrodontia is an uncommon developmental anomaly in which one or more teeth are larger than normal. Macrodontia affecting a single tooth is uncommon. No treatment is indicated for macrodontia. REF: Development Abnormalities of Teeth, page 165

OBJ: 14

71. Since the dens in dente is often a nonvital tooth, it may be seen in association with a. an impacted tooth. b. a periapical lesion. c. swelling and displacement of surrounding teeth. d. malocclusion. ANS: B

The dens in dente is vulnerable to caries and pulpal infection and therefore may be associated with a periapical lesion. Dens in dente is not associated with an impacted tooth. Dens in dente does not cause swelling or displacement of surrounding teeth. Dens in dente is not associated with malocclusion. REF: Dens Invaginatus, page 169

OBJ: 15

72. Your patient presents with several horizontal rows of deep pits traversing the surfaces of the

permanent central and lateral incisors, canines, and first molars. The pits are stained and unsightly. Which condition is suspected? a. Dens in dente b. Concrescence c. Enamel hypoplasia d. Oligodontia ANS: C

Enamel hypoplasia is the incomplete or defective formation of enamel, causing the alteration of tooth form or color. Dens in dente is a developmental anomaly that results when the enamel organ invaginates into the crown of a tooth before mineralization. Concrescence is a condition in which two adjacent teeth are united by cementum. Oligodontia is a term describing a type of hypodontia in which six or more teeth are congenitally missing. REF: Abnormalities of Tooth Structure, page 170

OBJ: 15

73. Pitting of the enamel may be seen in these conditions except one. Which is the exception? a. Measles b. Vitamin A deficiency c. Scarlet fever d. Talon cusp ANS: D


The presence of a talon cusp does not cause pitting of the enamel. Febrile illnesses, such as measles, that occur during the time of tooth formation can result in pitting of the enamel. Vitamin deficiencies, such as vitamins A, C, and D, that occur during the time of tooth formation can result in pitting of the enamel. Febrile illnesses, such as scarlet fever, that occur during the time of tooth formation can result in pitting of the enamel. REF: Abnormalities of Tooth Structure, page 170

OBJ: 15

74. The presence of Hutchinson incisors and mulberry molars would indicate the presence of

which condition? a. Hepatitis B b. HIV disease c. Syphilis d. Chickenpox ANS: C

Congenital syphilis is transmitted from an infected mother to her fetus; teeth affected in the child include the incisors and molars. Hepatitis B does not cause any changes in the sizes of permanent teeth. HIV disease does not cause any changes in the sizes of permanent teeth. Chickenpox that occurs during the time of tooth formation may result in the pitting of enamel. REF: Enamel Hypoplasia Resulting From Congenital Syphilis, page 171 OBJ: 15 75. Trauma or change in the environment at the time of birth can eventually cause enamel

hypoplasia in the child. Which cells are so sensitive and easily damaged to cause this defect? a. Ameloblasts b. Neutrophils c. Macrophages d. C-reactive proteins ANS: A

Ameloblasts form and are one of the most sensitive cell groups in the body. Neutrophils are the first white blood cells to arrive at a site of injury. Macrophages are the second type of white blood cell to arrive at a site of injury. C-reactive proteins are nonspecific proteins that become elevated during episodes of acute inflammation or infection. REF: Abnormalities of Tooth Structure, page 170

OBJ: 15


Chapter 06: Genetics Ibsen: Oral Pathology for the Dental Hygienist, 7th Edition MULTIPLE CHOICE 1. Which statement is correct concerning chromosomes? a. Chromosomes are located in the of the cell. b. Mature germ cells (ova and spermatozoa) contain 46 chromosomes. c. Chromosomes can only be seen clearly under a microscope when the nucleus and

cell are dividing. d. Chromosomes contain RNA, which directs the production of amino acids,

polypeptides, and proteins by the cell. ANS: C

Chromosomes can only be seen clearly under a microscope when the nucleus and cell are dividing. At other times the genetic material is dispersed in the nucleus. Chromosomes are located in the nucleus of the cell. They contain the hereditary units called genes that are transmitted from one generation to another. Each cell of the human body, with the exception of mature germ cells, has 46 chromosomes. Half of these chromosomes are derived from the father and the other half from the mother. Chromosomes contain DNA, which directs the production of amino acids, polypeptides, and proteins by the cell. REF: Chromosomes, page 188

OBJ: 1

2. The gap 2 (G2) phase of mitosis a. begins when mitotic division ends. b. ends when mitotic division begins. c. precedes the S phase. d. is when replication of DNA takes place. ANS: B

The G2 phase of mitosis ends when mitotic division takes place and follows the S phase. Replication of DNA takes place in the S phase. The gap 2 (G2) phase of mitosis ends when mitotic division begins. The G2 phase of mitosis follows the S phase. Replication of DNA takes place during the S phase. REF: Mitosis, page 188

OBJ: 2

3. The correct order for the stages of mitosis is a. prophase, anaphase, metaphase, telophase. b. prophase, metaphase, anaphase, telophase. c. prophase, metaphase, telophase, anaphase. d. metaphase, prophase, telophase, anaphase. ANS: B


The correct order for the stages of mitosis is prophase, metaphase, anaphase, telophase. During prophase the chromosomes are lining up toward metaphase; in metaphase, the chromosomes stain intensely and are arranged almost symmetrically at both sides of the center of the cell; in anaphase and telophase the chromatids are in the process of splitting. Metaphase occurs before anaphase during mitosis. Anaphase occurs before telophase during mitosis. Prophase is the first stage of mitosis, and anaphase occurs before telophase. REF: Stages of Mitosis, page 188

OBJ: 2

4. Primitive germ cells have _____ chromosomes and are termed _____. a. 23; diploid b. 46; haploid c. 23; haploid d. 46; diploid ANS: D

Primitive germ cells have 46 chromosomes and are called diploid, with two chromosomes for each pair. Meiosis is a two-step special type of cell division in which the primitive germ cells reduce their chromosome number by half and become mature germ cells. Primitive germ cells have 46 rather than 23 chromosomes. Primitive germ cells are called diploid rather than haploid. Primitive germ cells have 46 chromosomes and are called diploid. REF: Meiosis, page 189

OBJ: 3

5. Points of contact between the chromatid of one chromosome and the chromatid of the other

chromosome of a pair, where crossing over and exchange of chromosome segments occur are termed a. first meiosis. b. second meiosis. c. chiasmata. d. interpolation. ANS: C

Points of contact between the chromatid of one chromosome and the chromatid of the other chromosome of a pair, where crossing over and exchange of chromosome segments occur, are called chiasmata. This special aspect of the first meiosis takes place at metaphase. Crossing over occurs during first meiosis, but the contacts between pairs of chromosomes occur at chiasmata. Crossing over occurs during first meiosis; the second stage of meiosis is essentially a mitotic division in which each chromosome splits longitudinally. Crossing over and exchange of chromosome segments occur at the chiasmata, not interpolation. REF: First Meiosis, page 189

OBJ: 3

6. After metaphase of the first meiotic division a. the chromosomes split at the centromere and separate from one another. b. the chromosomes do not split at the centromere but separate from one another. c. each member of the pair migrates to one of the new cells, each of which contains

46 chromosomes but twice the final amount of DNA. d. each member of the pair migrates to one of the new cells, each of which contains 23 chromosomes but half the final amount of DNA.


ANS: B

After metaphase of the first meiotic division, the chromosomes do not split at the centromere but separate from one another. Each member of the pair migrates to one of the new cells, each of which contains 23 chromosomes but twice the final amount of DNA. After metaphase of the first meiotic division, the chromosomes do not split at the centromere but separate from one another. Each member of the pair migrates to one of the new cells, each of which contains 23 rather than 46 chromosomes but twice the final amount of DNA. Each member of the pair migrates to one of the new cells, each of which contains 23 chromosomes but twice rather than half the final amount of DNA. REF: First Meiosis, page 190

OBJ: 3

7. Which statement is true of oogenesis? a. Oogenesis starts around the time of puberty. b. The second meiosis is completed at the beginning of ovulation. c. Nondisjunction is more prevalent in male spermatogenesis than in female

oogenesis. d. The older the woman, the greater the chance of shedding a trisomic ovum. ANS: D

The older the woman, the greater the chance of shedding a trisomic ovum. Oogenesis starts around the third month of prenatal life, and the future ova remain in suspended crossing-over from about the time of birth until the time ovulation starts. The first meiosis rather than second meiosis is completed at the beginning of ovulation. Nondisjunction is more prevalent in female oogenesis than in male spermatogenesis. REF: First Meiosis, page 190

OBJ: 3

8. Features of second meiosis include a. a replication of DNA before initiation. b. essentially a mitotic division in which each chromosome splits longitudinally. c. formation of four cells from the parent cell. d. the formation of 2n DNA. ANS: B

Features of second mitosis include essentially a mitotic division in which each chromosome splits longitudinally. Replication of DNA does not occur before the second meiosis. After splitting, two cells rather than four are formed from the parent cell. The second mitosis results in the formation of two cells from the parent cell, each containing 1n DNA. REF: Second Meiosis, page 190

OBJ: 3

9. According to the Lyon hypothesis during the early period of embryonic development, the a. genetic activity of both of the X chromosomes in each cell of a female embryo is

inactivated. b. genetic activity of one of the X chromosomes in each cell of a female embryo is

inactivated. c. genetic activity of the Y chromosome in each cell of a male embryo is inactivated. d. inactivated chromosome forms a structure known as the pronucleus. ANS: B


According to the Lyon hypothesis, during the early period of embryonic development, the genetic activity of one of the X chromosomes in each cell of a female embryo is inactivated. The genetic activity of one of the two X chromosomes in each cell of a female embryo is inactivated. The genetic activity of one of the two X chromosomes in each cell of a female embryo rather than the genetic activity of the Y chromosome in each cell of a male embryo is inactivated. The inactivated chromosome forms a structure known as the Barr body. The Barr body appears as a dark dot at the periphery of the nucleus. REF: Lyon Hypothesis, page 190

OBJ: 4

10. In DNA a sequence of thymine, adenine, and cytosine (TAC) is always matched by a. adenine, cytosine, and guanine (ACG). b. guanine, adenine, and cytosine (GAC). c. adenine, thymine, and guanine (ATG). d. a five-carbon sugar and a phosphate. ANS: C

In DNA a sequence of thymine, adenine, and cytosine (TAC) is always matched by adenine, thymine, and guanine (ATG). The base adenine is always bound to the base thymine, and guanine is always bound to cytosine. Thymine, adenine, and cytosine (TAC) are always matched by adenine, thymine, and guanine (ATG). Thymine, guanine, and cytosine would be matched by adenine, cytosine, and guanine (ACG). Thymine, adenine, and cytosine (TAC) are always matched by adenine, thymine, and guanine (ATG). Cytosine, thymine, and guanine would be matched by guanine, adenine, and cytosine (GAC). The basic unit of DNA is called a nucleotide, which is formed by a nitrogen-containing base, a five-carbon sugar, and a phosphate. REF: Deoxyribonucleic Acid, page 190

OBJ: 1

11. During translation a. polypeptides form an amino acid, and one or more amino acids form a protein. b. amino acids form a polypeptide, and one or more polypeptides form a protein. c. proteins form a polypeptide, and one or more polypeptides form an amino acid. d. amino acids form a protein, and one or more proteins form a polypeptide. ANS: B

During translation, amino acids form a polypeptide, and one or more polypeptides form a protein. During translation, amino acids form a polypeptide, and one or more polypeptides form a protein. During translation, amino acids form a polypeptide, and one or more polypeptides form a protein. During translation, amino acids form a polypeptide, and one or more polypeptides form a protein. REF: Deoxyribonucleic Acid, pages 190-191

OBJ: 1

12. Which type of RNA carries the message for translation of DNA to ribosomes in the

cytoplasm? a. mRNA b. tRNA c. rRNA d. hnRNA


ANS: A

mRNA carries the message for transcription of DNA to ribosomes in the cytoplasm. tRNA transfers amino acids from the cytoplasm to the mRNA, positioning amino acids in the proper sequence to form polypeptides and subsequent proteins. rRNA combines with several polypeptides to form ribosomes. hnRNA is found within the nucleus. It is the precursor of mRNA. REF: Types of Ribonucleic Acid, page 191

OBJ:

1

13. Genes that are located at the same level in homologous chromosomes and that dictate the

same functions or characteristics are termed a. consanguineous. b. chromatids. c. alleles. d. precocious. ANS: C

Genes that are located at the same level in homologous chromosomes and that dictate the same functions or characteristics are called alleles. Genes that are located at the same level in homologous chromosomes and that dictate the same functions or characteristics are called alleles, not consanguineous. Genes that are located at the same level in homologous chromosomes and that dictate the same functions or characteristics are called alleles, not chromatids. Genes that are located at the same level in homologous chromosomes and that dictate the same functions or characteristics are called alleles, not precocious. REF: Genes and Chromosomes, page 192

OBJ: 1

14. If both loci for blood are AO, the a. person is said to have blood group O. b. person is said to have blood group A. c. person would be homozygous. d. characteristic is recessive. ANS: B

If both loci for blood are AO, the person is said to have blood group A. Only if both loci are empty does the person have blood group O. When the allelic genes are identical, the person is said to be homozygous for that gene or a homozygote. When the genes are different, the person is said to be heterozygous for that gene or a heterozygote. If a gene can express its effect clinically with a single dose (heterozygous), as in the combination AO (which equals blood group A), the characteristic is dominant. If the gene needs a double dose to exhibit its action (homozygous), the resulting characteristic is said to be recessive. REF: Genes and Chromosomes, page 192

OBJ: 1

15. Which is true about making a karyotype? a. Arginine may be used to arrest the mitosis of leukocytes at metaphase. b. Colchicine may be used to arrest the mitosis of leukocytes at metaphase. c. Colchicine may be used to arrest the mitosis of red blood cells at anaphase. d. It is used to evaluate molecular abnormalities in chromosomes. ANS: B


In making a karyotype colchicine may be used to arrest the mitosis of leukocytes at metaphase. A karyotype may use colchicine rather than arginine to arrest the mitosis of leukocytes at metaphase. In making a karyotype colchicine may be used to arrest the mitosis of leukocytes rather than red blood cells at metaphase. A karyotype is used to evaluate gross abnormalities in chromosomes. Molecular alterations occur at the DNA level and are not detectable microscopically. REF: Chromosomal Abnormalities, page 193

OBJ: 1

16. Which statement is true regarding gross chromosomal abnormalities involving alterations in

the number of human chromosomes? a. Aneuploidy is a complete second set of chromosomes, meaning a total of 46. b. Polyploidy is any number of chromosomes that do not represent an exact multiple of the total chromosome complement. c. Euploidy is a complete second set of chromosomes, meaning a total of 46. d. Aneuploidy may be represented by trisomy and monosomy. ANS: D

Aneuploidy may be represented by trisomy (a pair with an identical extra chromosome) and monosomy (a missing chromosome from a pair). Aneuploidy is any extra number of chromosomes that do not represent an exact multiple of the total chromosome complement. Polyploidy is three or four complete sets of chromosomes. This has been described occasionally in humans and is incompatible with life. Euploidy is a complete second set of chromosomes, the total number being 92. REF: Gross Chromosomal Abnormalities, page 193

OBJ: 5

17. Most cases of Down syndrome are associated with a. maternal consumption of alcohol. b. late maternal age at the time of conception. c. estrogen supplements. d. inhalation of vapors from volatile substances. ANS: B

Most cases of Down syndrome are associated with late maternal age at the time of conception. It is the most frequent of the trisomies. Most cases of Down syndrome are associated with late maternal age at the time of conception. Fetal alcohol syndrome is associated with maternal consumption of alcohol. Most cases of Down syndrome are associated with late maternal age at the time of conception, not estrogen supplements. Most cases of Down syndrome are associated with late maternal age at the time of conception, not inhalation of vapors from volatile substances. REF: Trisomy 21, page 193

OBJ: 5

18. Which clinical syndrome caused by gross chromosomal abnormalities is associated with

bilateral cleft lip and palate, microphthalmia or anophthalmia, and polydactyly? a. Trisomy 13 b. Trisomy 21 c. Turner syndrome d. Klinefelter syndrome


ANS: A

Trisomy 13 is associated with bilateral cleft lip and palate, microphthalmia or anophthalmia, superficial hemangioma of the forehead or nape of the neck, growth retardation, severe mental retardation, polydactyly of the hands and feet, clenching of the fist with the thumb under the fingers, rocker-bottom feet, heart malformations, and several anomalies of the external genitals. Trisomy 21 is associated with slanted eyes. Patients are generally shorter than normal, and heart abnormalities are present in more than 30% of individuals. Intelligence level varies from near normal to marked retardation. Turner syndrome is characterized by a female phenotype. Clinically, these women are of short stature and have webbing of the neck and edema of the hands and feet. They frequently exhibit a low hairline on the nape of the neck. The chest is broad with wide-spaced nipples. The aorta is frequently abnormal, and body hair is sparse. Klinefelter syndrome is characterized by a male phenotype. These patients are taller than normal and have wide hips and a female pubic hair distribution. About 50% have gynecomastia, and intelligence levels are lower than normal in 10% of affected individuals. REF: Trisomy 21, page 194

OBJ: 5

19. The majority of patients with Turner syndrome have a _____ phenotype and _____

chromosomes. a. male; 47 b. female; 46 c. female; 45 d. male; 44 ANS: C

The majority of patients with Turner syndrome have a female phenotype, and in the majority of cases, the karyotype has the normal 44 autosomal chromosomes and only one X chromosome. A normal female would have two X chromosomes: one from the mother and one from the father. Most cases of Turner syndrome are caused by nondisjunction of the X chromosome in the paternal gamete. The majority of patients with Turner syndrome have a female rather than a male phenotype and 45 rather than 47 chromosomes. In the majority of cases, the karyotype has the normal 44 autosomal chromosomes and only one X chromosome. The majority of patients with Turner syndrome have a female phenotype and 45 rather than 46 chromosomes. In the majority of cases, the karyotype has the normal 44 autosomal chromosomes and only one X chromosome. The majority of patients with Turner syndrome have a female rather than a male phenotype and 45 rather than 44 chromosomes. In the majority of cases, the karyotype has the normal 44 autosomal chromosomes and only one X chromosome. REF: Turner Syndrome, page 194

OBJ: 5

20. For autosomal-dominant inheritance a. males will be affected more often than females. b. the risk of having an affected offspring is 25% when a person has a gene for the

condition. c. all of the offspring will be affected by a condition that is transmitted by autosomaldominant inheritance. d. an individual can carry a gene with a dominant effect without presenting any clinical manifestations. ANS: D


For autosomal-dominant inheritance, an individual can carry a gene with a dominant effect without presenting any clinical manifestations. For autosomal-dominant inheritance, males and females are equally affected. For autosomal-dominant inheritance, the risk of having an affected offspring is 50% for each pregnancy when a person has a gene for that condition. For autosomal-dominant inheritance, genetic risk is governed by chance. None, less than half, half, more than half, or all of the offspring could be affected by a condition that is transmitted by autosomal-dominant inheritance. REF: Autosomal-Dominant Inheritance, page 195

OBJ: 6

21. A man’s X chromosome is transmitted to _____ of his daughters and _____ of his sons. a. none; none b. none; all c. all; none d. all; all ANS: C

A man’s X chromosome is transmitted to all of his daughters and none of his sons. The X chromosome in his sons comes from the mother. Consequently, no male-to-male transmission of X-linked traits occurs. REF: X-Linked Inheritance, page 195

OBJ: 6

22. A mother who is a carrier of an X-linked recessive trait has a 50% risk of giving birth to a(n) a. affected son or daughter. b. carrier son or daughter. c. affected son or a carrier daughter. d. carrier son or an affected daughter. ANS: C

A mother who is a carrier of an X-linked recessive trait has a 50% risk of having an affected son or a carrier daughter. Both daughters and sons have a 50% risk of getting the X chromosome with the gene for that condition. REF: X-Linked Inheritance, page 195

OBJ: 7

23. Which statement is true concerning hemophilia A? a. It is inherited as an X-linked dominant condition. b. It is inherited as a Y-linked recessive condition. c. All X chromosomes are abnormal in the female carrier. d. Female carriers tend to bleed more than usual after extraction of teeth or scaling

and curettage. ANS: D

Female carriers tend to bleed more than usual after extraction of teeth or scaling and curettage. It is inherited as an X-linked recessive, not X-linked dominant, condition. It is inherited as an X-linked rather than a Y-linked recessive condition. Some of the canceled X chromosomes in the female carrier have the abnormal gene, and others have the normal one. The female carrier is a mosaic. REF: Lyon Hypothesis and X-linked Recessive Traits, page 196 OBJ: 7


24. Cyclic neutropenia is a. characterized by episodes that generally persist for 21 to 27 days. b. characterized by oral manifestations consisting of severe ulcerative gingivitis or

gingivostomatitis. c. caused by a deletion on the long arm of chromosome 11 regions 14-21. d. inherited as an autosomal recessive condition. ANS: B

Cyclic neutropenia is characterized by oral manifestations consisting of severe ulcerative gingivitis or gingivostomatitis. The cycles usually occur in intervals of 12 to 27 days rather than 21 to 27 days, but in some patients, the intervals may be extended to several months. These episodes generally persist for 2 to 3 days. The gene is located on the short arm of chromosome 19. The inheritance pattern is autosomal dominant. REF: Cyclic Neutropenia, page 196

OBJ: 8

25. Which statement is true regarding management of patients with cyclic neutropenia? a. Preventive antibiotic therapy is instituted to protect against secondary

opportunistic infections. b. Treatment should be done when the circulating neutrophil count is low to reduce

the risk of complications such as gingival hemorrhage and secondary infection. c. Dental hygiene care increases the risk of opportunistic infections in patients with

cyclic neutropenia. d. Patients are treated periodically with vitamin D to reduce symptoms. ANS: A

Preventive antibiotic therapy is instituted to protect against secondary opportunistic infections. Treatment should be done when the circulating neutrophil count is normal to reduce the risk of complications such as gingival hemorrhage and secondary infection. Dental hygiene care reduces the risk of opportunistic infections in patients with cyclic neutropenia. Patients are treated periodically with granulocyte colony-stimulating factor (G-CSF) to reduce symptoms. REF: Cyclic Neutropenia, page 196

OBJ: 8

26. Murray-Puretic-Drescher syndrome is also known as a. cherubism. b. gingival fibromatosis with multiple hyaline fibromas. c. gingival fibromatosis with hypertrichosis, epilepsy, and mental retardation

syndrome. d. Laband syndrome. ANS: B

Murray-Puretic-Drescher syndrome is also known as gingival fibromatosis with multiple hyaline fibromas. Cherubism is an inherited disorder affecting the jaw bones and facies. Gingival fibromatosis with hypertrichosis, epilepsy, and mental retardation syndrome and Laband syndrome are all forms of gingival fibromatosis; however, Murray-Puretic-Drescher syndrome is known as gingival fibromatosis with multiple hyaline fibromas. REF: Gingival Fibromatosis With Multiple Hyaline Fibromas, page 198 OBJ: 8


27. Cherubism a. is inherited as an autosomal-dominant disease with marked penetrance in females

and variable expressivity and incomplete penetrance in males. b. most frequently involves the maxilla. c. is characterized by a unilateral facial deformity. d. reveals a typical “soap-bubble” or multilocular appearance on radiographic images. ANS: D

Cherubism reveals a typical “soap-bubble” or multilocular appearance on radiographic images. It usually occupies the ascending ramus of the mandible and extends into the molar and premolar areas. Cherubism is inherited as an autosomal-dominant disease with marked penetrance in males and variable expressivity and incomplete penetrance in females. Cherubism most frequently involves the mandible, but the change can affect either the mandible or the maxilla. Cherubism is characterized by a bilateral facial deformity. REF: Cherubism, page 198

OBJ: 8

28. The areas of bony radiolucency seen in cherubism a. resemble a central giant cell granuloma when seen under the microscope. b. occur in place of the teeth, and radiographs will reveal complete anodontia. c. will increase in size as the patient matures, even until the seventh or eighth decade

of life. d. will resolve, leaving the patient without any sign of facial deformity later in life. ANS: A

The areas of bony radiolucency seen in cherubism resemble a central giant cell granuloma when seen under the microscope. The bone lesions interfere with tooth development and eruption. Most of the patients with cherubism have pseudoanodontia because of delayed eruption. The size of the jaws of patients with cherubism tends to increase rapidly until about puberty and then generally remains stable. The facial deformity seen in cases of cherubism remains for life. REF: Cherubism, page 199

OBJ: 8

29. Which disorder affecting the jaw bones and facies is characterized by fusion of the anterior

portion of the maxillary gingiva to the upper lip from canine to canine? a. Gardner syndrome b. Mandibulofacial dysostosis c. Ellis–van Creveld syndrome d. Cherubism ANS: C


Ellis–van Creveld syndrome is characterized by fusion of the anterior portion of the maxillary gingiva to the upper lip from canine to canine. This syndrome has an autosomal-recessive inheritance pattern. The gene for this syndrome has been mapped to the short arm of chromosome 4 region 16. Gardner syndrome is characterized by the presence of osteomas in various bones, especially the frontal bones, mandible, and maxilla. Mandibulofacial dysostosis is characterized by a facies with downward sloping of the palpebral fissures, a hypoplastic nose, hypoplastic malar bones with hypoplasia or absence of the zygomatic process, abnormal and misplaced ears, and a receding chin. Cherubism is characterized by a progressive bilateral facial swelling that appears when the patient is between 2 and 4 years of age. REF: Ellis–van Creveld Syndrome, page 199

OBJ: 9

30. A patient with Ellis–van Creveld syndrome will a. show polydactyly on the radial side of the forearm. b. be a little person. c. lack an anterior maxillary vestibular sulcus. d. lack a posterior mandibular sulcus. ANS: C

A patient with Ellis–van Creveld syndrome will lack an anterior maxillary vestibular sulcus. A patient with Ellis–van Creveld syndrome will show polydactyly on the ulnar side of the forearm, and fingernails and toenails will be hypoplastic and deformed. Individuals affected by Ellis–van Creveld syndrome are dwarfs (different from little people) because of distal shortening of the extremities. Other skeletal anomalies include curvature of the legs and feet. A patient with Ellis–van Creveld syndrome will lack an anterior maxillary vestibular sulcus rather than a posterior mandibular sulcus. REF: Ellis–van Creveld Syndrome, page 199

OBJ: 9

31. A patient with unilateral or bilateral aplasia of the clavicles will have which characteristics? a. Hyperplastic paranasal sinuses b. Short and wide neck c. Mushroom shape of the cranium because of premature closure of the fontanelles d. Be able to approximate their shoulders to the midline ANS: D

A patient with unilateral or bilateral aplasia of the clavicles will be able to approximate his or her shoulders to the midline. Various other bone anomalies can also be present. A patient with unilateral or bilateral aplasia of the clavicles will have paranasal sinuses that are lacking or hypoplastic rather than hyperplastic. A patient with unilateral or bilateral aplasia of the clavicles will have a long and narrow neck because of unilateral or bilateral aplasia or hypoplasia of the clavicles. A patient with unilateral or bilateral aplasia of the clavicles will have a cranium that develops a mushroom shape because the fontanelles remain open rather than close prematurely. REF: Cleidocranial Dysplasia, page 200

OBJ: 9

32. Which syndrome is characterized by intestinal polyps, which become malignant at age 30 and

after? a. Osteogenesis imperfecta


b. Nevoid basal cell carcinoma syndrome c. Mandibulofacial dysostosis d. Gardner syndrome ANS: D

Gardner syndrome is characterized by intestinal polyps, which become malignant at age 30 and after. It is also known as familial colorectal polyposis. Polyposis primarily affects the colon and rectum and generally develops before puberty. Some authors recommend intestinal resection when the polyps appear because their malignant transformation into adenocarcinoma is invariable, especially with increasing age. Osteogenesis imperfecta is characterized by defective collagen and results in abnormally formed bones that fracture easily. Nevoid basal cell carcinoma syndrome is characterized by mild hypertelorism and mild prognathism, with frontal and parietal enlargement and a broad nasal root. Mandibulofacial dysostosis is characterized by a mouth that appears fishlike, with downward sloping of the lip commissures. The facies shows downward sloping of the palpebral fissures, a hypoplastic nose, hypoplastic malar bones with hypoplasia or absence of the zygomatic process, abnormal and misplaced ears, and a receding chin. REF: Gardner Syndrome, page 200

OBJ: 9

33. Oral manifestations of nevoid basal cell carcinoma syndrome consist of multiple cysts of the

jaws that are a. central giant cell granulomas. b. peripheral giant cell granulomas. c. odontogenic keratocysts. d. calcifying epithelial odontogenic tumors. ANS: C

Oral manifestations of nevoid basal cell carcinoma syndrome consist of multiple cysts of the jaws that are odontogenic keratocysts. These cysts vary in size; they can be very large and have a marked tendency to recur after surgical removal. On occasion, an ameloblastoma arises in them as part of this syndrome. They develop as early as 5 to 6 years of age in some affected patients and interfere with normal development of the jawbones and teeth. Oral manifestations of nevoid basal cell carcinoma syndrome consist of multiple cysts of the jaws that are odontogenic keratocysts and not central giant cell granulomas. Oral manifestations of nevoid basal cell carcinoma syndrome consist of multiple cysts of the jaws that are odontogenic keratocysts and not peripheral giant cell granulomas. Oral manifestations of nevoid basal cell carcinoma syndrome consist of multiple cysts of the jaws that are odontogenic keratocysts and not calcifying epithelial odontogenic tumors. REF: Nevoid Basal Cell Carcinoma Syndrome, page 201

OBJ: 8

34. The basic defect responsible for osteogenesis imperfecta is produced by various mutations

affecting the genes that encode type I collagen, resulting in a. lack of otic ossicles. b. unilateral or bilateral aplasia or hypoplasia of clavicles. c. overgrowth of cortical bone in the midline of the palate. d. abnormally formed bones that fracture easily. ANS: D


The basic defect responsible for osteogenesis imperfecta is produced by various mutations affecting the genes that encode type I collagen, resulting in abnormally formed bones that fracture easily. Mandibulofacial dysostosis is associated with lack of otic ossicles. Cleidocranial dysplasia is associated with unilateral or bilateral aplasia or hypoplasia of clavicles. Torus palatinus is associated with a bony overgrowth at the midline of the hard palate. REF: Osteogenesis Imperfecta, page 201 OBJ: 8 35. Hard, dense, cortical bone on the buccal aspect of the alveolar ridge near the maxillary first

molar would be termed a a. radiolucent torus palatinus. b. radiopaque torus mandibularis. c. maxillary exotosis. d. radiolucent exostosis. ANS: C

Hard, dense, cortical bone on the buccal aspect of the alveolar ridge the maxillary first molar would be termed a radiopaque exostosis. Exostoses are generally asymptomatic unless traumatized and may be single, multiple, unilateral, and bilateral; these occur less frequently than either palatal or mandibular tori. Torus palatinus is a radiopaque bony overgrowth that occurs at the midline of the hard palate. Torus mandibularis is a radiopaque bony overgrowth that occurs on the lingual aspect of the mandible in the area of the premolar teeth. Hard, dense, cortical bone on the buccal aspect of the alveolar ridge by the maxillary first molar would be termed a radiopaque exostosis rather than radiolucent exostosis. REF: Maxillary Exostosis, page 202

OBJ: 8

36. Telangiectasias of the oral mucosa for a patient with hereditary hemorrhagic telangiectasia

will be especially prominent in which location? a. Tip and anterior dorsum of the tongue b. Palate c. Gingiva d. Buccal mucosa ANS: A

Telangiectasias of the oral mucosa for a patient with hereditary hemorrhagic telangiectasia will be especially prominent on the tip and anterior dorsum of the tongue. The palate, gingiva, and buccal mucosa are often affected but to a lesser degree. Hemorrhage from sites in the oral cavity, mainly the lips and tongue, is second in frequency to epistaxis. Telangiectasias of the oral mucosa for a patient with hereditary hemorrhagic telangiectasia will be especially prominent on the tip and anterior dorsum of the tongue rather than the palate. Telangiectasias of the oral mucosa for a patient with hereditary hemorrhagic telangiectasia will be especially prominent on the tip and anterior dorsum of the tongue rather than the gingiva. Telangiectasias of the oral mucosa for a patient with hereditary hemorrhagic telangiectasia will be especially prominent on the tip and anterior dorsum of the tongue rather than the buccal mucosa. REF: Hereditary Hemorrhagic Telangiectasia, page 203

OBJ: 8

37. Multiple mucosal neuroma syndrome includes multiple mucosal neuromas, medullary

carcinoma of the thyroid gland, and


a. b. c. d.

telangiectases. pheochromocytoma. gastrointestinal polyposis. blue sclera.

ANS: B

Multiple mucosal neuroma syndrome includes multiple mucosal neuromas, medullary carcinoma of the thyroid gland, and pheochromocytoma. The mucosal neuromas are prominent on the lips and the anterior dorsal surface of the tongue. They generally appear in the first few years of life. Medullary carcinoma of the thyroid has been diagnosed in more than 75% of patients with this syndrome; it generally develops in the second decade of life. Metastatic lesions develop frequently, and about 20% of patients die as a consequence of metastasis. Telangiectases are a component of Osler–Rendu–Parkes Weber syndrome. Gastrointestinal polyposis is associated with Peutz–Jeghers syndrome. Blue sclera is associated with osteogenesis imperfecta. REF: Multiple Mucosa Neuroma Syndrome, page 203

OBJ: 9

38. Early diagnosis of pheochromocytoma is imperative because of the high malignant potential

of a. b. c. d.

squamous cell carcinoma. basal cell carcinoma. thyroid carcinoma. osteogenic sarcoma.

ANS: C

Early diagnosis of pheochromocytoma is imperative because of the high malignant potential of thyroid carcinoma. Pheochromocytoma itself is a benign neoplasm that generally develops in ganglia around the adrenal glands. The tumor is often bilateral and is responsible for night sweats, high blood pressure, and episodes of severe diarrhea. The pheochromocytoma induces increased urinary levels of epinephrine and other substances. Early diagnosis of pheochromocytoma is imperative because of the high malignant potential of thyroid carcinoma rather than squamous cell carcinoma. Early diagnosis of pheochromocytoma is imperative because of the high malignant potential of thyroid carcinoma rather than basal cell carcinoma. Early diagnosis of pheochromocytoma is imperative because of the high malignant potential of thyroid carcinoma rather than osteogenic sarcoma. REF: Pheochromocytoma, page 204

OBJ: 9

39. Neurofibromatosis of von Recklinghausen is characterized by a. multiple neurofibromas appearing as macules of various sizes. b. inevitable malignant transformation. c. oral involvement in about 90% of patients. d. café au lait pigmentation of the skin in 90% of patients. ANS: D


Neurofibromatosis of von Recklinghausen is characterized by café au lait pigmentation of the skin in 90% of patients. Multiple neurofibromas, which appear as papules and growths of various sizes, are seen on the facial skin, especially the eyelids. Malignant transformation of the neurofibromas occurs in an estimated 3% to 15% of patients with neurofibromatosis. Oral involvement is seen in about 10% of patients and is characterized by single or multiple tumors at any location in the oral mucosa, the most frequent being the lateral borders of the tongue. REF: Neurofibromatosis of von Recklinghausen, page 204

OBJ: 9

40. Which syndrome is characterized by multiple melanotic macular pigmentations of the skin

and mucosa, which are associated with gastrointestinal polyposis? a. Osler–Rendu–Parkes Weber syndrome b. Van der Woude syndrome c. Gorlin syndrome d. Peutz–Jeghers syndrome ANS: D

Peutz–Jeghers syndrome is characterized by multiple melanotic macular pigmentations of the skin and mucosa, which are associated with gastrointestinal polyposis. Osler–Rendu–Parkes Weber syndrome is also known as hereditary hemorrhagic telangiectasia. Van der Woude syndrome consists of cleft lip-palate and congenital lip pits. Gorlin syndrome is also known as nevoid basal cell carcinoma syndrome. REF: Peutz–Jeghers Syndrome, page 204

OBJ: 10

41. Intestinal polyps seen with Peutz–Jeghers syndrome are a. associated with inevitable malignant transformation into adenocarcinomas. b. hamartomas. c. caused by the adenomatous polyposis coli (APC) gene located on the long arm of

chromosome 5. d. associated with osteomas in various bones, especially the frontal bones, mandible,

and maxilla. ANS: B

Intestinal polyps seen with Peutz–Jeghers syndrome are hamartomas. Intestinal polyps seen with Peutz–Jeghers syndrome develop mostly in the small intestine; only rarely do they undergo malignant transformation. Gardner syndrome is associated with inevitable malignant transformation of intestinal polyps into adenocarcinomas. The adenomatous polyposis coli (APC) gene responsible for Gardner syndrome is located on the long arm of chromosome 5. One of the basic components of Gardner syndrome is the presence of osteomas in various bones, especially the frontal bones, mandible, and maxilla. REF: Peutz–Jeghers Syndrome, page 204

OBJ: 10

42. Snow-capped amelogenesis imperfecta is a variation of type _____ amelogenesis imperfecta. a. I: hypoplastic b. II: hypocalcified c. III: hypomaturation d. IV: hypoplastic–hypomaturation ANS: C


Snow-capped amelogenesis imperfecta is a variation of type III: hypomaturation amelogenesis imperfecta. It is characterized by a hypomaturation of the surface enamel of the occlusal third of all teeth in both dentitions. The maxillary teeth are more severely affected with this whitish discoloration. The enamel in those areas is of regular hardness and smooth. It does not fracture or chip from the crown. Snow-capped amelogenesis imperfecta is a variation of type III: hypomaturation amelogenesis imperfecta rather than type I: hypoplastic amelogenesis imperfecta. Snow-capped amelogenesis imperfecta is a variation of type III: hypomaturation amelogenesis imperfecta rather than type II: hypocalcified amelogenesis imperfecta. Snowcapped amelogenesis imperfecta is a variation of type III: hypomaturation amelogenesis imperfecta rather than type IV: hypoplastic–hypomaturation amelogenesis imperfecta. REF: Amelogenesis Imperfecta, page 206

OBJ: 11

43. Which statement about the hereditary opalescent form of dentinogenesis imperfecta is true? a. It is associated with taurodontic teeth. b. Teeth have bulbous crowns with a color that varies from opalescent brown to

brownish blue. c. Large pulp chambers and root canals are seen on radiographs. d. Roots are long and thick with periapical radiopacities. ANS: B

The hereditary opalescent form of dentinogenesis imperfecta is associated with teeth that have bulbous crowns with a color that varies from opalescent brown to brownish blue. Taurodontic teeth are associated with the hypoplastic–hypomaturation type of amelogenesis imperfecta. No pulp chambers or root canals are seen on radiographs of patients with the hereditary opalescent form of dentinogenesis imperfecta. Roots are short and thin with periapical radiolucencies on radiographs of patients with the hereditary opalescent form of dentinogenesis imperfecta. REF: Dentinogenesis Imperfecta, page 206

OBJ:

12

44. The basic defect found with radicular dentin dysplasia lies in the a. stellate reticulum. b. epithelial root sheath. c. stratum intermedium. d. dental papilla. ANS: B

The basic defect found with radicular dentin dysplasia seems to lie in the epithelial root sheath. This guides the formation of the root. The basic defect found with radicular dentin dysplasia seems to lie in the epithelial root sheath and not the stellate reticulum. The basic defect found with radicular dentin dysplasia seems to lie in the epithelial root sheath and not the stratum intermedium. The basic defect found with radicular dentin dysplasia seems to lie in the epithelial root sheath and not the dental papilla. REF: Radicular Dentin Dysplasia, page 206

OBJ: 12

45. The primary dentition of a patient with coronal dentin dysplasia will appear a. normal. b. opaque with an amber color. c. translucent with an amber color.


d. translucent with a blue color. ANS: C

The primary dentition of a patient with coronal dentin dysplasia will appear translucent with an amber color. Radiographs show a lack of pulp chambers and small root canals. Permanent teeth present normal crown formation with normal color. Radiographs show thistle-shaped pulp chambers in single-rooted teeth and a bow tie appearance of the pulp chambers of permanent molars. The primary dentition of a patient with coronal dentin dysplasia will appear translucent with an amber color rather than normal. The primary dentition of a patient with coronal dentin dysplasia will appear translucent rather than opaque with an amber color. The primary dentition of a patient with coronal dentin dysplasia will appear translucent with an amber color rather than blue. REF: Coronal Dentin Dysplasia, page 206

OBJ: 12

46. Hypohidrotic ectodermal dysplasia is characterized by a. hypodontia, hypotrichosis, and hypohidrosis. b. hyperdontia, hypotrichosis, and hypohidrosis. c. hypodontia, hypertrichosis, and hyperhidrosis. d. hyperdontia, hypertrichosis, and hyperhidrosis. ANS: A

Hypohidrotic ectodermal dysplasia is characterized by hypodontia (partial anodontia), hypotrichosis (less than normal amount of hair), and hypohidrosis (abnormally diminished secretion of sweat). Some patients die of hyperthermia (greatly increased body temperature) after prolonged exposure to the sun or heavy exercise. Hypohidrotic ectodermal dysplasia is characterized by hypodontia rather than hyperdontia, hypotrichosis, and hypohidrosis. Hypohidrotic ectodermal dysplasia is characterized by hypodontia, hypotrichosis rather than hypertrichosis, and hypohidrosis rather than hyperhidrosis. Hypohidrotic ectodermal dysplasia is characterized by hypodontia rather than hyperdontia, hypotrichosis rather than hypertrichosis, and hypohidrosis rather than hyperhidrosis. REF: Hypohidrotic Ectodermal Dysplasia, page 208

OBJ: 8

47. Frontal bossing, depressed nasal bridge, protuberant lips, and almost complete lack of scalp

hair are characteristic of a person with which condition? a. Hypophosphatemic vitamin D-resistant rickets b. Hypophosphatasia c. Hypohidrotic ectodermal dysplasia d. Dentin dysplasia ANS: C


Frontal bossing, depressed nasal bridge, protuberant lips, and almost complete lack of scalp hair are characteristic of a person with hypohidrotic ectodermal dysplasia. The hair that is present is usually blond, short, fine, and stiff. The skin is soft, thin, and very dry. Sebaceous glands are also lacking. Hypophosphatemic vitamin D-resistant rickets is generally characterized by short stature and bowing of the legs, especially if the condition is present from childhood. Hypophosphatasia is characterized by rachitic-like changes, such as bowing of legs and multiple fractures. Radicular dentin dysplasia is characterized by radiographs that show total or partial lack of pulp chambers and root canals. Coronal dentin dysplasia is characterized by radiographs that show thistle-shaped pulp chambers in single-rooted teeth and a bow tie appearance of the pulp chambers of permanent molars. Permanent teeth may or may not have pulp stones. REF: Hypohidrotic Ectodermal Dysplasia, page 208

OBJ: 8

48. Hypophosphatasia is characterized by a. an increase in serum alkaline phosphatase levels. b. abnormal formation of enamel and dentin. c. exfoliation of teeth without evidence of periodontal or gingival disease. d. an autosomal-dominant inheritance pattern. ANS: C

Hypophosphatasia is characterized by exfoliation of teeth without evidence of periodontal or gingival disease. The basic defect in this condition is a decrease in serum alkaline phosphatase levels with increased urinary and plasma levels of phosphoethanolamine. Agenesis or abnormal formation of cementum in these patients leads to spontaneous premature shedding of primary teeth, especially mandibular incisors. The basic defect in hypophosphatasia is a decrease in serum alkaline phosphatase levels with increased urinary and plasma levels of phosphoethanolamine. Agenesis or abnormal formation of cementum rather than enamel and dentin in patients with hypophosphatasia leads to spontaneous premature shedding of primary teeth, especially mandibular incisors. The inheritance pattern of hypophosphatasia is autosomal-recessive rather than autosomal-dominant. REF: Hypophosphatasia, page 208

OBJ: 8

49. Characteristic radiographic oral findings of hypophosphatemic vitamin D-resistant rickets

include a. obliterated pulp chambers. b. large pulp chambers with very long pulp horns. c. normal pulp chambers with nonexistent roots. d. large pulp chambers with nonexistent pulp horns. ANS: B


Characteristic radiographic oral findings of hypophosphatemic vitamin D-resistant rickets include large pulp chambers with very long pulp horns. In addition, the dentin exhibits pronounced cracks that extend to the dentinoenamel junction. These cracks induce fracture of the enamel with micro exposure of the pulp and subsequent pulpal infection. Characteristic radiographic oral findings of hypophosphatemic vitamin D-resistant rickets include large pulp chambers with very long pulp horns rather than obliterated pulp chambers. Characteristic radiographic oral findings of hypophosphatemic vitamin D-resistant rickets include large pulp chambers with very long pulp horns rather than normal pulp chambers with nonexistent roots. Characteristic radiographic oral findings of hypophosphatemic vitamin D-resistant rickets include large pulp chambers with very long pulp horns rather than large pulp chambers with nonexistent pulp horns. REF: Hypophosphatemic Vitamin D-resistant Rickets, page 209 OBJ: 8 50. Pegged or absent maxillary lateral incisors a. affect the secondary but not primary dentition. b. have a prevalence of 10% in the white population. c. are accompanied by congenitally lacking premolar teeth in 10% to 20% of the

population. d. are generally an autosomal-recessive trait with variable expressivity. ANS: C

Pegged or absent maxillary lateral incisors are accompanied by congenitally lacking premolar teeth in 10% to 20% of the population. Both the primary and secondary dentition can be affected by pegged or absent maxillary lateral incisors, but mostly the latter. Pegged or absent maxillary lateral incisors are prevalent in 1% to 3% of the white population and in about 7% of the Asian population. Pegged or absent maxillary lateral incisors are generally an autosomal-dominant trait with variable expressivity. REF: Pegged or Absent Maxillary Lateral Incisors, page 209

OBJ: 8

51. Each statement about chromosomes is true except one. Which one is the exception? a. Chromosomes are located in the nucleus of the cell. b. Hereditary units called genes are found on chromosomes. c. Chromosomes can only be seen clearly under a microscope when the nucleus and

cell are dividing. d. Chromosomes contain RNA, which directs the production of amino acids,

polypeptides, and proteins by the cell. ANS: D

Chromosomes contain DNA, which directs the production of amino acids, polypeptides, and proteins by the cell. Chromosomes are located in the nucleus of the cell. They contain the hereditary units called genes that are transmitted from one generation to another. Each cell of the human body, with the exception of mature germ cells, has 46 chromosomes. Half of these chromosomes are derived from the father, and the other half from the mother. Chromosomes can only be seen clearly under a microscope when the nucleus and cell are dividing. At other times the genetic material is dispersed in the nucleus. REF: Chromosomes, page 188

OBJ: 1


52. The gap 1, S, and gap 2 phases are part of what type of cell division? a. First meiosis b. Second meiosis c. Mitosis d. Hypohidrosis ANS: C

After each cell division is completed and before the next division can occur, the cell enters the gap 1 phase, which is followed by the S phase, in which replication of the DNA takes place. The gap 2 phase follows the S phase and ends when mitotic division begins. First meiosis is part of a two-step special cell division and does not include the gap 1, S, and gap 2 phases. Second meiosis is part of a two-step special cell division and does not include the gap 1, S, and gap 2 phases. Hypohidrosis is an abnormally diminished secretion of sweat and is not part of cell division. REF: Mitosis, page 188

OBJ: 2

53. During meiosis, if chromosomes crossing over do not separate and both migrate to the same

cell, a germ cell with an extra chromosome results. What is the term to describe this occurrence? a. Nondisjunction b. Chiasmata c. Translation d. Alleles ANS: A

Nondisjunction occurs when chromosomes that were crossing over do not separate, and both migrate to the same cell. Points of contact between a chromatid of one chromosome and a chromatid of the other chromosome of a pair, where crossing over and exchange of chromosome segments occur, are called chiasmata. During translation, amino acids form a polypeptide, and one or more polypeptides form a protein. Genes that are located at the same level in homologous chromosomes and that dictate the same functions or characteristics are called alleles. REF: First Meiosis, page 190

OBJ: 3

54. Inactivation of the genetic activity of one of the X chromosomes in each cell of a female

embryo during the early period of embryonic development is referred to as a. Barr body. b. nucleotide. c. karyotype. d. the Lyon hypothesis. ANS: D

This inactivation of one of the X chromosomes in a female embryo was postulated by Mary Lyon and is known as the Lyon hypothesis. The Barr body is the condensed chromatin of the inactivated X chromosome, not the concept of inactivation of its genetic activity. The nucleotide is the basic unit of DNA. A karyotype is the photomicrographic representation of a person’s chromosomal constitution arranged according to the Denver classification. REF: Lyon Hypothesis, page 190

OBJ: 4


55. Euploid, polyploid, and aneuploid are all terms relating to a(n) _____ of chromosomes. a. alteration in the structure b. abnormal rearrangement c. alteration in the number d. duplication ANS: C

Euploid, polyploid, and aneuploid are all terms relating to an alteration in the number of chromosomes. Euploid, polyploid, and aneuploid are all terms relating to an alteration in the number of chromosomes and not the structure of chromosomes. Euploid, polyploid, and aneuploid are all terms relating to an alteration in the number of chromosomes and not the rearrangement of chromosomes. Duplication relates to abnormalities in chromosomal structure. REF: Alterations in Number and Structure of Chromosomes, page 193 OBJ: 5 56. Genetic risk is a mathematical estimate of probability governed by chance. A condition having

autosomal-dominant inheritance is transmitted vertically from one generation to the next with males affected more than females. a. Both statements are true. b. Both statements are false. c. The first statement is true; the second is false. d. The first statement is false; the second is true. ANS: C

The first statement is true; the second statement is false. Males and females are equally affected. REF: Autosomal-Dominant Inheritance, page 195

OBJ: 6

57. Genetic heterogeneity can cause amelogenesis imperfecta because of its inheritance pattern

(X-linked, autosomal recessive, or autosomal dominant). a. Both the statement and reason are correct and related. b. Both the statement and reason are correct but not related. c. The statement is correct, but the reason is not. d. The statement is not correct, but the reason is correct. e. Neither the statement nor the reason is correct. ANS: A

Genetic heterogeneity can cause amelogenesis imperfecta because of its inheritance pattern (X-linked, autosomal recessive, or autosomal dominant). Both the statement and reason are correct and related. REF: Genetic Heterogeneity, page 196

OBJ: 7

58. Each statement is true regarding focal palmoplantar and gingival hyperkeratosis except one.

Which one is the exception? a. Marked hyperkeratosis follows the normal contour of the gingiva. b. The free gingiva is highly affected with hyperkeratosis.


c. Oral changes start in childhood and increase with age. d. Oral hyperkeratinization is bandlike and a few millimeters in width. ANS: B

In patients with focal palmoplantar and gingival hyperkeratosis, the free gingiva is not affected. Marked hyperkeratosis follows the normal contour of the gingiva in patients with focal palmoplantar and gingival hyperkeratosis. In patients with focal palmoplantar and gingival hyperkeratosis, oral changes start in childhood and increase with age. Oral hyperkeratinization is bandlike and a few millimeters in width in patients with focal palmoplantar and gingival hyperkeratosis. REF: Focal Palmoplantar and Gingival Hyperkeratosis, page 198 OBJ: 8 59. Which syndrome is characterized by a fusion of the anterior portion of the maxillary gingiva

to the upper lip from canine to canine? a. Osler–Rendu–Parkes Weber syndrome b. Ellis–van Creveld syndrome c. Gorlin syndrome d. Peutz–Jeghers syndrome ANS: B

Ellis–van Creveld syndrome is characterized by a fusion of the anterior portion of the maxillary gingiva to the upper lip from canine to canine. Osler–Rendu–Parkes Weber syndrome, also known as hereditary hemorrhagic telangiectasia, is characterized by multiple capillary dilations of the skin and mucous membranes. Gorlin syndrome, also known as nevoid basal cell carcinoma syndrome, is characterized by multiple cysts of the jaws that are odontogenic keratocysts. Peutz–Jeghers syndrome is characterized by multiple melanotic macular pigmentations of the skin and mucosa, which are associated with gastrointestinal polyposis. REF: Ellis–van Creveld Syndrome, page 199

OBJ: 8

60. Chromosomes line up at the equatorial plate during the _____ stage of mitosis. a. telophase b. metaphase c. prophase d. anaphase ANS: B

Chromosomes line up at the equatorial plate during the metaphase stage of mitosis. During metaphase chromosomes are actually formed by two identical vertical halves, each composed of either left or right, short and long arms, and half of the centromere. The chromatids are in the process of splitting during telophase. During prophase the chromosomes are lining up toward metaphase. The chromatids are in the process of splitting during anaphase. REF: Stages of Mitosis, page 188

OBJ: 2

61. A codon in DNA is formed by a sequence of _____ bases. a. two b. three


c. four d. six ANS: B

A codon in DNA is formed by a sequence of three bases. It encodes an amino acid. A codon in DNA is formed by a sequence of three rather than two bases. A codon in DNA is formed by a sequence of three rather than four bases. A codon in DNA is formed by a sequence of three rather than six bases. REF: Deoxyribonucleic Acid, page 190

OBJ: 1

62. Ribonucleic acid (RNA) contains uracil rather than the base _____ found in DNA. a. adenine b. guanine c. cytosine d. thymine ANS: D

RNA contains uracil rather than the base thymine found in DNA. RNA also differs from DNA in that it is usually single-stranded, and its sugar is a ribose rather than the deoxyribose found in DNA. RNA contains uracil rather than the base thymine found in DNA. It does not replace adenine. RNA contains uracil rather than the base thymine found in DNA. It does not replace guanine. RNA contains uracil rather than the base thymine found in DNA. It does not replace cytosine. REF: Ribonucleic Acid, page 191

OBJ: 1

63. Gingival and periodontal disease has been reported in _____% of individuals with Down

syndrome. a. 10 b. 25 c. 50 d. 90 ANS: D

Gingival and periodontal disease has been reported in 90% of individuals with Down syndrome. Premature loss of teeth is seen frequently. Hypodontia, abnormally shaped teeth, and anomalies in eruption with malposition and crowding of teeth are common findings. Gingival and periodontal disease has been reported in 90% rather than 10% of individuals with Down syndrome. Gingival and periodontal disease has been reported in 90% rather than 25% of individuals with Down syndrome. Gingival and periodontal disease has been reported in 90% rather than 50% of individuals with Down syndrome. REF: Trisomy 21, page 193

OBJ: 5

64. A karyotype containing XXXY indicates _____ syndrome. a. Down b. Klinefelter c. Turner d. cri du chat ANS: B


A karyotype containing XXXY indicates Klinefelter syndrome. Down syndrome is trisomy of chromosome 21. A karyotype of XO represents Turner syndrome. Cri du chat syndrome results from a deletion on the short arm of chromosome 5. REF: Klinefelter Syndrome, page 194

OBJ: 5

65. For autosomal-dominant inheritance, _____ pertains to the degree to which an individual is

affected. a. penetrance b. expressivity c. heterogeneity d. consanguinity ANS: B

For autosomal-dominant inheritance, expressivity pertains to the degree to which an individual is affected. Penetrance refers to the number of individuals affected. The term heterogeneity is used when a condition has more than one inheritance pattern and differences exist in the degree of clinical manifestation for each of the inherited varieties. Consanguinity means a family relationship and is generally used to describe matings or marriages among close relatives, usually including first cousins. REF: Autosomal-Dominant Inheritance, page 195

OBJ: 6

66. For parents who are carriers of the same recessive trait, the chance of giving birth to an

affected child is _____%. a. 0 b. 25 c. 50 d. 100 ANS: B

For parents who are carriers of the same recessive trait, the chance of having an affected child is 25%. The risk of having a homozygotic normal child is 25%, and the chance of having a heterozygotic carrier is 50% for each pregnancy. For parents who are carriers of the same recessive trait, the chance of having an affected child is 25% rather than zero. Each parent must contribute the recessive trait, and this will occur 25% of the time as a mathematic probability. For parents who are carriers of the same recessive trait, the chance of having an affected child is 25% rather than 50%. Each parent must contribute the recessive trait, and this will occur 25% of the time as a mathematic probability. For parents who are carriers of the same recessive trait, the chance of having an affected child is 25% rather than 100%. Each parent must contribute the recessive trait, and this will occur 25% of the time as a mathematic probability. REF: Autosomal-Recessive Inheritance, page 195 67. Blood group _____ is an example of gene codominance. a. O b. A c. B d. AB

OBJ: 6


ANS: D

Blood group AB is an example of gene codominance. A and B are dominant over O; but when A and B are allelic, the result is blood group AB in which both genes are exhibited. Blood group O is recessive. Groups A and B are dominant over O; but when A and B are allelic, the result is blood group AB in which both genes are exhibited. Groups A and B are dominant over O; but when A and B are allelic, the result is blood group AB in which both genes are exhibited. REF: Genetic Heterogeneity, page 196

OBJ: 7

68. __________ is an inherited disorder affecting the gingiva and periodontium, which is

characterized by hyperkeratosis of the palms of the hands and soles of the feet. a. Cyclic neutropenia b. Papillon–Lefèvre syndrome c. Gingival fibromatosis d. Laband syndrome ANS: B

Papillon–Lefèvre syndrome is an inherited disorder affecting the gingiva and periodontium, which is characterized by hyperkeratosis of the palms of the hands and soles of the feet. Cyclic neutropenia is characterized by a cyclic decrease in the number of circulating neutrophilic leukocytes. Gingival fibromatosis is a component of several inherited syndromes. The gingival hypertrophy generally develops early in life, and within a few years the teeth are completely covered. Laband syndrome is characterized by gingival fibromatosis, dysplastic or absent nails, and malformed nose and ears because of soft and pliable cartilage formation, hepatosplenomegaly, and hypoplasia of terminal phalanges of the fingers and toes with a resultant froglike appearance. REF: Papillon–Lefèvre Syndrome, page 197

OBJ: 7

69. A patient with Papillon–Lefèvre syndrome will show a marked gingivoperiodontal

inflammatory condition in the primary teeth at about _____ years of age. a. 0 to 0.5 b. 1.5 to 2 c. 4 to 6 d. 8 to 9 ANS: B

A patient with Papillon–Lefèvre syndrome will show a marked gingivoperiodontal inflammatory condition in the primary teeth at about 1.5 to 2 years of age. It is marked by edema, bleeding, alveolar bone resorption, and mobility of teeth with consequent exfoliation. At about 8 or 9 years of age, the gingivoperiodontal destruction of the permanent dentition is repeated in the same manner as occurred in the primary dentition. All primary teeth are lost before 14 years of age. A patient with Papillon–Lefèvre syndrome will show a marked gingivoperiodontal inflammatory condition in the primary teeth at about 1.5 to 2 years of age rather than 0 to 0.5 years of age. A patient with Papillon–Lefèvre syndrome will show a marked gingivoperiodontal inflammatory condition in the primary teeth at about 1.5 to 2 years of age rather than 4 to 6 years of age. A patient with Papillon–Lefèvre syndrome will show a marked gingivoperiodontal inflammatory condition in the primary teeth at about 1.5 to 2 years of age rather than 8 to 9 years of age.


REF: Papillon–Lefèvre Syndrome, page 197

OBJ: 8

70. A patient with _____ will have a mouth that appears fishlike, with downward sloping of the

lip commissures. a. osteogenesis imperfecta b. mandibulofacial dysostosis c. cherubism d. cleidocranial dysplasia ANS: B

A patient with mandibulofacial dysostosis will have a mouth that appears fishlike, with downward sloping of the lip commissures. The facies shows downward sloping of the palpebral fissures, a hypoplastic nose, hypoplastic malar bones with hypoplasia or absence of the zygomatic process, abnormal and misplaced ears, and a receding chin. The lower eyelids show a cleft of the outer third, with a lack of lashes medial to it. The ears may exhibit tags, which on occasion can also be seen near the angle of the mouth. Osteogenesis imperfecta is characterized by a dentinogenesis imperfecta-like condition. Primary teeth are affected in 80% of patients, whereas permanent teeth are affected in only 35% of these individuals. The crowns, roots, and pulp chambers are generally smaller than normal. Cherubism is characterized by radiographs of the jaws that show a typical “soap-bubble” or multilocular appearance, which usually occupies the ascending ramus of the mandible and extends into the molar and premolar areas. Cleidocranial dysplasia is characterized by a premaxilla that is generally underdeveloped, resulting in pseudoprognathism. These patients have many supernumerary teeth, sometimes even simulating a third dentition. REF: Mandibulofacial Dysostosis, page 200

OBJ: 8

71. _________ is a consistent feature of mandibulofacial dysostosis. a. Hypertrichosis b. Deafness c. Hypertelorism d. Kyphoscoliosis ANS: B

Deafness is a consistent feature of mandibulofacial dysostosis. Hypertrichosis (excessive growth of hair) may be associated with gingival fibromatosis. Hypertelorism (increased distance between the eyes) and kyphoscoliosis (a combination of scoliosis, which is a lateral curving of the spine, and kyphosis, which is an abnormal vertical curvature of the spine) are both associated with nevoid basal cell carcinoma syndrome. Kyphoscoliosis is associated with nevoid basal cell carcinoma syndrome. REF: Mandibulofacial Dysostosis, page 200 72. Cases of facial clefting occur in about 1 in _____ births. a. 80 b. 200 c. 800 d. 2000 ANS: C

OBJ: 8


Cases of facial clefting occur in about 1 in 800 births. The majority of cases of facial clefting are multifactorial in origin. A large number of inherited syndromes can include cleft lip and palate or isolated cleft palate as a component. Cases of facial clefting occur in about 1 in 800 rather than 1 in 80 births. Cases of facial clefting occur in about 1 in 800 rather than 1 in 200 births. Cases of facial clefting occur in about 1 in 800 rather than 1 in 2000 births. REF: Isolated Cleft Palate and Cleft Lip With or Without Cleft Palate, page 202 OBJ: 8 73. White sponge nevus is characterized by a white, corrugated, soft, folding oral mucosa always

affecting _____ mucosa. a. attached b. lingual c. buccal d. tonsillar ANS: C

White sponge nevus is characterized by a white, corrugated, soft, folding oral mucosa always affecting buccal mucosa. The lesions are bilateral in most patients. The whitening is produced by a thick layer of keratin, which at times desquamates and leaves a raw mucosal surface. White sponge nevus is characterized by a white, corrugated, soft, folding oral mucosa always affecting buccal and not attached mucosa. White sponge nevus is characterized by a white, corrugated, soft, folding oral mucosa always affecting buccal and not lingual mucosa. White sponge nevus is characterized by a white, corrugated, soft, folding oral mucosa always affecting buccal and not tonsillar mucosa. REF: White Sponge Nevus, page 205

OBJ: 9

74. The _____ type of amelogenesis imperfecta is characterized by tooth enamel that does not

develop to a normal thickness because of failure of the ameloblasts to lay down enamel matrix properly. a. hypoplastic b. hypocalcified c. hypomaturation d. hypoplastic-hypomaturation ANS: A

The hypoplastic type of amelogenesis imperfecta is characterized by tooth enamel that does not develop to a normal thickness because of failure of the ameloblasts to lay down enamel matrix properly. The hypocalcified type of amelogenesis imperfecta is characterized by poorly calcified enamel of normal thickness. The hypomaturation type of amelogenesis imperfecta is characterized by enamel of mottled appearance but normal thickness. The hypoplastichypomaturation type of amelogenesis imperfecta is characterized by thin enamel that is yellow to brown and pitted. It is associated with taurodontic teeth. REF: Amelogenesis Imperfecta, page 205

OBJ: 11

75. Radicular dentin dysplasia is characterized by teeth with _____ crowns and _____ roots. a. normal; normal b. normal; abnormal c. abnormal; normal


d. abnormal; abnormal ANS: B

Radicular dentin dysplasia is characterized by teeth with normal crowns and abnormal roots. REF: Radicular Dentin Dysplasia, page 206

OBJ: 12

76. __________ is characterized by very large, pyramid-shaped molars with large pulp chambers. a. Amelogenesis imperfecta b. Dentinogenesis imperfecta c. Taurodontism d. Dentin dysplasia ANS: C

Taurodontism is characterized by very large, pyramid-shaped molars with large pulp chambers. The furcation of the roots is displaced apically, and these teeth are classified according to the degree of furcation displacement. It is frequently found in Klinefelter syndrome and is associated with many other syndromes as well. Taurodontism and not amelogenesis imperfecta is characterized by very large, pyramid-shaped molars with large pulp chambers. Taurodontism and not dentinogenesis imperfecta is characterized by very large, pyramid-shaped molars with large pulp chambers. Taurodontism and not dentin dysplasia is characterized by very large, pyramid-shaped molars with large pulp chambers. REF: Taurodontism, page 209

OBJ: 12

77. Which term describes less than the normal amount of hair? a. Hypoplastic b. Hypotrichosis c. Haploid d. Nondisjunction ANS: B

Hypotrichosis is the presence of less than the normal amount of hair. Hypoplastic refers to underdevelopment. Haploid is defined as having a single set of chromosomes. Nondisjunction is a term used in genetics when chromosomes that are crossing over do not separate, and therefore both migrate to the same cell. REF: Vocabulary, page 187

OBJ: 1

78. Self-replication, or the ability to duplicate itself, is achievable by a. DNA. b. amino acids. c. meiosis. d. genetic heterogeneity. ANS: A

DNA has the ability to duplicate itself (self-replication) where it can create exact copies of itself. Amino acids are the main components of protein. Meiosis is the two-step cellular division of the original germ cells. Genetic heterogeneity refers to having more than one inheritance pattern. REF: Chromosomes, page 188

OBJ: 1


79. The constriction present in all chromosomes that joins the short and long arms is termed a. trisomy. b. nondisjunction. c. centromere. d. translocation. ANS: C

Centromere is the constricted portion of the chromosome that divides the short arms from the long arms. Trisomy is a pair of chromosomes with an identical extra chromosome. Nondisjunction is a term used in genetics when chromosomes that are crossing over do not separate, and therefore both migrate to the same cell. Translocation is a portion of a chromosome attached to another chromosome. REF: Vocabulary, page 186

OBJ: 1

80. Oral manifestations seen with a person with Down syndrome include the following except

one. Which is the exception? a. Fissured tongue b. Gingival and periodontal disease c. Hypodontia d. Macrodontia ANS: D

Macrodontia is a rare condition and not typically seen in persons with Down syndrome. Fissured tongue is frequently seen in patients with trisomy 21, or Down syndrome. Gingival and periodontal disease has been reported in 90% of affected persons. Hypodontia is a common oral manifestation. REF: Trisomy 21, page 193

OBJ: 5

81. Cyclic neutropenia is so named because this condition a. has more than one inheritance pattern. b. has environmental factors that participate in its cause. c. displays a decrease in the number of neutrophils that occur in cycles of 21 to 27

days. d. allows persons to carry a gene with a dominant effect without any clinical

manifestations. ANS: C

This condition is characterized by a cyclic decrease in the number of neutrophils, and the cycles usually occur in intervals of 21 to 27 days. The term genetic heterogeneity is used when a condition has more than one inheritance pattern. Multifactorial inheritance is used to describe a hereditary pattern where environmental factors participate in the cause of a condition. Cyclic neutropenia is not related to persons who carry a gene with a dominant effect. REF: Cyclic Neutropenia, page 196

OBJ: 8

82. Persons with cyclic neutropenia display a decrease in the number of which circulating cells? a. Chromosomes


b. Neutrophils c. Phenotypes d. RNA ANS: B

Neutrophils are white blood cells that arrive at the site of injury; a decrease in the number of circulating neutrophils is termed neutropenia. Chromosomes are structures located in the nucleus of cells. Phenotypes refer to the entire physical, biochemical, and physiologic makeup of an individual. RNA is ribonucleic acid. REF: Cyclic Neutropenia, page 196

OBJ: 8

83. Identify the term that describes a mating or marriage between close relatives: a. Chiasmata b. Consanguinity c. Trisomy d. Translocation ANS: B

Consanguinity means a familial relationship and is generally used to describe matings or marriages between close relatives, such as first cousins. Chiasmata is the intercrossing of chromatids of the same or homologous chromosomes that takes place at metaphase of first meiosis for the purpose of genetic recombination. Trisomy is a pair of chromosomes with an identical extra chromosome. Translocation is a portion of the chromosome attached to another chromosome. REF: Autosomal-Recessive Inheritance, page 195

OBJ: 1

84. A patient with Papillon–Lefèvre syndrome displays severe periodontal disease until the last

tooth has been exfoliated. The skin manifestations of hyperkeratosis remain present for the life of the patient. a. Both statements are true. b. Both statements are false. c. The first statement is true; the second statement is false. d. The first statement is false; the second statement is true. ANS: A

Both statements are true. The gingival and periodontal condition of these patients regains a normal appearance once the last tooth has been lost. The skin manifestations of hyperkeratinization of the hands and feet remain for life. REF: Papillon–Lefèvre Syndrome, page 197

OBJ: 8

85. What age are all of the permanent teeth typically lost in a patient with Papillon–Lefèvre

syndrome? a. 10 b. 14 c. 22 d. 28 ANS: B

All permanent teeth are lost before 14 years of age with Papillon–Lefèvre syndrome.


REF: Papillon–Lefèvre Syndrome, page 197

OBJ: 8

86. Between the ages of and 4, what is the first clinical sign of cherubism? a. Cleft palate b. Bilateral facial swelling c. Fissured tongue d. Severe mental handicap ANS: B

The first clinical manifestation is a progressive bilateral facial swelling that appears when the patient is between the ages of and 4 years. A large number of inherited syndromes can include cleft palate, but cherubism is not one of these syndromes. Fissured tongue is not associated with cherubism. Severe mental handicap is not associated with cherubism. REF: Cherubism, page 198

OBJ: 8

87. Multiple osteomas and odontomas seen with Gardner syndrome can cause which oral

complication? a. Teeth fail to erupt. b. Teeth demonstrate increased mobility. c. Increased supernumerary teeth d. Teeth exfoliate prematurely. ANS: A

Osteomas are present in various bones with Gardner syndrome, especially the frontal bones, maxilla, and mandible. Multiple odontomas also occur in the jaws, and teeth may exhibit hypercementosis and fail to erupt. Mobility is not an oral manifestation associated with Gardner syndrome. Increased supernumerary teeth may occur in a patient with cleidocranial dysplasia. Teeth do not exfoliate prematurely in a patient with Gardner syndrome. REF: Gardner Syndrome, page 200

OBJ: 8

88. Where is the most common intraoral site for the development of neurofibromas in a patient

with von Recklinghausen disease? a. Floor of the mouth b. Labial commissures c. Palate d. Lateral borders of the tongue ANS: D

Tumors may arise at any intraoral location, the most frequent being the lateral borders of the tongue. Single or multiple tumors may arise at any location in the oral cavity; the tumors are not seen on the floor of the mouth. It is rare to see neurofibromas at the labial commissures. Neurofibromas rarely develop along the palatal tissues. REF: Neurofibromatosis of von Recklinghausen, page 204

OBJ: 8

89. In addition to the development of the neurofibromas, what is the prominent clinical

manifestation seen with a patient with von Recklinghausen disease? a. Café au lait pigmentation of the skin


b. Spider-like telangiectases on the lips, eyelids, and nose c. Abnormally formed bones that fracture easily d. Blue sclera ANS: A

Café au lait pigmentation of the skin is present from the first decade of life in 90% of patients with neurofibromatosis. Spider-like telangiectases are clinically evident in a patient with hereditary hemorrhagic telangiectasia. Abnormally formed bones that fracture easily are a clinical manifestation of a patient with osteogenesis imperfecta. Blue sclerae is also seen in patients with osteogenesis imperfecta. REF: Neurofibromatosis of von Recklinghausen, page 204

OBJ: 8

90. Your patient presents with hypodontia, hypohidrosis, and nearly complete lack of sweat and

sebaceous glands. Also noticeable is a lack of eyelashes and eyebrows. Which hereditary condition is present? a. Amelogenesis imperfecta b. Osteogenesis imperfecta c. Hypohidrotic ectodermal dysplasia d. Dentin dysplasia ANS: C

The major components of hypohidrotic ectodermal dysplasia include: hypodontia, hypotrichosis, hypohidrosis, lacking of sweat and sebaceous glands, thin hair, soft skin, and a lack of eyelashes and eyebrows. Amelogenesis imperfecta are a group of inherited conditions affecting the enamel of teeth and having no associated systemic defects. The basic defect of osteogenesis imperfecta is produced by poor collagen, which results in abnormally formed bones that fracture easily. Dentin dysplasia typically displays teeth with short roots, lack of pulp chambers, and multiple periapical radiolucencies. REF: Hypohidrotic Ectodermal Dysplasia, page 208

OBJ: 8

MATCHING

For each genetic definition below, select the corresponding genetic term from the list provided. a. Autosomes b. Gamete c. Locus d. Mutation e. Phenotype f. Syndrome 1. 2. 3. 4. 5. 6.

Set of signs or symptoms occurring together Position occupied by a gene on a chromosome Nonsex chromosomes that are identical for men and women Permanent change in the arrangement of genetic material The observable appearance of an individual Spermatozoon or ovum


1. 2. 3. 4. 5. 6.

ANS: F ANS: C ANS: A ANS: D ANS: E ANS: B

REF: REF: REF: REF: REF: REF:

Vocabulary, page 187 Vocabulary, page 187 Vocabulary, page 186 Vocabulary, page 187 Vocabulary, page 187 Vocabulary, page 187

OBJ: OBJ: OBJ: OBJ: OBJ: OBJ:

1 1 1 1 1 1

For each clinical syndrome below, select the corresponding chromosomal abnormality from the list provided. a. Characterized by multiple abnormalities in various organs b. Down syndrome as a result of nondisjunction c. Nondisjunction of the X chromosome in the paternal gamete d. Deletion on the short arm of chromosome 5 e. Fertilized ovum has two X chromosomes plus a Y chromosome 7. 8. 9. 10. 11.

Cri du chat Trisomy 13 Klinefelter Turner Trisomy 21

7. ANS: D REF: Clinical Syndromes Resulting From Gross Chromosomal Abnormalities, page 195 OBJ: 5 8. ANS: A REF: Clinical Syndromes Resulting From Gross Chromosomal Abnormalities, page 194 OBJ: 5 9. ANS: E REF: Clinical Syndromes Resulting From Gross Chromosomal Abnormalities, page 194 OBJ: 5 10. ANS: C REF: Clinical Syndromes Resulting From Gross Chromosomal Abnormalities, page 194 OBJ: 5 11. ANS: B REF: Clinical Syndromes Resulting From Gross Chromosomal Abnormalities, page 193 OBJ: 5


Chapter 07: Neoplasia Ibsen: Oral Pathology for the Dental Hygienist, 7th Edition MULTIPLE CHOICE 1. Which choice represents a characteristic of malignant tumors? a. Encapsulated b. Ability to spread to distant sites c. Benign d. Cells resemble normal cells ANS: B

Malignant tumors have the ability to spread to distant sites. Cancer is synonymous with malignancy. Encapsulated is a characteristic of benign rather than malignant tumors. It means the tumor is walled off by surrounding fibrous connective tissue. Sometimes a benign tumor can invade adjacent structures, but it does not have the ability to spread to distant sites. The cells of benign tumors almost always resemble normal cells, whereas malignant tumors vary in their histologic appearance. REF: Classification of Tumors, page 219

OBJ: 2

2. Anaplastic means the tumor a. is composed of cells that vary in size and shape. b. has cells with darker nuclei than those of normal cells. c. has cells that exhibit an increased nuclear-to-cytoplasmic ratio. d. does not resemble the tissue from which it was derived. ANS: D

Anaplastic means the tumor does not resemble the tissue from which it was derived. Pleomorphic means the tumor is composed of cells that vary in size and shape. The nuclei of malignant tumor cells are often hyperchromatic; that is, they are darker than those of normal cells. The nuclei of malignant tumor cells exhibit an increased nuclear-to-cytoplasmic ratio, but anaplastic means the tumor does not resemble the tissue from which it was derived. REF: Classification of Tumors, page 220

OBJ: 1

3. A benign tumor of bone is called a(n) a. osteoma. b. osteogenic sarcoma. c. chondroma. d. carcinoma. ANS: A

A benign tumor of bone is called an osteoma. The prefix of the name of a tumor is determined by the tissue or cell of origin. The suffix -oma is used to indicate a tumor. Malignant tumors of epithelium are called carcinomas, and malignant tumors of connective tissue are called sarcomas. Thus a malignant tumor of bone is an osteogenic sarcoma. A chondroma is a benign tumor of cartilage. A carcinoma is a malignant tumor of epithelium. REF: Name of Tumors, page 220

OBJ: 5


4. A papilloma is a _____ tumor derived from _____ epithelium. a. benign; salivary gland b. malignant; squamous c. benign; squamous d. malignant; odontogenic ANS: C

A papilloma is a benign tumor derived from squamous epithelium. Three different types of epithelial tumors occur in the oral cavity: tumors derived from squamous epithelium, tumors derived from salivary gland epithelium, and tumors derived from odontogenic epithelium. A papilloma is a benign tumor derived from squamous rather than salivary gland epithelium. An adenoma is a benign tumor derived from salivary gland epithelium. A papilloma is a benign rather than malignant tumor derived from squamous epithelium. A squamous cell carcinoma is a malignant tumor derived from squamous epithelium. A papilloma is a benign tumor derived from squamous epithelium rather than a malignant tumor derived from odontogenic epithelium. REF: Papilloma, page 221

OBJ: 5

5. Where do most cases of papilloma occur? a. Hard palate b. Buccal mucosa c. Soft palate or tongue d. Retromolar area ANS: C

Most cases of papilloma occur on the soft palate or tongue. The papilloma is a benign tumor that presents as a small, exophytic, pedunculated, or sessile growth. These tumors are composed of numerous papillary projections that may either be white or the color of normal mucosa. The more keratin, the whiter the lesion appears clinically. Most cases of papilloma occur on the soft palate or tongue rather than the hard palate. Most cases of papilloma occur on the soft palate or tongue rather than the buccal mucosa. Most cases of papilloma occur on the soft palate or tongue rather than the retromolar area. REF: Papilloma, page 221

OBJ: 5

6. Idiopathic leukoplakia a. is a malignant lesion of the oral mucosa. b. can be rubbed off with gauze squares. c. is caused by direct irritation, as in tobacco pouch keratosis. d. does not have a specific known cause. ANS: D

Idiopathic leukoplakia does not have a specific known cause. Idiopathic leukoplakia is considered a premalignant rather than malignant lesion. Leukoplakia cannot be rubbed off and cannot be diagnosed as a specific disease. Most leukoplakias are the result of hyperkeratosis or a combination of epithelial hyperplasia and hyperkeratosis. Idiopathic leukoplakia does not have a specific known cause. REF: Leukoplakia, page 221

OBJ: 3


7. An intraoral mucosal lesion that shows a mixture of red and white areas is termed a. erythroplakia. b. speckled erythroplakia. c. speckled leukoplakia. d. leukoplakia. ANS: C

An intraoral mucosal lesion that shows a mixture of red and white areas is generally called speckled leukoplakia. An intraoral mucosal lesion that shows a mixture of red and white areas is generally called speckled leukoplakia rather than erythroplakia. An intraoral mucosal lesion that shows a mixture of red and white areas is generally called speckled leukoplakia rather than speckled erythroplakia. An intraoral mucosal lesion that shows a mixture of red and white areas is generally called speckled leukoplakia rather than simply leukoplakia. REF: Erythroplakia, page 223

OBJ: 3

8. Which statement about erythroplakia is true? a. Most cases of erythroplakia occur on the buccal mucosa and vestibule. b. Erythroplakia is more common than leukoplakia. c. When examined microscopically, 90% of cases of erythroplakia demonstrate

epithelial dysplasia or squamous cell carcinoma. d. Erythroplakia is considered a less serious clinical finding than leukoplakia. ANS: C

When examined microscopically, 90% of cases of erythroplakia demonstrate epithelial dysplasia or squamous cell carcinoma. Most cases of erythroplakia occur in the floor of the mouth, tongue, and soft palate. Erythroplakia is much less common than leukoplakia. Erythroplakia is considered a more serious clinical finding than leukoplakia. A biopsy must be performed to establish a definitive diagnosis. REF: Erythroplakia, page 223

OBJ: 3

9. Which statement about epithelial dysplasia is true? a. It is considered a malignant condition. b. Unlike squamous cell carcinoma, cellular changes in epithelial dysplasia may

revert to normal if the stimulus, such as tobacco smoking, is removed. c. It presents clinically as an erythematous lesion and not a white lesion

(leukoplakia). d. Lesions often arise on the hard and soft palate. ANS: B

Unlike squamous cell carcinoma, cellular changes in epithelial dysplasia may revert to normal if the stimulus, such as tobacco smoking, is removed. Epithelial dysplasia is considered a premalignant condition. Epithelial dysplasia may present clinically as an erythematous lesion, as a white lesion, or as a mixed erythematous and white lesion. Epithelial dysplasia lesions often arise in the floor of the mouth or tongue. REF: Epithelial Dysplasia, page 223

OBJ: 2

10. Severe epithelial dysplasia involving the full thickness of the epithelium is termed


a. b. c. d.

anaplasia. squamous cell carcinoma. carcinoma in situ. metastasis.

ANS: C

Severe epithelial dysplasia involving the full thickness of the epithelium is called carcinoma in situ. Anaplasia is characterized by a loss of differentiation of cells and their orientation to one another. Squamous cell carcinoma is a malignant tumor of squamous epithelium. Metastasis means that neoplastic cells are transported to parts of the body remote from the primary tumor and that new tumors are established at those sites. REF: Epithelial Dysplasia, page 223

OBJ: 2

11. Squamous cell carcinoma of the oral epithelium usually metastasizes first to which site? a. Spleen b. Lungs c. Liver d. Lymph nodes ANS: D

Squamous cell carcinoma of the oral epithelium usually metastasizes first to the lymph nodes of the neck. From there, it proceeds to more distant sites such as the lungs and liver. Squamous cell carcinoma of the oral epithelium usually metastasizes first to the lymph nodes of the neck rather than the spleen. REF: Squamous Cell Carcinoma, page 223

OBJ: 5

12. “Keratin pearls” are a characteristic feature of a. squamous cell carcinoma. b. basal cell carcinoma. c. pleomorphic adenoma. d. monomorphic adenoma. ANS: A

“Keratin pearls” are a characteristic feature of squamous cell carcinoma. Well-differentiated tumors show keratin formation because keratin is a product of squamous epithelium. The neoplastic cells contain large hyperchromatic nuclei and numerous mitotic figures. Some of the mitotic figures appear normal, whereas others are bizarre. “Keratin pearls” are a characteristic feature of squamous cell carcinoma and not basal cell carcinoma. “Keratin pearls” are a characteristic feature of squamous cell carcinoma and not pleomorphic adenoma. “Keratin pearls” are a characteristic feature of squamous cell carcinoma and not monomorphic adenoma. REF: Squamous Cell Carcinoma, page 224

OBJ: 5

13. Squamous cell carcinomas on the vermilion border of the lips and skin of the face are a. associated with lack of exposure to the sun and vitamin D deficiency. b. more common in individuals with dark skin. c. associated with a poorer prognosis than that for squamous cell carcinoma of the

oral mucosa.


d. associated with color changes of the vermilion border from dark pink and uniform

to mottled grayish pink. ANS: D

Squamous cell carcinomas on the vermilion border of the lips and skin of the face are associated with color changes of the vermilion border from dark pink and uniform to mottled grayish-pink. Squamous cell carcinomas are associated with exposure to the sun. Squamous cell carcinomas tend to be more common in individuals with fair skin. Squamous cell carcinomas of the lips and skin are associated with a much better prognosis than that for squamous cell carcinomas of the oral mucosa. REF: Squamous Cell Carcinoma, page 224

OBJ: 5

14. Which choice is not regarded as an initiating factor in the development of oral cancer? a. Cigar, pipe, and cigarette smoking b. Snuff dipping and tobacco chewing c. Consumption of alcohol d. Chronic irritation ANS: D

No evidence exists that chronic irritation is an initiating factor in the development of oral cancer. The most significant risk factor associated with the development of squamous cell carcinoma is tobacco use, including cigar, pipe, and cigarette smoking, snuff dipping, and tobacco chewing. Alcohol consumption appears to add to the risk of oral squamous cell carcinoma, especially in individuals who also use tobacco products. REF: Squamous Cell Carcinoma, page 225

OBJ: 5

15. According to the TNM staging system for oral squamous carcinoma, T2, N1, M0 would mean

tumor _____ cm in diameter, _____ palpable nodes, and _____ metastasis. a. less than 2; ipsilateral; no distant b. 2 to 4; ipsilateral; no distant c. 2 to 4; contralateral; no distant d. 2 to 4; contralateral; distant ANS: B

According to the TNM staging system for oral squamous carcinoma, T2, N1, M0 would mean tumor 2 to 4 cm in diameter, ipsilateral palpable nodes, and no distant metastasis. According to the TNM staging system for oral squamous carcinoma, T1, N1, M0 would mean tumor less than 2 cm in diameter, ipsilateral palpable nodes, and no distant metastasis. According to the TNM staging system for oral squamous carcinoma, T2, N2, M0 would mean tumor 2 to 4 cm in diameter, contralateral palpable nodes, and no distant metastasis. According to the TNM staging system for oral squamous carcinoma, T2, N2, M1 would mean tumor 2 to 4 cm in diameter, contralateral palpable nodes, and distant metastasis. REF: Squamous Cell Carcinoma, Box 7.1, Table 7.3, page 226

OBJ: 5

16. Verrucous carcinoma a. has a better prognosis than other forms of squamous cell carcinomas. b. appears clinically as a quick-growing exophytic tumor with a pebbly white surface. c. is characterized by poorly differentiated epithelium with atypical cells.


d. shows invasion of tumor cells through the basement membrane. ANS: A

Verrucous carcinoma has a better prognosis than other forms of squamous cell carcinomas. Verrucous carcinoma appears clinically as a slow-growing exophytic tumor with a pebbly white and red surface. Verrucous carcinoma is characterized by well-differentiated epithelium, does not contain atypical cells, and exhibits broad-based rete pegs that penetrate deeply into the connective tissue. The epithelial basement membrane is intact in cases of verrucous carcinoma, and the tumor does not show invasion of tumor cells through the basement membrane, as is seen in squamous cell carcinoma. REF: Verrucous Carcinoma, page 226

OBJ: 5

17. Most cases of verrucous carcinoma occur in the _____ years old. a. posterior lateral border of the tongue of women over 40 b. floor of the mouth of women over 70 c. vestibule and buccal mucosa of men over 55 d. bone of the mandible of men over 20 ANS: C

Most cases of verrucous carcinoma occur in the vestibule and buccal mucosa of men over 55 years old. Most cases of verrucous carcinoma occur in the vestibule and buccal mucosa of men over 55 years old rather than on the posterior lateral border of the tongue of women over 40 years old. Most cases of verrucous carcinoma occur in the vestibule and buccal mucosa of men over 55 years old rather than on the floor of the mouth of women over 70 years old. Most cases of verrucous carcinoma occur in the vestibule and buccal mucosa of men over 55 years old rather than in the bone of the mandible of men over 20 years old. REF: Verrucous Carcinoma, page 226

OBJ: 5

18. Basal cell carcinoma is a _____ tumor that _____ in the oral cavity. a. benign; occurs b. malignant; occurs c. benign; does not occur d. malignant; does not occur ANS: D

Basal cell carcinoma is a malignant tumor that does not occur in the oral cavity. It is associated with excessive exposure to the sun. It frequently arises on the skin of the face and appears as a non healing ulcer with characteristic rolled borders. Basal cell carcinoma is a malignant tumor, not a benign tumor, and does not occur in the oral cavity. REF: Basal Cell Carcinoma, page 227

OBJ: 5

19. As a general rule, a patient should be referred to an oral and maxillofacial surgeon or

dermatologist to have a biopsy performed on any non healing ulcer of the skin or lips that has been present for more than a. 24 hours. b. 2 weeks. c. 3 months.


d. 1 year. ANS: B

As a general rule, a patient should be referred to an oral and maxillofacial surgeon or dermatologist to have a biopsy performed on any non healing ulcer of the skin or lips that has been present for more than 10 days. A biopsy should be performed on any non healing ulcer of the skin or lips that has been present for more than 10 days, not 24 hours. A biopsy should be performed on any non healing ulcer of the skin or lips that has been present for more than 10 days, not 3 months. A biopsy should be performed on any non healing ulcer of the skin or lips that has been present for more than 10 days, not 1 year. REF: Basal Cell Carcinoma, page 227

OBJ: 2

20. Intraoral minor salivary gland tumors are most commonly located a. on the labial and buccal mucosa. b. at the junction of the hard and soft palate. c. on the retromolar area. d. on the floor of the mouth. ANS: B

Intraoral minor salivary gland tumors are most commonly located at the junction of the hard and soft palate. They can also occur on the labial and buccal mucosa, the retromolar area, the floor of the mouth, and rarely the tongue. Tumors of minor salivary gland origin are much more common in the upper lip than in the lower lip. REF: Salivary Gland Tumors, page 227

OBJ: 5

21. A benign tumor of salivary gland origin is termed a(n) a. leiomyoma. b. lipoma. c. adenoma. d. papilloma. ANS: C

A benign tumor of salivary gland origin is called an adenoma. Although some malignant salivary gland tumors are called adenocarcinomas, most have more specific names such as adenoid cystic carcinoma and mucoepidermoid carcinoma. A leiomyoma is a benign tumor of smooth muscle. A lipoma is a benign tumor of mature fat cells. A papilloma is a benign tumor of squamous epithelium. REF: Salivary Gland Tumors, page 227

OBJ: 5

22. Which is the most common salivary gland neoplasm? a. Adenoid cystic carcinoma b. Mucoepidermoid carcinoma c. Pleomorphic adenoma d. Monomorphic adenoma ANS: C


The pleomorphic adenoma is the most common salivary gland neoplasm. It accounts for about 90% of all benign salivary gland tumors. The pleomorphic adenoma rather than the adenoid cystic carcinoma is the most common salivary gland neoplasm. The pleomorphic adenoma rather than the mucoepidermoid carcinoma is the most common salivary gland neoplasm. The pleomorphic adenoma rather than the monomorphic adenoma is the most common salivary gland neoplasm. REF: Pleomorphic Adenoma, page 227

OBJ: 5

23. The most common intraoral location for the pleomorphic adenoma is the a. parotid gland. b. submandibular gland. c. palate. d. floor of the mouth. ANS: C

The most common intraoral location for the pleomorphic adenoma is the palate. The most common extraoral location is the parotid gland. However, these tumors can occur wherever salivary gland tissue is present. The most common extraoral location is the parotid gland. The most common intraoral location for the pleomorphic adenoma is the palate not the submandibular gland. The most common intraoral location for the pleomorphic adenoma is the palate not the floor of the mouth. REF: Pleomorphic Adenoma, page 227

OBJ: 5

24. The pleomorphic adenoma a. is a malignant tumor. b. appears clinically as a painful dome-shaped mass. c. occurs most often in children. d. appears as a slowly enlarging, nonulcerated, painless, dome-shaped mass. ANS: D

The pleomorphic adenoma appears as a slowly enlarging, nonulcerated, painless, domeshaped mass. The pleomorphic adenoma is a benign salivary gland tumor, although it has been known to undergo malignant transformation. This occurrence is known as a carcinoma arising in a pleomorphic adenoma. The pleomorphic adenoma appears as a slowly enlarging, nonulcerated, painless, dome-shaped mass. Most pleomorphic adenomas occur in individuals over 40 years of age, and a female predilection has been noted. Pleomorphic adenomas have also been reported in children. REF: Pleomorphic Adenoma, page 227

OBJ: 5

25. The Warthin tumor a. is a unique type of pleomorphic adenoma. b. presents as a painless, soft mass, usually in the sublingual gland. c. often develops bilaterally in adult women. d. contains sheets of lymphocytes that surround the cystic structures. ANS: D


The Warthin tumor contains sheets of lymphocytes that surround the cystic structures. In some cases the lymphoid component demonstrates germinal center formation. This tumor presents as a painless, soft, compressible or fluctuant mass, usually located in the parotid gland. It often develops bilaterally and occurs predominantly in adult men. The Warthin tumor is a type of monomorphic rather than pleomorphic adenoma. The Warthin tumor presents as a painless, soft, compressible or fluctuant mass, usually located in the parotid gland rather than the sublingual gland. REF: Monomorphic Adenoma, page 228 OBJ: 5 26. Which salivary gland tumor has a “Swiss cheese” appearance under the microscope? a. Pleomorphic adenoma b. Monomorphic adenoma c. Adenoid cystic carcinoma d. Mucoepidermoid carcinoma ANS: C

The adenoid cystic carcinoma has a “Swiss cheese” appearance under the microscope. These round and oval islands represent cylinders of tumor. The adenoid cystic carcinoma rather than the pleomorphic adenoma has a “Swiss cheese” appearance under the microscope. The adenoid cystic carcinoma rather than the monomorphic adenoma has a “Swiss cheese” appearance under the microscope. The adenoid cystic carcinoma rather than the mucoepidermoid carcinoma has a “Swiss cheese” appearance under the microscope. REF: Adenoid Cystic Carcinoma, page 229

OBJ: 5

27. The adenoid cystic carcinoma is a. a benign tumor of salivary gland origin. b. well encapsulated and does not infiltrate surrounding tissue. c. most commonly seen intraorally on the palate. d. painless. ANS: C

The adenoid cystic carcinoma is most commonly seen intraorally on the palate. The adenoid cystic carcinoma is a malignant tumor of salivary gland origin that can originate from either major or minor salivary gland tissue. The adenoid cystic carcinoma is unencapsulated and infiltrates surrounding tissue. Pain is often present with the adenoid cystic carcinoma because of the tendency for these tumors to surround nerves. REF: Adenoid Cystic Carcinoma, page 229

OBJ: 5

28. Adenoid cystic carcinoma is commonly seen in which demographic group? a. Men b. Women of childbearing age c. Fifth and sixth decades of life d. Children ANS: C


Adenoid cystic carcinoma is most commonly diagnosed in the fifth and sixth decades of life. It is more common in women than in men. Adenoid cystic carcinoma is more common in women than in men. Adenoid cystic carcinoma is most commonly diagnosed in the fifth and sixth decades of life, after childbearing age. Adenoid cystic carcinoma is most commonly diagnosed in the fifth and sixth decades of life, not in children. REF: Adenoid Cystic Carcinoma, page 229

OBJ: 5

29. Which tumor is an unencapsulated, infiltrating tumor composed of a combination of mucous

cells interspersed with squamous-like epithelial cells? a. Adenoid cystic carcinoma b. Mucoepidermoid carcinoma c. Monomorphic adenoma d. Pleomorphic adenoma ANS: B

The mucoepidermoid carcinoma is an unencapsulated, infiltrating tumor composed of a combination of mucous cells interspersed with squamous-like epithelial cells called epidermoid cells. Adenoid cystic carcinoma is composed of small, deeply staining, uniform epithelial cells arranged in perforated round to oval islands. Monomorphic adenoma is composed of a uniform pattern of epithelial cells. These tumors do not have the connective tissue–like component seen in a pleomorphic adenoma. Pleomorphic adenoma is an unencapsulated tumor composed of tissue that appears to be a mixture of both epithelium and connective tissue. REF: Mucoepidermoid Carcinoma, page 230

OBJ: 5

30. A central mucoepidermoid carcinoma is derived from either salivary gland tissue trapped

within bone or a. a periapical abscess surrounding the apex of a nonvital tooth. b. the transformed epithelial lining of a dentigerous cyst. c. a lateral periodontal cyst. d. an incisive canal cyst. ANS: B

A central mucoepidermoid carcinoma is derived from either salivary gland tissue trapped within bone or the transformed epithelial lining of a dentigerous cyst. A central mucoepidermoid carcinoma is derived from either salivary gland tissue trapped within bone or the transformed epithelial lining of a dentigerous cyst rather than from a periapical abscess surrounding the apex of a nonvital tooth. A central mucoepidermoid carcinoma is derived from either salivary gland tissue trapped within bone or the transformed epithelial lining of a dentigerous cyst rather than from a lateral periodontal cyst. A central mucoepidermoid carcinoma is derived from either salivary gland tissue trapped within bone or the transformed epithelial lining of a dentigerous cyst rather than from an incisive canal cyst. REF: Mucoepidermoid Carcinoma, page 230

OBJ: 5

31. Which is the most common malignant salivary gland tumor found in children? a. Adenoid cystic carcinoma b. Mucoepidermoid carcinoma c. Monomorphic adenoma


d. Pleomorphic adenoma ANS: B

The mucoepidermoid carcinoma is the most common malignant salivary gland tumor in children. The adenoid cystic carcinoma is also a malignant tumor; however, the mucoepidermoid carcinoma is the most common malignant salivary gland tumor in children. The mucoepidermoid carcinoma is the most common malignant salivary gland tumor in children. Both the monomorphic adenoma and pleomorphic adenoma are benign tumors, although pleomorphic adenomas have been known to undergo malignant transformation. The mucoepidermoid carcinoma is the most common malignant salivary gland tumor in children. REF: Mucoepidermoid Carcinoma, page 230

OBJ: 5

32. The ameloblastoma a. occurs exclusively in the mandible. b. is a benign, encapsulated tumor. c. is commonly known as a Pindborg tumor. d. may cause expansion of bone. ANS: D

The ameloblastoma may cause expansion of bone. It is a benign, slow-growing but locally aggressive epithelial odontogenic tumor that may arise in either the maxilla or mandible. The ameloblastoma may arise in either the maxilla or mandible. The ameloblastoma is a benign, unencapsulated tumor that infiltrates into surrounding tissue and can cause extensive destruction. The calcifying epithelial odontogenic tumor, rather than the ameloblastoma, is also known as a Pindborg tumor. REF: Ameloblastoma, page 231

OBJ: 4

33. Which tumor most closely resembles an ameloblastoma with ameloblast-like epithelial cells

surrounding areas that look like stellate reticulum? a. Calcifying epithelial odontogenic tumor b. Adenomatoid odontogenic tumor c. Calcifying odontogenic cyst d. Odontogenic myxoma ANS: C

The epithelium of the calcifying odontogenic cyst resembles that seen in an ameloblastoma, consisting of ameloblast-like cells and stellate reticulum-like areas. The epithelium of the calcifying odontogenic cyst, rather than the calcifying epithelial odontogenic tumor, resembles that seen in an ameloblastoma, consisting of ameloblast-like cells and stellate reticulum-like areas. The epithelium of the calcifying odontogenic cyst, rather than the adenomatoid odontogenic tumor, resembles that seen in an ameloblastoma, consisting of ameloblast-like cells and stellate reticulum-like areas. The epithelium of the calcifying odontogenic cyst, rather than the odontogenic myxoma, resembles that seen in an ameloblastoma, consisting of ameloblast-like cells and stellate reticulum-like areas. REF: Calcifying Odontogenic Cyst, page 233 34. Most ameloblastomas are located in the a. anterior maxilla.

OBJ: 4


b. posterior maxilla. c. anterior mandible. d. posterior mandible. ANS: D

Most ameloblastomas are located in the posterior mandible. Eighty percent of all ameloblastomas arise in the mandible, most often in the molar or ramus area. Most ameloblastomas are located in the posterior mandible rather than the anterior maxilla. Most ameloblastomas are located in the posterior mandible rather than the posterior maxilla. Most ameloblastomas are located in the posterior mandible rather than the anterior mandible. REF: Ameloblastoma, page 231

OBJ: 4

35. Which tumor occurs most frequently in the anterior part of the jaws? a. Ameloblastoma b. Calcifying epithelial odontogenic tumor c. Adenomatoid odontogenic tumor d. Odontogenic myxoma ANS: C

Seventy percent of adenomatoid odontogenic tumors involve the anterior part of the jaws. In contrast, 80% of ameloblastomas arise in the mandible, most often in the molar and ramus area. Although the calcifying epithelial odontogenic tumor can occur anywhere in the maxilla or mandible, the bicuspid and molar areas are the most common locations. Most cases of odontogenic myxoma occur in the posterior mandible. REF: Adenomatoid Odontogenic Tumor, page 231

OBJ: 5

36. How does an adenomatoid odontogenic tumor differ from a dentigerous cyst on radiographic

images? a. It will not be associated with an impacted tooth. b. It extends beyond the cementoenamel junction and can involve 50% to 60% of the root. c. It will not form calcifications within the tumor, as does a dentigerous cyst. d. It will not form a well-circumscribed radiolucency. ANS: B

An adenomatoid odontogenic tumor will extend beyond the cementoenamel junction and can involve 50% to 60% of the root. Because of its frequent association with an impacted tooth, an adenomatoid odontogenic tumor often simulates a dentigerous cyst. Calcifications do form within the adenomatoid odontogenic tumor, and radiopaque areas of varying size will be visible on radiographs. An adenomatoid odontogenic tumor appears as a well-circumscribed radiolucency. REF: Adenomatoid Odontogenic Tumor, page 232

OBJ: 5

37. The adenomatoid odontogenic tumor features ductlike structures that a. resemble those found in major salivary glands. b. resemble those found in minor salivary glands. c. are actually ameloblast-like cells that resemble ducts because of their circular

arrangement.


d. are actually odontoblast-like cells that resemble ducts because of their circular

arrangement. ANS: C

The adenomatoid odontogenic tumor features ductlike structures that are actually ameloblastlike cells that resemble ducts because of their circular arrangement. Eosinophilic material is seen in the center of these structures, and calcifications also form in this tumor. REF: Adenomatoid Odontogenic Tumor, page 232

OBJ: 5

38. Ghost cells are characteristic of which lesion? a. Calcifying odontogenic cyst b. Adenomatoid odontogenic tumor c. Calcifying epithelial odontogenic tumor d. Ameloblastoma ANS: A

Ghost cells are characteristic of the calcifying odontogenic cyst. These cells exhibit a clear central area and are thought to represent degenerating epithelial cells. Ghost cells are characteristic of the calcifying odontogenic cyst and not an adenomatoid odontogenic tumor. Ghost cells are characteristic of the calcifying odontogenic cyst and not a calcifying epithelial odontogenic tumor. Ghost cells are characteristic of the calcifying odontogenic cyst and not an ameloblastoma. REF: Calcifying Odontogenic Cyst, page 232

OBJ: 5

39. Which statement is true of the odontogenic myxoma? a. It occurs most often in people over 50. b. It exhibits a unilocular radiolucency with well-defined margins. c. The tumor may become quite large and cause displacement of teeth. d. Most cases arise in the posterior maxilla. ANS: C

The odontogenic myxoma may become quite large and cause displacement of teeth. The odontogenic myxoma occurs most often in young individuals between 10 and 29 years of age. No sex predilection is noted. The odontogenic myxoma exhibits a multilocular, honeycombed radiolucency with poorly defined margins. Most cases of odontogenic myxoma arise in the posterior mandible. REF: Odontogenic Myxoma, page 234

OBJ: 5

40. The ameloblastic fibroma a. occurs most often in adults over 50. b. occurs more commonly in males than females. c. is most commonly located in the mandibular anterior region. d. is most commonly located in the maxillary anterior region. ANS: B


The ameloblastic fibroma occurs more commonly in males than in females. The most common location for an ameloblastic fibroma is the mandibular bicuspid and molar region, not the mandibular anterior region. Most cases of ameloblastic fibroma occur in individuals younger than 20 years of age rather than in adults over 50. The most common location for an ameloblastic fibroma is the mandibular bicuspid and molar region, not the maxillary anterior region. REF: Ameloblastic Fibroma, page 236

OBJ: 5

41. A compound odontoma a. consists of a mass of enamel, dentin, cementum, and pulp that does not resemble a

normal tooth. b. usually occurs in the posterior mandible. c. appears on radiographs as a cluster of miniature teeth surrounded by a radiolucent halo. d. appears as a radiolucent mass surrounded by a thin radiopaque halo. ANS: C

A compound odontoma appears on radiographs as a cluster of miniature teeth surrounded by a radiolucent halo. A complex odontoma consists of a mass of enamel, dentin, cementum, and pulp that does not resemble a normal tooth. The compound odontoma is usually located in the anterior maxilla, and the complex odontoma most commonly occurs in the posterior mandible. A complex odontoma appears as a radiopaque mass surrounded by a thin radiolucent halo. REF: Odontoma, page 236

OBJ: 5

42. The central cemento ossifying fibroma is most likely derived from which cells? a. Ameloblasts b. Odontoblasts c. Osteoblasts d. Cells of the periodontal ligament ANS: D

The peripheral ossifying fibroma is most likely derived from cells of the periodontal ligament, not from ameloblasts. The peripheral ossifying fibroma is most likely derived from cells of the periodontal ligament, not from odontoblasts. The peripheral ossifying fibroma is most likely derived from cells of the periodontal ligament, not from osteoblasts. The peripheral ossifying fibroma is most likely derived from cells of the periodontal ligament. It is more common in females than in males and often occurs in young individuals. REF: Central Cementifying Fibroma and Central Ossifying Fibroma, page 234 OBJ: 5 43. A lipoma a. is a tumor of peripheral nerve tissue. b. appears clinically as a yellowish mass surfaced by a thin layer of epithelium. c. is found most commonly intraorally on the tongue. d. occurs most often in men over 60. ANS: B


A lipoma appears clinically as a yellowish mass surfaced by a thin layer of epithelium. A lipoma is a benign tumor of mature fat cells. The most common intraoral locations for lipomas are the buccal mucosa and the vestibule. The majority of lipomas occur in individuals over 40 years of age, and no sex predilection is noted. REF: Lipoma, page 237

OBJ: 5

44. The most common intraoral location for a neurofibroma is the a. floor of the mouth. b. vestibule. c. tongue. d. tonsillar pillar. ANS: C

The most common intraoral location for a neurofibroma is the tongue. Microscopic examination of a neurofibroma reveals a fairly well-delineated but unencapsulated proliferation of spindle-shaped Schwann cells and perineural fibroblasts. The most common intraoral location for a neurofibroma is the tongue and not the floor of the mouth. The most common intraoral location for a neurofibroma is the tongue and not the vestibule. The most common intraoral location for a neurofibroma is the tongue and not the tonsillar pillar. REF: Neurofibroma and Schwannoma, page 237

OBJ: 5

45. The granular cell tumor a. most often occurs on the tongue followed by the buccal mucosa. b. appears as a painful, ulcerated nodule. c. occurs most commonly in children. d. occurs most often in men. ANS: A

The granular cell tumor most often occurs on the tongue followed by the buccal mucosa. It most likely arises from a neural or primitive mesenchymal cell. The granular cell tumor appears as a painless, nonulcerated nodule. The majority of cases of granular cell tumor occur in adults. A female sex predilection is noted for the granular cell tumor. REF: Granular Cell Tumor, page 238

OBJ: 5

46. A rhabdomyosarcoma is a _____ tumor of _____ muscle. a. benign; smooth b. malignant; smooth c. benign; striated d. malignant; striated ANS: D

A rhabdomyosarcoma is a malignant tumor of striated muscle. It is the most common malignant soft tissue tumor of the head and neck in children. A rhabdomyosarcoma is a malignant tumor of striated muscle, not a benign tumor of smooth muscle. A leiomyoma is a benign tumor of smooth muscle. A rhabdomyosarcoma is a malignant tumor of striated muscle, not a malignant tumor of smooth muscle. A leiomyosarcoma is a malignant tumor of smooth muscle. A rhabdomyosarcoma is a malignant tumor of striated muscle, not a benign tumor of striated muscle. A rhabdomyoma is a benign tumor of striated muscle.


REF: Tumors of Muscles, page 239

OBJ: 5

47. Which statement about hemangiomas is true? a. The gingiva is the most common intraoral location. b. They are more common in boys than in girls. c. They appear as variably sized, deep red or blue lesions that do not blanch when

pressure is applied. d. They may occur in adults as a response to trauma and represent an abnormal

proliferation of blood vessels during the healing process. ANS: D

Hemangiomas may occur in adults as a response to trauma and represent an abnormal proliferation of blood vessels during the healing process. The tongue is the most common intraoral location for a hemangioma. Hemangiomas are more common in girls than in boys. Hemangiomas appear as variably sized, deep red or blue lesions that frequently blanch when pressure is applied. REF: Vascular Tumors, page 239

OBJ: 5

48. Kaposi sarcoma is a. caused by a human herpesvirus. b. seen only in association with HIV infection. c. most often seen intraorally on the tongue. d. a malignant striated muscle tumor. ANS: A

Kaposi sarcoma is caused by a human herpesvirus (HHV) that is called both human herpesvirus type 8 (HHV-8) and Kaposi sarcoma–associated herpesvirus (KSAH). Kaposi sarcoma may occur in patients with other forms of immunodeficiency in addition to HIV, specifically patients who have received immunosuppressive drug therapy as a result of organ transplantation. The hard palate and gingiva are the most common intraoral sites of Kaposi sarcoma. Kaposi sarcoma is a malignant vascular tumor. REF: Malignant Vascular Tumors, page 240

OBJ: 5

49. Malignant melanoma usually presents as a(n) a. slowly enlarging white mass. b. rapidly enlarging blue-to-black mass. c. slowly enlarging papule. d. exophytic, wartlike lesion. ANS: B

Malignant melanoma usually presents as a rapidly enlarging blue-to-black mass. It is a very aggressive tumor that exhibits unpredictable behavior and early metastasis. Malignant melanoma usually presents as a rapidly enlarging blue-to-black mass rather than as a slowly enlarging white mass. Malignant melanoma usually presents as a rapidly enlarging blue-toblack mass rather than as a slowly enlarging papule. Malignant melanoma usually presents as a rapidly enlarging blue-to-black mass rather than as an exophytic, wartlike lesion. REF: Melanoma, page 241

OBJ: 5


50. Which statement about malignant melanoma is true? a. It has a benign counterpart known as the benign melanoma. b. Most malignant melanomas arise on the skin as a result of prolonged exposure to

chemicals such as benzene. c. Primary malignant melanoma of the oral cavity is rare. d. The most common intraoral location is the tongue. ANS: C

Primary malignant melanoma of the oral cavity is rare. All melanomas are malignant; there is no benign counterpart. Most malignant melanomas arise on the skin as a result of prolonged exposure to sunlight. The most common intraoral locations are the palate and maxillary gingiva. REF: Melanoma, page 241

OBJ: 5

51. A “sunburst” pattern is associated with a. Kaposi sarcoma. b. osteoma. c. osteosarcoma. d. multiple myeloma. ANS: C

A “sunburst” pattern is associated with osteosarcoma. It is a malignant tumor of bone-forming tissue. It is the most common primary malignant tumor of bone in patients under 40 years of age. Patients may experience a diffuse swelling or mass that is often painful. A “sunburst” pattern is associated with osteosarcoma, not Kaposi sarcoma. A “sunburst” pattern is associated with osteosarcoma, not osteoma. A “sunburst” pattern is associated with osteosarcoma, not multiple myeloma. REF: Osteosarcoma, page 242

OBJ: 5

52. Acute leukemia is characterized by a proliferation of _____ cells. a. immature white blood b. mature white blood c. immature plasma d. mature plasma ANS: A

Acute leukemia is characterized by a proliferation of immature white blood cells. It is most common in children and young adults. Leukemia occurs more often in males than in females. Acute leukemia is characterized by a proliferation of immature white blood cells rather than mature white blood cells. Acute leukemia is characterized by a proliferation of immature white blood cells rather than immature plasma cells. Acute leukemia is characterized by a proliferation of immature white blood cells rather than mature plasma cells. REF: Leukemia, page 242

OBJ: 5

53. The most common location for an intraoral lymphoma is the a. floor of the mouth. b. posterior lateral border of the tongue.


c. tonsillar area. d. vestibule. ANS: C

The most common location for an intraoral lymphoma is the tonsillar area. Rarely, a lymphoma may present as a primary lesion in the oral soft tissues or bone. Most lymphomas involve either lymph nodes or aggregates of lymphoid tissue that are located in the digestive tract from the oral cavity to the anus. The most common location for an intraoral lymphoma is the tonsillar area, not the floor of the mouth. The most common location for an intraoral lymphoma is the tonsillar area, not the posterior lateral border of the tongue. The most common location for an intraoral lymphoma is the tonsillar area, not the vestibule. REF: Lymphoma (Non-Hodgkin Lymphoma), page 242

OBJ: 5

54. Bence Jones proteins are associated with a. osteosarcoma. b. leukemia. c. lymphoma. d. multiple myeloma. ANS: D

Bence Jones proteins are associated with multiple myeloma. Bence Jones proteins are associated with multiple myeloma, not osteosarcoma. Bence Jones proteins are associated with multiple myeloma, not leukemia. Bence Jones proteins are associated with multiple myeloma, not lymphoma. REF: Multiple Myeloma, page 244

OBJ: 5

55. Multiple myeloma is a systemic, malignant proliferation of a. red blood cells. b. plasma cells. c. eosinophils. d. basophils. ANS: B

Multiple myeloma is a systemic, malignant proliferation of plasma cells. It causes destructive lesions in bone. Patients usually experience bone pain and swelling. Pathologic fracture of an involved bone is common and typically occurs in bones weakened as a result of their destruction by the proliferation of neoplastic plasma cells. Multiple myeloma is a systemic, malignant proliferation of plasma cells rather than red blood cells. Multiple myeloma is a systemic, malignant proliferation of plasma cells rather than eosinophils. Multiple myeloma is a systemic, malignant proliferation of plasma cells rather than basophils. REF: Multiple Myeloma, page 242

OBJ: 5

56. Where is the most frequent intraoral site for metastatic tumors of the jaws? a. Nares b. Maxilla c. Mandible d. Zygoma ANS: C


The most frequent intraoral site for metastatic tumors of the jaws is the mandible. The most frequent intraoral site for metastatic tumors of the jaws is the mandible, not the nares. The most frequent intraoral site for metastatic tumors of the jaws is the mandible, not the maxilla. The most frequent intraoral site for metastatic tumors of the jaws is the mandible, not the zygoma. REF: Metastatic Tumors, page 244

OBJ: 5

57. The benign oral pathologic lesion that occurs frequently in the head and neck area and is

classified as capillary or cavernous is termed a. hematoma. b. lipoma. c. adenoma. d. hemangioma. ANS: D

The hemangioma is a benign proliferation of capillaries containing numerous small capillaries called capillary hemangiomas or, if containing larger blood vessels, cavernous hemangiomas. A hematoma is a lesion that results from the accumulation of blood within tissue as a result of trauma. A lipoma is a benign tumor of adipose tissue. An adenoma is a benign tumor of the salivary glands. REF: Vascular Tumors, page 239

OBJ: 5

58. All are common warning signs of malignant neoplasms except one. Which is the exception? a. Leukoplakia b. Bleeding c. Erythroplakia d. Pigmentation ANS: B

Bleeding is not a classic warning sign of malignancy. Leukoplakia, erythroplakia, pigmentation, and a mass are all common warning signs of malignant neoplasms. REF: Melanocytic Nevus, page 241 | Leukoplakia, page 221 | Erythroplakia, page 223 OBJ: 2 59. Which is a common characteristic of benign neoplasms? a. Inability to spread to distant sites b. Nonencapsulated c. Undifferentiated and pleomorphic d. No resemblance to surrounding cells ANS: A

A benign neoplasm does not have the ability to spread to distant sites. A benign neoplasm is more commonly encapsulated than nonencapsulated. Benign neoplasms are commonly well differentiated and not pleomorphic. Benign neoplasms commonly resemble normal and surrounding cells. REF: Classification of Tumors, page 219 60. A common location of the oral neurofibroma is the

OBJ: 2


a. b. c. d.

tongue. palate. gingiva. buccal mucosa.

ANS: A

The tongue is the most common location of the oral neurofibroma. The palate is not a common location of the oral neurofibroma. The gingiva is not a common location of the oral neurofibroma. The buccal mucosa is not a common location of the oral neurofibroma. REF: Neurofibroma and Schwannoma, page 237

OBJ: 5

61. A benign cementoblastoma has all these radiographic characteristics except one. Which is the

exception? a. Well circumscribed b. Radiopaque c. Attached to root d. Multilocular ANS: D

The benign cementoblastoma does not appear as multilocular on a radiograph. The benign cementoblastoma is a cementum-producing lesion that is fused to the root(s) and radiographically appears as a well-defined radiopaque mass with a surrounding radiolucent halo. REF: Cementoblastoma, page 235

OBJ: 5

62. The diagnosis of oral osteosarcoma is dependent on a. clinical examination. b. palpation. c. radiographic interpretation. d. patient symptoms. ANS: C

Osteosarcoma is a malignant tumor of bone-forming tissue and is diagnosed through the use of radiographs. The diagnosis of osteosarcoma is not dependent on clinical examination but is dependent on radiographic interpretation. The diagnosis of osteosarcoma is not dependent on palpation but is dependent on radiographic interpretation. The diagnosis of osteosarcoma is not dependent on patient symptoms but is dependent on radiographic interpretation. REF: Osteosarcoma, page 242

OBJ: 5

63. All carcinomas are malignant tumors that originate from squamous epithelium except one.

Which is the exception? a. Adenoid cystic b. Basal cell c. Verrucous d. Squamous cell ANS: A


Adenoid cystic carcinoma originates from either major or minor salivary gland tissue. Basal cell carcinoma originates from stratified squamous epithelium. Verrucous carcinoma originates from squamous epithelium. Squamous cell carcinoma originates from squamous epithelium. REF: Salivary Gland Tumors, page 227

OBJ: 5

64. The distinction between dangerous and innocuous leukoplakia is best determined by the a. clinical appearance of the lesion. b. identification of predisposing factors. c. patient’s previous experience. d. biopsy of the lesion. ANS: D

The microscopic appearance of leukoplakia varies; therefore, a biopsy is essential to establish a definitive diagnosis between dangerous and innocuous leukoplakia. The clinical appearance of dangerous and innocuous leukoplakia can be identical. Predisposing factors are similar between dangerous and innocuous leukoplakia. A patient’s previous experience has no direct correlation between dangerous and innocuous leukoplakia. REF: Leukoplakia, page 221

OBJ: 3

65. The name of the benign tumor that is slow growing but locally aggressive and derived in

association with an impacted tooth and dentigerous cyst is termed a. calcifying odontogenic cyst. b. odontogenic myxoma. c. compound odontoma. d. ameloblastoma. ANS: D

Ameloblastomas are slow growing but locally aggressive and can be associated with an impacted tooth and dentigerous cyst. Calcifying odontogenic cysts appear radiographically as a well-defined unilocular or multilocular radiolucency with radiopaque calcifications. Odontogenic myxoma is a benign mesenchymal odontogenic tumor that exhibits a multilocular, honeycombed radiolucency with poorly defined margins. Compound odontomas appear radiographically as a cluster of numerous miniature teeth surrounded by a radiolucent halo. REF: Ameloblastoma, page 231

OBJ: 3

66. Epithelial dysplasia is a microscopy-based diagnosis; it indicates disordered growth and is

considered a premalignant condition. a. Both statements are true. b. Both statements are false. c. The first statement is true; the second is false. d. The first statement is false; the second is true. ANS: A

Both statements are true. Epithelial dysplasia is a microscopic diagnosis that indicates disordered growth and is considered a premalignant condition. Both statements are true.


REF: Epithelial Dysplasia, page 223

OBJ: 2

67. The multiple myeloma is a benign proliferation of plasma cells that produce large amounts of

immunoglobulin. Patients usually experience bone pain and swelling. a. Both statements are true. b. Both statements are false. c. The first statement is true; the second is false. d. The first statement is false; the second is true. ANS: D

The first statement is false; the second is true. Multiple myeloma is a malignant proliferation of plasma cells. REF: Multiple Myeloma, pages 242, 244

OBJ: 5

68. This benign tumor of fat cells that clinically appears as a yellowish mass surfaced by a thin

layer of epithelium is referred to as a a. chondroma. b. rhabdomyosarcoma. c. lipoma. d. lymphoma. ANS: C

A lipoma is a benign tumor of fat cells that clinically appears as a yellowish mass surfaced by a thin layer of epithelium. Chondroma is a benign tumor of the cartilage. A rhabdomyosarcoma is a malignant tumor of striated muscle. Lymphoma is a malignant tumor of lymphoid tissue. REF: Lipoma, page 237

OBJ: 5

69. Squamous cell carcinoma of the oral epithelium usually metastasizes first to the lymph nodes

of the neck. Common distant sites include a. kidneys and spleen. b. lungs and liver. c. breast and lungs. d. liver and spleen. ANS: B

Squamous cell carcinoma of the oral epithelium usually metastasizes first to the lymph nodes of the neck and then to more distant sites such as the lungs and liver. Squamous cell carcinoma of the oral epithelium does not usually metastasize to the kidneys and spleen. It usually metastasizes first to the lymph nodes of the neck and then to more distant sites such as the lungs and liver. Squamous cell carcinoma of the oral epithelium does not usually metastasize to the breast and lungs. It usually metastasizes first to the lymph nodes of the neck. From there, it proceeds to more distant sites such as the lungs and liver. Squamous cell carcinoma of the oral epithelium does not usually metastasize to the liver and spleen. It usually metastasizes first to the lymph nodes of the neck. From there, it proceeds to more distant sites such as the lungs and liver. REF: Squamous Cell Carcinoma, page 223

OBJ: 5


70. A benign tumor of cartilage is termed a(n) a. osteoma. b. lipoma. c. chondroma. d. carcinoma. ANS: C

A chondroma is a benign tumor of cartilage. A benign tumor of bone is called an osteoma. A lipoma is a benign tumor of fat cells. A carcinoma is a malignant tumor of epithelium. REF: Tumors of Cartilage, page 242

OBJ: 5

71. According to the TNM staging system for oral squamous carcinoma, T3, N0, M0 would mean

tumor _____ cm in diameter, _____ palpable nodes, and _____ metastasis. a. less than; ipsilateral; no distant b. is 2 to 4; ipsilateral; no distant c. greater than 4; no; no distant d. greater than 4; contralateral; distant ANS: C

According to the TNM staging system for oral squamous carcinoma, T3, N0, M0 would mean the tumor is greater than 4 cm in diameter, no palpable nodes, and no distant metastasis. According to the TNM staging system for oral squamous carcinoma, T1, N1, M0 would mean tumor less than 2 cm in diameter, ipsilateral palpable nodes, and no distant metastasis. According to the TNM staging system for oral squamous carcinoma, T2, N1, M0 would mean tumor 2 to 4 cm in diameter, ipsilateral palpable nodes, and no distant metastasis. According to the TNM staging system for oral squamous carcinoma, T3, N2, M1 would mean the tumor is greater than 4 cm in diameter, contralateral palpable nodes, and distant metastasis. REF: Box 7.1, page 226

OBJ: 5

72. _____ means new growth. a. Metastasis b. Neoplasia c. Hypertrophy d. Hyperplasia ANS: B

Neoplasia means new growth. It is a process in which cells exhibit uncontrolled proliferation. A neoplasm is a mass of such cells. Although the word tumor means swelling, it is commonly used as a synonym for neoplasm. Metastasis is transport of neoplastic cells to parts of the body remote from the primary tumor and the establishment of new tumors at those sites. Hypertrophy means the cells have increased in size. Hyperplasia is an abnormal increase in the number of cells in an organ or tissue. REF: Description of Neoplasia, page 219

OBJ: 1

73. A _____ both invades and destroys surrounding tissue and has the ability to spread throughout

the body. a. benign tumor b. malignant tumor


c. hypertrophy d. hyperplasia ANS: B

A malignant tumor both invades and destroys surrounding tissue and has the ability to spread throughout the body. A benign tumor or neoplasm remains localized. Hypertrophy is an increase in the size of cells in an organ or tissue. Hyperplasia is an increase in the number of cells in an organ or tissue. REF: Classification of Tumors, page 219

OBJ: 1

74. Sarcomas are _____ common than carcinomas. a. 10 times more b. 3 times more c. 3 times less d. 10 times less ANS: D

Sarcomas are about 10 times less common than carcinomas. Sarcomas are malignant tumors of connective tissue, and carcinomas are malignant tumors of connective tissue. Sarcomas are about 10 times less common, rather than 10 times more common, than carcinomas. Sarcomas are about 10 times less common, rather than 3 times more common, than carcinomas. Sarcomas are about 10 times less common, rather than 3 times less common, than carcinomas. REF: Name of Tumors, page 220

OBJ: 2

75. Approximately _____% of leukoplakias examined microscopically demonstrate epithelial

dysplasia. a. 1 to 2 b. 5 to 25 c. 50 to 60 d. 90 ANS: B

Approximately 5% to 25% of leukoplakias examined microscopically demonstrate epithelial dysplasia. When examined microscopically, a leukoplakia may show epithelial dysplasia, a premalignant condition, or even squamous cell carcinoma, a malignant tumor of squamous epithelium. Approximately 5% to 25%, not 1% to 2%, of leukoplakias examined microscopically demonstrate epithelial dysplasia. Approximately 5% to 25%, not 50% to 60%, of leukoplakias examined microscopically demonstrate epithelial dysplasia. Approximately 5% to 25%, not 90%, of leukoplakias examined microscopically demonstrate epithelial dysplasia. REF: Leukoplakia, page 221

OBJ: 3

76. Patients who have undergone radiation therapy for malignant tumors of the head and neck

often experience severe xerostomia as a result of radiation damage to _____ tissue. a. oral mucosa b. striated muscle c. salivary gland d. tooth


ANS: C

Patients who have undergone radiation therapy for malignant tumors of the head and neck often experience severe xerostomia as a result of radiation damage to salivary gland tissue. These patients require preventive dental care consisting of nutritional counseling, application of topical fluoride, and meticulous home care. Patients who have undergone radiation therapy for malignant tumors of the head and neck often experience severe xerostomia as a result of radiation damage to salivary gland tissue rather than oral mucosa tissue. Patients who have undergone radiation therapy for malignant tumors of the head and neck often experience severe xerostomia as a result of radiation damage to salivary gland tissue rather than striated muscle tissue. Patients who have undergone radiation therapy for malignant tumors of the head and neck often experience severe xerostomia as a result of radiation damage to salivary gland tissue rather than tooth tissue. REF: Squamous Cell Carcinoma, Treatment and Prognosis, page 226 OBJ: 5 77. A basal cell carcinoma will _____ metastasize. a. never b. rarely c. often d. always ANS: B

A basal cell carcinoma will rarely metastasize. It is a locally invasive tumor that can become quite large and disfiguring if it is not removed. Surgical excision is the treatment of choice, and radiation therapy may be used to treat large lesions. A basal cell carcinoma will rarely rather than never metastasize. A basal cell carcinoma will rarely rather than often metastasize. A basal cell carcinoma will rarely rather than always metastasize. REF: Basal Cell Carcinoma, page 227

OBJ: 5

78. The _____ is a unique type of monomorphic adenoma. a. verrucous carcinoma b. mucoepidermoid carcinoma c. cylindroma d. papillary cystadenoma lymphomatosum ANS: D

The papillary cystadenoma lymphomatosum is a unique type of monomorphic adenoma. Microscopic examination of this variant demonstrates an encapsulated tumor composed of two types of tissue: epithelial and lymphoid. The epithelial component is neoplastic. The papillary cystadenoma lymphomatosum is a unique type of monomorphic adenoma. Verrucous carcinoma is an epithelial tumor rather than a salivary gland tumor. The papillary cystadenoma lymphomatosum is a unique type of monomorphic adenoma. Mucoepidermoid carcinoma is an epithelial tumor rather than a salivary gland tumor. The papillary cystadenoma lymphomatosum is a unique type of monomorphic adenoma. Cylindroma or adenoid cystic carcinoma is an epithelial tumor rather than a salivary gland tumor. REF: Monomorphic Adenoma, page 228 OBJ: 5


79. When a mucoepidermoid carcinoma arises within bone, it will usually be found in the _____

region. a. maxillary anterior b. maxillary posterior c. mandibular anterior d. mandibular posterior ANS: D

When a mucoepidermoid carcinoma arises within bone, it will usually be in the mandibular posterior region. In this location, it appears either as a unilocular or multilocular radiolucency. When a mucoepidermoid carcinoma arises within bone, it will usually be in the mandibular posterior region rather than the maxillary anterior region. When a mucoepidermoid carcinoma arises within bone, it will usually be in the mandibular posterior region rather than the maxillary posterior region. When a mucoepidermoid carcinoma arises within bone, it will usually be in the mandibular posterior region rather than the mandibular anterior region. REF: Mucoepidermoid Carcinoma, page 230

OBJ: 5

80. An ameloblastoma may appear as a _____ on radiographic images. a. unilocular radiolucency b. multilocular opacity c. unilocular or multilocular radiolucency d. unilocular or multilocular radiopacity ANS: C

An ameloblastoma may appear as a unilocular or multilocular radiolucency on radiographic images. The classic radiographic appearance of an ameloblastoma is a multilocular soapbubble or honeycombed radiolucency. In smaller tumors the radiolucency may be unilocular. REF: Ameloblastoma, page 231

OBJ: 4

81. A(n) _____ is composed of islands and sheets of polyhedral epithelial cells with deposits of

amyloid-like material that is thought to represent a form of abnormal enamel protein. a. adenomatoid odontogenic tumor b. calcifying epithelial odontogenic tumor c. ameloblastoma d. mucoepidermoid carcinoma ANS: B

A calcifying epithelial odontogenic tumor is composed of islands and sheets of polyhedral epithelial cells with deposits of amyloid-like material that is thought to represent a form of abnormal enamel protein. A calcifying epithelial odontogenic tumor, not an adenomatoid odontogenic tumor, is composed of islands and sheets of polyhedral epithelial cells. A calcifying epithelial odontogenic tumor, not an ameloblastoma, is composed of islands and sheets of polyhedral epithelial cells. A calcifying epithelial odontogenic tumor, not a mucoepidermoid carcinoma, is composed of islands and sheets of polyhedral epithelial cells. REF: Calcifying Epithelial Odontogenic Tumor, page 231

OBJ: 5

82. The majority of adenomatoid odontogenic tumors occur in _____ years of age. a. females over 40


b. males over 40 c. females under 20 d. males under 20 ANS: C

The majority (70%) of adenomatoid odontogenic tumors occur in females under 20 years of age. The maxilla is more commonly involved than the mandible. Many adenomatoid odontogenic tumors are associated with impacted teeth. The majority of adenomatoid odontogenic tumors occur in females under 20 years of age rather than females over 40. The majority of adenomatoid odontogenic tumors occur in females under 20 years of age rather than males over 40. The majority of adenomatoid odontogenic tumors occur in females under 20 years of age rather than males under 20. REF: Adenomatoid Odontogenic Tumor, page 231

OBJ: 5

83. A _____ is a benign proliferation of capillaries. a. hematoma b. hemangioma c. hamartoma d. lymphangioma ANS: B

A hemangioma is a benign proliferation of capillaries. It is a common vascular lesion considered by many to represent a developmental lesion rather than a tumor because hemangiomas do not generally exhibit an unlimited growth potential. A hemangioma rather than a hematoma is a benign proliferation of capillaries. A hemangioma rather than a hamartoma is a benign proliferation of capillaries. A hemangioma rather than a lymphangioma is a benign proliferation of capillaries. REF: Vascular Tumors, page 239

OBJ: 5

84. Osteosarcoma occurs more often in the _____ and more often in _____. a. maxilla; males b. maxilla; females c. mandible; males d. mandible; females ANS: C

Osteosarcoma occurs more often in the mandible than the maxilla and more often in males than in females. Osteosarcoma occurs more often in the mandible than the maxilla. Osteosarcoma occurs more often in males than in females. REF: Osteosarcoma, page 242

OBJ: 5

85. Approximately _____% of patients with chondrosarcoma involving the jaws survive 5 years

after the diagnosis. a. 10 b. 30 c. 50 d. 70 ANS: B


Approximately 30% of patients with chondrosarcoma involving the jaws survive 5 years after the diagnosis. Chondrosarcomas are treated with wide surgical excision. Radiation therapy and chemotherapy are not effective. The prognosis is poor. Approximately 30% rather than 10% of patients with chondrosarcoma involving the jaws survive 5 years after the diagnosis. Approximately 30% rather than 50% of patients with chondrosarcoma involving the jaws survive 5 years after the diagnosis. Approximately 30% rather than 70% of patients with chondrosarcoma involving the jaws survive 5 years after the diagnosis. REF: Tumors of Cartilage, page 242

OBJ: 5

86. All terms are associated with malignancy except one. Which is the exception? a. Invasive b. Encapsulated c. Cancer d. Metastasis ANS: B

Encapsulated signifies a growth surrounded by a capsule of fibrous connective tissue. This type of growth better describes a benign lesion. Invasive refers to the infiltration and active destruction of surrounding tissues, as seen with a malignant growth. Another term for cancer is malignancy. Metastasis is the transport of neoplastic cells to parts of the body remote from the primary tumor and the establishment of new tumors in those sites. REF: Classification of Tumors, pages 219-220

OBJ: 1

87. What is the term to describe a malignant tumor in bone? a. Osteosarcoma b. Osteoma c. Lipoma d. Carcinoma ANS: A

An osteosarcoma is a malignant tumor of bone. An osteoma is a benign tumor of bone. A lipoma is a benign tumor of fat. A carcinoma is a malignant tumor of epithelium. REF: Name of Tumors, page 220

OBJ: 2

88. Which lesion most closely resembles a papilloma? a. Benign mixed tumor b. Leukoplakia c. Epithelial dysplasia d. Verruca vulgaris ANS: D

The papilloma presents as a small, exophytic, pedunculated growth composed of numerous papillary projections, similar to verruca vulgaris (common wart). A benign mixed tumor is a benign salivary gland tumor. Leukoplakia is a white plaque-like lesion that cannot be rubbed off and cannot be diagnosed clinically as a specific disease. Epithelial dysplasia may present as a red, white, or mixed red and white lesion. REF: Tumors of Squamous Epithelium, page 221

OBJ: 6


89. Leukoplakia cannot be diagnosed clinically as a specific disease; therefore, it must often be

examined in a laboratory setting. When examined microscopically, leukoplakia may reveal the following except one. Which is the exception? a. Squamous cell carcinoma b. Hemangioma c. Epithelial dysplasia d. Premalignant condition ANS: B

A hemangioma is a benign proliferation of capillaries. Early tumors of squamous cell carcinoma may reveal a leukoplakia appearance. When examined microscopically, a leukoplakia may show epithelial dysplasia in approximately 5% to 25% of cases. The laboratory results of a leukoplakia biopsy may also reveal a premalignant condition. REF: Leukoplakia, page 221

OBJ: 6

90. Surgery, chemotherapy, radiation therapy, or a combination of all three may be used to treat

which lesion? a. Papilloma b. Fibroma c. Adenoma d. rhabdomyosarcoma ANS: D

The rhabdomyosarcoma is an aggressive malignant tumor that is best treated by a combination of multidrug chemotherapy, radiation therapy, and surgery. REF: p. 239

OBJ: 6

91. The papilloma will appear whiter clinically because of additional amounts of what substance? a. Keratin b. Exudate c. Granulation tissue d. Macrophages ANS: A

The color of the papilloma depends on the amount of surface keratin. The more keratin, the whiter the lesion will appear. Exudate is fluid with a high protein content and consists of white blood cells, fibrin and other protein molecules. Granulation tissue is the initial connective tissue formed in healing. The macrophage is the second type of white blood cell to arrive at the site of injury. REF: Tumors of Squamous Epithelium, page 221

OBJ: 6

92. Which intraoral lesion appears as a smooth red patch or a granular red and velvety patch? a. Sarcoma b. Papilloma c. Erythroplakia d. Leukoplakia ANS: C


Erythroplakia is a term to describe an intraoral lesion that appears as a smooth red patch or a granular red and velvety patch. A sarcoma is a malignant tumor of connective tissue. A papilloma appears as a white exophytic pedunculated growth. Leukoplakia appears as a white patch or plaque. REF: Erythroplakia, page 223

OBJ: 6

93. Squamous cell carcinoma may appear anywhere in the oral cavity; where is the most common

site? a. Floor of the mouth b. Gingiva c. Retromolar pad d. Midline of the hard palate ANS: A

Squamous cell carcinomas may occur anywhere in the oral cavity, but the most sites of involvement include the floor of the mouth. The gingiva is not a common site for squamous cell carcinoma. The retromolar pad is not a common site for squamous cell carcinoma. The midline of the hard palate is not a common site for squamous cell carcinoma, although it may appear on the soft palate. REF: Squamous Cell Carcinoma, page 224

OBJ: 6

94. Which is the most significant factor for the development of squamous cell carcinoma? a. Sun exposure b. Tobacco use c. Age d. Chronic irritant ANS: B

The most significant risk factor for the development of squamous cell carcinoma is tobacco, including cigar, pipe, and cigarette smoking. Sun exposure is a risk factor for the development of basal cell carcinoma. The majority of cases occur in patients over the age of 40, which is a young population. Chronic irritants, such as an ill-fitting denture, are not initiating factors in the development of squamous cell carcinoma. REF: Squamous Cell Carcinoma, page 225

OBJ: 6

95. Which is the false statement about basal cell carcinoma? a. It often occurs in white adults with fair complexions. b. It appears as a non healing ulcer with rolled borders. c. It is usually seen on the skin of the face. d. It occurs intraorally. ANS: D

Basal cell carcinoma does not occur in the oral cavity. Basal cell carcinoma does occur in fairskinned persons. Basal cell carcinoma appears as a non healing ulcer with characteristic rolled borders. Basal cell carcinoma frequently arises on the skin of the face. REF: Basal Cell Carcinoma, page 227

OBJ: 6


96. Where is the most common intraoral location for the development of minor salivary gland

tumors? a. Tongue b. Junction of the hard and soft palate c. Interdental papilla d. Floor of the mouth ANS: B

Intraorally, minor salivary gland tumors are located most commonly at the junction of the hard and soft palates. Minor salivary gland tumors rarely occur on the tongue. Minor salivary glands are not found in the interdental papilla. Minor salivary glands are found in the floor of the mouth, but this is not the most common location. REF: Salivary Gland Tumors, page 227

OBJ: 6

97. Because the origin of these tumors is glandular epithelium, benign tumors of salivary gland

origin are termed a. adenomas b. nevi c. chondromas d. lipomas ANS: A

An adenoma is a benign tumor that originates from glandular epithelium. Nevi are tumors of melanocytes. A chondroma is a benign tumor of cartilage. A lipoma is a benign tumor of fat. REF: Salivary Gland Tumors, page 227

OBJ: 6

98. Clinical signs and symptoms of a pleomorphic adenoma include the following except one.

Which is the exception? a. Nonulcerated lesion b. Painless c. Quickly enlarging d. Dome-shaped mass ANS: C

The pleomorphic adenoma is a slowly enlarging mass. Clinically, the pleomorphic adenoma appears as a nonulcerated mass. The pleomorphic adenoma is a painless lesion. Clinically, the pleomorphic adenoma is typically dome-shaped. REF: Pleomorphic Adenoma (Benign Mixed Tumor), page 227 OBJ: 6 99. What is the classic radiographic appearance of the ameloblastoma? a. Multilocular, soap-bubble radiolucency b. Associated with the crown of an unerupted tooth c. Unilocular or multilocular radiolucency with radiopaque areas within d. Multilocular lesions with poorly defined borders, causing tooth displacement ANS: A


The classic radiographic appearance of an ameloblastoma is a multilocular soap bubble-like or honeycombed radiolucency. The adenomatoid odontogenic tumor appears as a well-defined radiolucency usually associated with an impacted tooth. The calcifying odontogenic cyst appears as a unilocular or multilocular radiolucency with calcifications within. Multilocular lesions with poorly defined borders that can cause tooth displacement may be seen with the odontogenic myxoma. REF: Ameloblastoma, page 231

OBJ: 6

100. What is the treatment for a benign cementoblastoma? a. No treatment needed b. Surgical excision and drainage c. Enucleation of tumor and tooth extraction d. Surgery and radiation therapy follow-up ANS: C

Treatment of the benign cementoblastoma consists of enucleation of the tumor and removal of the involved tooth. Treatment is required for the benign cementoblastoma. Surgical excision and drainage is not the correct treatment protocol. Surgery and radiation therapy is not required for the benign cementoblastoma. REF: Cementoblastoma, page 235

OBJ: 6

MATCHING

For each neoplasm description below, select the corresponding type of neoplasm from the list provided. a. Benign b. Malignant 1. 2. 3. 4. 5. 6. 7.

Cells closely resemble their cells of origin. Mitosis is uncontrolled with rapid division. Growth expands by invasion and replaces normal tissue. The majority of these neoplasms are encapsulated. Recurrence of the tumor is not common. In naming the neoplasm, the word sarcoma or carcinoma is used. Cells are poorly differentiated.

1. ANS: A OBJ: 2 2. ANS: B OBJ: 2 3. ANS: B OBJ: 2 4. ANS: A OBJ: 2 5. ANS: A OBJ: 2 6. ANS: B 7. ANS: B

REF: Classification of Tumors, page 220 REF: Classification of Tumors, page 220 REF: Classification of Tumors, page 220 REF: Classification of Tumors, page 219 REF: Classification of Tumors, page 219 REF: Name of Tumors, page 220 OBJ: 2 REF: Classification of Tumors, page 220


OBJ: 2

For each vocabulary word below, select the corresponding definition from the list provided. a. Complete surgical removal without cutting into the lesion b. Deviating from the usual or natural type c. Confined to the site of origin without an invasion of neighboring tissues d. Tooth forming e. Occurring within bone f. Occurring in various forms g. Microscopic staining that is more intense than normal 8. 9. 10. 11. 12. 13. 14.

Central Odontogenic In situ Hyperchromatic Pleomorphic Aberrant Enucleation

8. 9. 10. 11. 12. 13. 14.

ANS: E ANS: D ANS: C ANS: G ANS: F ANS: B ANS: A

REF: REF: REF: REF: REF: REF: REF:

Vocabulary, page 218 Vocabulary, page 219 Vocabulary, page 219 Vocabulary, page 219 Vocabulary, page 219 Vocabulary, page 218 Vocabulary, page 218

OBJ: OBJ: OBJ: OBJ: OBJ: OBJ: OBJ:

1 1 1 1 1 1 1


Chapter 08: Nonneoplastic Diseases of Bone Ibsen: Oral Pathology for the Dental Hygienist, 7th Edition MULTIPLE CHOICE 1. Which statement is true of periapical cemento-osseous dysplasia? a. It is a neoplasm. b. The lesion is painful and occurs most often in the posterior maxilla. c. Teeth in the area are usually not vital. d. Older lesions are primarily radiopaque. ANS: D

Early lesions of periapical cemento-osseous dysplasia are radiolucent, whereas older lesions become more radiopaque. Periapical cemento-osseous dysplasia does not represent a neoplasm. The periapical cemento-osseous dysplasia lesion is asymptomatic and is discovered on routine radiographic examination. It occurs most commonly in the anterior mandible of patients older than 30 years of age. Teeth in the area affected by periapical cemento-osseous dysplasia are vital unless they are coincidentally carious or have been traumatized. REF: Periapical Cemento-Osseous Dysplasia, pages 259-260

OBJ: 4

2. Which type of lesions typically arises in the posterior mandible and appears as an isolated,

well-delineated radiolucent-to-radiopaque lesion that is less than 1.5 cm in size? a. Periapical cemento-osseous dysplasia b. Focal cemento-osseous dysplasia c. Florid cemento-osseous dysplasia d. Monostotic fibrous dysplasia ANS: B

Focal cemento-osseous dysplasia typically arises in the posterior mandible and appears as an isolated, well-delineated radiolucent-to-radiopaque lesion that is less than 1.5 cm in size. Periapical cemento-osseous dysplasia occurs most commonly in the anterior mandible. Florid cemento-osseous dysplasia typically affects more than one quadrant of the maxilla and mandible, often in the posterior area. Monostotic fibrous dysplasia commonly affects the mandible or maxilla, and the maxilla is more frequently involved than the mandible. REF: Focal Cemento-Osseous Dysplasia, page 260

OBJ: 4

3. Surgical treatment would most likely be recommended for a patient with ________ dysplasia. a. periapical cemento-osseous b. focal cemento-osseous c. florid cemento-osseous d. fibrous ANS: D


Surgical treatment would most likely be recommended for a patient with fibrous dysplasia. Fibrous dysplasia is treated surgically by recontouring the affected bone for cosmetic reasons. No treatment exists for severe and progressive polyostotic fibrous dysplasia. No treatment is necessary for periapical cemento-osseous dysplasia once the condition is recognized. Once a definitive diagnosis has been established for focal cemento-osseous dysplasia, no further treatment is necessary. Treatment for florid cemento-osseous dysplasia is often unnecessary. However, in an edentulous patient, the sclerotic masses may perforate the mucosa, resulting in a communication between the oral environment and the underlying bone. This may lead to osteomyelitis, requiring surgical intervention. REF: Polyostotic Fibrous Dysplasia, page 262

OBJ: 6

4. Fibrous dysplasia is characterized by a. necrosis and sloughing of bony sequestra. b. replacement of bone with normal fibrous connective tissue with no calcification. c. replacement of bone with abnormal connective tissue with no calcification. d. replacement of bone with abnormal connective tissue with varying amounts of

calcification. ANS: D

Fibrous dysplasia is characterized by replacement of bone with abnormal connective tissue with varying amounts of calcification. Fibrous dysplasia is characterized by replacement of bone with abnormal connective tissue with varying amounts of calcification rather than necrosis and sloughing of bony sequestra. Fibrous dysplasia is characterized by replacement of bone with abnormal connective tissue with varying amounts of calcification. Fibrous dysplasia is characterized by replacement of bone with abnormal connective tissue with varying amounts of calcification. REF: Fibrous Dysplasia, page 261

OBJ: 6

5. Characteristics of monostotic fibrous dysplasia include that a. it is commonly diagnosed in middle-aged women. b. the expanding nature of the lesion may lead to malocclusion, tipping, or

displacement of adjacent teeth. c. it involves a painful swelling or bulging of the buccal plate of the maxilla or

mandible when seen orally. d. it involves multiple bones. ANS: B

Characteristics of monostotic fibrous dysplasia include the possibility that the expanding nature of the lesion may lead to malocclusion, tipping, or displacement of adjacent teeth. Monostotic fibrous dysplasia is most commonly diagnosed in children and young adults; no sex predilection is seen. Clinical examination of a patient with monostotic fibrous dysplasia reveals a painless swelling or bulging of the buccal plate of the maxilla or mandible. Monostotic fibrous dysplasia involves a single bone. REF: Monostotic Fibrous Dysplasia, page 261 6. Café au lait skin macules are associated with a. monostotic fibrous dysplasia. b. polyostotic fibrous dysplasia.

OBJ: 6


c. Paget disease of bone. d. osteomalacia. ANS: B

Café au lait skin macules are associated with polyostotic fibrous dysplasia. Café au lait skin macules are associated with polyostotic fibrous dysplasia rather than monostotic fibrous dysplasia. Café au lait skin macules are associated with polyostotic fibrous dysplasia rather than Paget disease of bone. Café au lait skin macules are associated with polyostotic fibrous dysplasia rather than osteomalacia. REF: Polyostotic Fibrous Dysplasia, page 261

OBJ: 8

7. Paget disease of bone a. typically occurs in children under the age of 18. b. is painless. c. may reveal a “cotton-wool” appearance on radiographs. d. results in shrinkage of affected bone. ANS: C

Paget disease of bone may reveal a “cotton-wool” appearance on radiographs. Paget disease of bone occurs most commonly in men over the age of 50. The patient often complains of pain in cases of Paget disease. Enlargement of the affected bone is common in cases of Paget disease. REF: Paget Disease of Bone, page 262

OBJ: 9

8. Mosaic bone may be observed in a patient with Paget disease as a result of a. lack of trabeculae. b. prominent reversal lines resulting from resorption and deposition of bone. c. the presence of “giant cells.” d. a “ground glass” appearance. ANS: B

Mosaic bone may be observed in a patient with Paget disease as a result of prominent reversal lines caused by resorption and deposition of bone. Mosaic bone may be observed in a patient with Paget disease as a result of prominent reversal lines caused by resorption and deposition of bone. With Paget disease, the connective tissue between the bony trabeculae is so well vascularized that overlying skin may feel warm to the touch. Mosaic bone may be observed in a patient with Paget disease as a result of prominent reversal lines caused by resorption and deposition of bone. Giant cells are located around foreign bodies. Mosaic bone may be observed in a patient with Paget disease as a result of prominent reversal lines caused by resorption and deposition of bone. A “ground glass” appearance is associated with fibrous dysplasia. REF: Paget Disease of Bone, page 263

OBJ: 9

9. Which statement is true of osteomalacia? a. It is the result of a deficiency in phosphate. b. In adults it may be related to various problems such as malabsorption syndrome. c. It may be treated by the use of antacids. d. Accelerated tooth eruption is associated with osteomalacia. ANS: B


In adults, osteomalacia may be related to various problems such as malabsorption syndrome, drugs, liver and kidney disease, and the long-term use of antacids. Osteomalacia is the result of deficiency in calcium, not phosphate. Treatment of osteomalacia includes nutritional supplementation with vitamin D and dietary calcium, not antacids. Delayed tooth eruption and periodontal disease have been associated with osteomalacia. Pathologic fractures may also occur. REF: Osteomalacia, page 264

OBJ: 11

10. All conditions are benign fibrous-osseous lesions that affect the maxilla and mandible except

one. Which is the exception? a. Periapical cemento-osseous dysplasia b. Fibrous dysplasia c. Florid cemento-osseous dysplasia d. Gingival fibromatosis ANS: D

Gingival fibromatosis is an inherited disorder that affects the gingiva and periodontium. Periapical cemento-osseous dysplasia is a benign fibrous-osseous lesion that affects the maxilla and mandible. Fibrous dysplasia is a benign fibrous-osseous lesion that affects the maxilla and mandible. Florid cemento-osseous dysplasia is a benign fibrous-osseous lesion that affects the maxilla and mandible. REF: Benign Fibro-Osseous Lesions, pages 259-261

OBJ: 3

11. Which type of lesion occurs most often in black women older than age 40 and is characterized

by disordered cementum and bone, typically affecting more than one quadrant of the maxilla and mandible in the posterior regions? a. Periapical cemento-osseous dysplasia b. Focal cemento-osseous dysplasia c. Florid cemento-osseous dysplasia d. Monostotic fibrous dysplasia ANS: C

Florid cemento-osseous dysplasia occurs most often in black women older than 40 years of age and is characterized by disordered cementum and bone, typically affecting more than one quadrant of the maxilla and mandible, and often in the posterior area. Periapical cementoosseous dysplasia affects the periapical bone and occurs most often in the anterior mandible. Focal cemento-osseous dysplasia is more common in white women between 30 and 50 years of age. Monostotic fibrous dysplasia is characterized by involvement of a single bone and is commonly diagnosed in children and young adults. REF: Florid Cemento-Osseous Dysplasia, page 260

OBJ: 4

12. Cementoma is a term once used for periapical cemento-osseous dysplasia. Because the disease

does not represent a neoplasm, cementoma has been replaced with periapical cementoosseous dysplasia. a. Both statements are true. b. Both statements are false. c. The first statement is true, and the second is false. d. The first statement is false, and the second is true.


ANS: A

Both statements are true. Cementoma is a term once used for periapical cemento-osseous dysplasia. Because the disease does not represent a neoplasm, cementoma has been replaced with periapical cemento-osseous dysplasia. Both statements are true, and the other options are false. REF: Periapical Cemento-Osseous Dysplasia, page 259

OBJ: 4

13. For which disease of bone is surgical recontouring most often recommended as its treatment

modality? a. Periapical cemento-osseous dysplasia b. Florid cemento-osseous dysplasia c. Polyostotic fibrous dysplasia d. Focal cemento-osseous dysplasia ANS: C

Most often, the recommended treatment for polyostotic fibrous dysplasia is surgical recontouring. Treatment for periapical cemento-osseous dysplasia is typically unnecessary. Treatment for florid cemento-osseous dysplasia is typically unnecessary. Treatment for focal cemento-osseous dysplasia is typically unnecessary. REF: Polyostotic Fibrous Dysplasia, page 262

OBJ: 7

14. Numerous gritty pieces of soft and hard tissue that contain fibrous connective tissue

interspersed with bone trabeculae and cementum-like material is characteristic of which condition? a. Focal cemento-osseous dysplasia b. Central cementifying fibroma c. Florid cemento-osseous dysplasia d. Ossifying fibroma ANS: A

Numerous gritty pieces of soft and hard tissue that contain fibrous connective tissue interspersed with bone trabeculae and cementum-like material are characteristic of focal cemento-osseous dysplasia. This characteristic is distinctly different from the surgical features of central cementifying or ossifying fibromas. This characteristic is not found in florid cemento-osseous dysplasia. This characteristic is distinctly different from the surgical features of central cementifying or ossifying fibromas. REF: Focal Cemento-Osseous Dysplasia, page 260

OBJ: 5

15. The most common type of fibrous dysplasia, characterized by involvement of a single bone, is

termed a. Paget disease. b. polyostotic fibrous dysplasia. c. monostotic fibrous dysplasia. d. central giant cell granuloma. ANS: C


Monostotic fibrous dysplasia, the most common type of fibrous dysplasia, is characterized by involvement of a single bone. Paget disease is characterized by resorption, osteoblastic repair, and remineralization of involved bone, typically the pelvis and spinal column. It is not a type of fibrous dysplasia. Polyostotic fibrous dysplasia is characterized by involvement of more than one bone. Central giant cell granuloma is not a type of fibrous dysplasia. REF: Monostotic Fibrous Dysplasia, page 261

OBJ: 6

16. Light brown skin macules associated with polyostotic fibrous dysplasia are termed a. Koplik spots. b. café au lait spots. c. brown tumors. d. gritty tissue. ANS: B

Café au lait skin macules are associated with polyostotic fibrous dysplasia and appear as light brown skin macules. Koplik spots are erythematous macules associated with measles. Brown tumors are associated with hyperparathyroidism. Gritty tissue is characteristic of focal cemento-osseous dysplasia. REF: Polyostotic Fibrous Dysplasia, page 261

OBJ: 6

17. Osteitis deformans and leontiasis ossea are other names for what chronic metabolic bone

disease? a. Central giant cell granuloma b. Jaffe-Lichtenstein c. Paget disease d. Albright syndrome ANS: C

Osteitis deformans and leontiasis ossea are other names for Paget disease. Central giant cell lesion is another name for central giant cell granuloma. Jaffe-Lichtenstein is a type of fibrous dysplasia. Albright syndrome is the most severe form of fibrous dysplasia. REF: Paget Disease of Bone, page 262

OBJ: 9

18. This 74-year-old patient presents with an enlargement of the maxilla, pain, and a chief

complaint that his dentures no longer fit. Radiographic examination reveals a patchy radiolucency and radiopacity that resembles cotton wool. These findings are indicative of what disease of bone? a. Osteomalacia b. Paget disease of bone c. Fibrous dysplasia d. Florid cemento-osseous dysplasia ANS: B

This description and photograph are indicative of Paget disease. Osteomalacia does not present with an enlarged maxilla and cotton-wool appearance on radiographs. Fibrous dysplasia does not present with an enlarged maxilla and cotton-wool appearance on radiographs. Florid cemento-osseous dysplasia does not present with an enlarged maxilla and cotton-wool appearance on radiographs.


REF: Paget Disease of Bone, page 262

OBJ: 9

19. Giant cell granulomas located in the gingiva or alveolar mucosa are referred to as central.

Giant cell granulomas located within the maxilla or mandible are referred to as peripheral. a. Both statements are true. b. Both statements are false. c. The first statement is true; the second is false. d. The first statement is false; the second is true. ANS: B

Both statements are false. Central giant cell granulomas are located within the maxilla or mandible, and peripheral giant cell granulomas are located in the gingiva or alveolar mucosa. Both statements are false. The first statement is also false. The second statement is also false. REF: Central Giant Cell Granuloma (Central Giant Cell Lesion), page 263 OBJ: 10 20. A lesion of bone identical to the central giant cell granuloma and termed brown tumor occurs

in patients with what type of disease? a. Hyperparathyroidism b. Peripheral giant cell granuloma c. Hypothyroidism d. Peripheral ossifying granuloma ANS: A

A lesion of bone identical to the central giant cell granuloma and called a brown tumor occurs in patients with hyperparathyroidism. A brown tumor does not occur in patients with peripheral giant cell granuloma. A brown tumor does not occur in patients with hypothyroidism. A brown tumor does not occur in patients with peripheral ossifying granuloma. REF: Central Giant Cell Granuloma (Central Giant Cell Lesion), page 264 OBJ: 10 21. An aneurysmal bone cyst contains the following except one. Which is the exception? a. Fibrous connective tissue b. Epithelial lining c. Multinucleated giant cells d. Blood-filled spaces ANS: B

An aneurysmal bone cyst is a pseudocyst that has no epithelial lining. An aneurysmal bone cyst is a pseudocyst that consists of fibrous connective tissue. An aneurysmal bone cyst is a pseudocyst that consists of multinucleated giant cells. An aneurysmal bone cyst is a pseudocyst that consists of blood-filled spaces surrounded by multinucleated giant cells and fibrous connective tissue. REF: Aneurysmal Bone Cyst, page 264

OBJ: 1

22. Osteomalacia is a disease of bone that develops over a long period as a result of an iron

deficiency. Delayed tooth eruption and periodontal disease are associated with osteomalacia.


a. b. c. d.

Both statements are true. Both statements are false. The first statement is true, and the second is false. The first statement is false, and the second is true.

ANS: D

Osteomalacia is a result of a calcium deficiency, not iron deficiency. Delayed tooth eruption and periodontal disease are associated with osteomalacia. The first statement is false, and the second is true. REF: Osteomalacia, page 264

OBJ: 11

23. All statements are characteristic of osteomalacia except one. Which is the exception? a. It is the result of a deficiency in calcium. b. In adults it may be related to the chronic use of antacids. c. It is treated with vitamin D and calcium nutritional supplements. d. The pattern of diseased bone is best described as mosaic bone. ANS: D

The pattern of disease best described as mosaic bone is seen in Paget disease and is not associated with osteomalacia. Osteomalacia is the result of a deficiency in calcium. In adults osteomalacia may be related to various problems such as malabsorption syndrome, drugs, liver and kidney disease, and the long-term use of antacids. Treatment of osteomalacia includes nutritional supplementation with vitamin D and dietary calcium. REF: Osteomalacia, page 264

OBJ: 11

24. _______________ fibrous dysplasia is characterized by endocrine abnormalities, including

precocious puberty in females and stunting or deformity of skeletal growth in both sexes as a result of early epiphyseal plate closure. a. Craniofacial b. Jaffe-Lichtenstein type c. Albright syndrome d. Monostotic ANS: C

Albright syndrome fibrous dysplasia is characterized by endocrine abnormalities, including precocious puberty in females and stunting or deformity of skeletal growth in both sexes as a result of early epiphyseal plate closure. Craniofacial fibrous dysplasia involves the maxilla with extension into the sinuses and adjacent zygoma, sphenoid, and occipital bones. JaffeLichtenstein type fibrous dysplasia involves multiple bones along with café au lait macules on the skin. Monostotic fibrous dysplasia is characterized by involvement of a single bone. REF: Polyostotic Fibrous Dysplasia, page 261

OBJ: 8

25. “Ground glass” is the classic radiographic description of _________________. a. Osteomalacia b. Paget disease of bone c. Fibrous dysplasia d. Florid cemento-osseous dysplasia ANS: C


“Ground glass” is the radiographic description that is associated with fibrous dysplasia. A “ground glass” appearance is associated with fibrous dysplasia, not osteomalacia. A “ground glass” appearance is associated with fibrous dysplasia, not Paget disease of bone. A “ground glass” appearance is associated with fibrous dysplasia, not florid cemento-osseous dysplasia. REF: Fibrous Dysplasia, page 261

OBJ: 7

26. Laboratory evaluation reveals that the serum _______________ level is significantly elevated

in active Paget disease. a. alkaline phosphatase b. platelet c. hematocrit d. leukocyte ANS: A

Laboratory evaluation reveals that the serum alkaline phosphatase level is significantly elevated in active Paget disease. Laboratory evaluation reveals that the serum alkaline phosphatase level, not the platelet level, is significantly elevated in active Paget disease. Laboratory evaluation reveals that the serum alkaline phosphatase level, not the hematocrit level, is significantly elevated in active Paget disease. Laboratory evaluation reveals that the serum alkaline phosphatase level, not the leukocyte level, is significantly elevated in active Paget disease. REF: Paget Disease of Bone, page 262

OBJ: 9

27. When osteomalacia occurs in young children, it is usually caused by a nutritional deficiency

in vitamin _____. a. A b. D c. E d. K ANS: B

When osteomalacia occurs in young children, it is usually caused by a nutritional deficiency in vitamin D. The associated disease is called rickets. When osteomalacia occurs in young children, it is usually caused by a nutritional deficiency in vitamin D, not vitamin A. When osteomalacia occurs in young children, it is usually caused by a nutritional deficiency in vitamin D, not vitamin E. When osteomalacia occurs in young children, it is usually caused by a nutritional deficiency in vitamin D, not vitamin K. REF: Osteomalacia, page 264

OBJ: 11

28. Early lesions of periapical cemento-osseous dysplasia are well circumscribed and radiolucent

and may resemble which condition? a. Periapical abscess b. Benign cementoblastoma c. Odontoma d. Osteoma ANS: A


Early lesions of periapical cemento-osseous dysplasia are well circumscribed and radiolucent which may resemble a periapical abscess. The benign cementoblastoma is a well-defined radiopaque mass in continuity with the root of the affected tooth. The odontoma is an odontogenic tumor composed of enamel, dentin, cementum, and pulp. It appears as a cluster of miniature teeth, or, a radiopaque mass. The osteoma is a benign tumor of normal compact bone, which also has a radiopaque component. REF: Periapical Cemento-Osseous Dysplasia, page 260

OBJ: 4

29. Characteristics of periapical cemento-osseous dysplasia include the following except one.

Which is the exception? a. The lesions are seen in the anterior mandible. b. The common patient complaint is pain upon chewing. c. A predilection for black women exists. d. The condition is discovered on routine radiographic examination. ANS: B

This condition is asymptomatic. No pain upon chewing is experienced. Periapical cementoosseous dysplasia occurs most commonly in the anterior mandible. A predilection for black females older than age 30 exists. Because it is asymptomatic, periapical cemento-osseous dysplasia is discovered on radiographic examination. REF: Periapical Cemento-Osseous Dysplasia, page 260

OBJ: 4

30. What is one difference between the diagnosis of periapical cemento-osseous dysplasia and

florid cemento-osseous dysplasia? a. Florid cemento-osseous dysplasia usually affects more than one quadrant of the maxilla and mandible, including posterior regions. b. Periapical cemento-osseous dysplasia affects black females; florid cementoosseous dysplasia affects all ethnicities. c. The two conditions have various radiographic appearances. d. Symptoms of the two conditions vary from asymptomatic to extremely painful. ANS: A

Florid cemento-osseous dysplasia usually affects more than one quadrant of the maxilla and mandible, including posterior regions. It differs radiographically in location from periapical cemento-osseous dysplasia, which occurs most commonly in the anterior mandible. Black women older than 30 appear to be more commonly affected by both conditions. Periapical cemento-osseous dysplasia is asymptomatic and visible on radiographs with well-defined radiolucency to radiopacity at the area of the tooth apex. Florid cemento-osseous dysplasia is also asymptomatic in most cases and visible on radiographs with multiple areas of radiolucency and radiopacity. REF: Periapical Cemento-Osseous Dysplasia, page 260 | Florid Cemento-Osseous Dysplasia, page 260 OBJ: 4 31. Asymptomatic florid cemento-osseous dysplasia usually requires no treatment. However, in

an edentulous patient, the sclerotic masses may perforate the mucosa causing which complication? a. Fibrous dysplasia b. Aneurysmal bone cyst


c. Osteomyelitis d. Central giant cell granuloma ANS: C

Osteomyelitis is the infection and inflammation of the bone and bone marrow that may occur if the edentulous mucosa is perforated. Fibrous dysplasia is the replacement of bone with abnormal fibrous connective tissue. An aneurysmal bone cyst is a pseudocyst seen in young adults; it may cause expansion of the involved bone. Central giant cell granuloma is composed of connective tissue containing multinucleated giant cells. REF: Florid Cemento-Osseous Dysplasia, page 260

OBJ: 5

32. Café au lait spots appear on the skin as which color(s)? a. Reddish pink b. Light brown c. Yellow white d. Black ANS: B

This name was given to these lesions because the spots mimic the color of coffee with milk. Café au lait spots appear as light brown macules. Yellow white may be a color to describe Fordyce granules. REF: Polyostotic Fibrous Dysplasia, page 261

OBJ: 8

33. The painless jaw swelling seen in monostotic fibrous dysplasia may cause all symptoms

except one. Which is the exception? a. Tooth mobility b. Malocclusion c. Displacement of adjacent teeth d. Tipping of teeth ANS: A

A clinical examination reveals a painless swelling or bulging of the buccal plate of the mandible or maxilla, yet the teeth are rarely mobile. Malocclusion may be seen because of the swelling and bulging jaws. Displacement of adjacent teeth is common if the swelling continues. Tipping of the teeth may be seen because of the expanding nature of this lesion. REF: Fibrous Dysplasia, page 261

OBJ: 6

34. Characteristics of polyostotic fibrous dysplasia include the following except one. Which is the

exception? a. Most cases are asymptomatic. b. Male predilection c. Occurs in children under 10 years of age d. Seen in association with skin lesions ANS: B

A definite female predilection is seen. Polyostotic fibrous dysplasia is characterized by involvement of more than one bone, usually affecting the skull, clavicles, and long bones. Most cases are asymptomatic. Most cases involve children under 10 years of age. The skin lesions associated with polyostotic fibrous dysplasia are café au lait spots.


REF: Polyostotic Fibrous Dysplasia, page 261

OBJ: 6

35. Why is treatment with radiation therapy for polyostotic fibrous dysplasia contraindicated? a. It is too severe of a treatment for a young child. b. An association exists with malignant transformation to osteosarcoma. c. Because of the defined borders of the lesions, surgery is the treatment of choice. d. The lesions remain asymptomatic and localized. ANS: B

Radiation treatment is contraindicated because it has been associated with malignant transformation to osteosarcoma. If necessary, radiation therapy may be used on a young child. The abnormal bone blends into the adjacent normal bone, making it difficult to determine the periphery of the lesion. Polyostotic fibrous dysplasia involves more than one bone, so it is not a localized condition. REF: Polyostotic Fibrous Dysplasia, page 262

OBJ: 8

36. All are likely complaints of a patient with Paget disease of bone except one. Which is the

exception? a. “My partial denture does not fit.” b. “I can’t hear you, can you speak louder?” c. “I have spaces between my teeth that I never had before.” d. “My tongue is always sore.” ANS: D

Tongue soreness should not be a common complaint. Edentulous or partially edentulous patients may complain that their dentures no longer fit as the bone enlarges with Paget disease. Common manifestations of Paget disease include deafness, dizziness, and headache. Spacing between the teeth increases as the bone enlarges. REF: Paget Disease of Bone, pages 262-263

OBJ: 9

37. The serum alkaline phosphatase level is significantly elevated in persons with active Paget

disease. This is detected through which diagnostic process? a. Historical b. Laboratory c. Therapeutic d. Differential ANS: B

An elevated serum alkaline phosphatase level is significant in the diagnosis of Paget disease. This information can be gained from clinical laboratory tests. Historical information includes personal, family, and medical histories not related to the serum alkaline phosphatase level. Therapeutic means may not be an appropriate diagnostic process for Paget disease. Differential diagnosis is not necessary when correct laboratory procedures are performed. REF: Paget Disease of Bone, page 262

OBJ: 9

38. The aneurysmal bone cyst is a “pseudocyst” which means: a. No epithelial lining exists. b. The cyst does not appear on radiographic images.


c. The cyst is really a fibroma. d. Malignant potential is present. ANS: A

Pseudocysts, including the aneurysmal bone cyst, contain no epithelial lining. This cyst does appear on radiographic images with a honeycomb or soap bubble appearance. The cyst is not a fibroma. Malignant potential is not present. REF: Aneurysmal Bone Cyst, page 264

OBJ: 10

39. What is a common feature of the central giant cell granuloma? a. Heat and cold sensitivity b. Fast-growing lesion c. High recurrence rate d. Divergence of roots of teeth adjacent to the lesion ANS: D

Divergence of the roots of teeth adjacent to the lesion is a common feature of central giant cell granuloma. Heat and cold sensitivity has not been reported with central giant cell granuloma. The lesion is slow growing and destructive. Lesions are treated by surgical removal and occasionally recur. REF: Central Giant Cell Granuloma (Central Giant Cell Lesion), page 263 OBJ: 10 40. Osteomalacia is a disease of bone that develops over a long period of time as a result of

deficiency of which substance? a. Connective tissue b. Calcium c. Fluoride d. Vitamin A ANS: B

This is a disease of bone that occurs because of a calcium deficiency. Osteomalacia is not related to a deficiency of connective tissue. Osteomalacia is not related to a deficiency of fluoride. When osteomalacia occurs in young children, it is caused by a deficiency of vitamin D. REF: Osteomalacia, page 264

OBJ: 11

41. Where does the disease extend when fibrous dysplasia involves the maxilla? a. Nasal cavity b. Orbits c. Maxillary sinus d. Periapical areas of maxillary molars ANS: C

Obliteration of the maxillary sinus may be seen when fibrous dysplasia involves the maxilla, as well as involvement with the zygoma, sphenoid, and occipital bones. Fibrous dysplasia does not typically affect the area of the nasal cavity. Fibrous dysplasia does not typically affect the area of the orbits. Fibrous dysplasia can affect the periapical regions of maxillary molars, but the enlargement extends further.


REF: Monostotic Fibrous Dysplasia, page 261

OBJ: 6

42. The most severe form of polyostotic fibrous dysplasia is termed Albright syndrome. This

condition is characterized by endocrine abnormalities such as precocious puberty that may begin at which age? a. 2 years b. 4 years c. 6 years d. 8 years ANS: A

Precocious puberty, such as menses, pubic hair, and breast development, may begin in children as young as age 2. REF: Polyostotic Fibrous Dysplasia, page 261

OBJ: 6


Chapter 09: Oral Manifestations of Systemic Diseases Ibsen: Oral Pathology for the Dental Hygienist, 7th Edition MULTIPLE CHOICE 1. Excess production by the anterior pituitary gland is termed a. hyperparathyroidism. b. hyperthyroidism. c. hyperpituitarism. d. hyperglycemia. ANS: C

Excess production by the anterior pituitary gland is termed hyperpituitarism. Hyperparathyroidism results from excess secretion of parathyroid hormone, which is secreted by the parathyroid glands. Hyperthyroidism is characterized by excessive production of thyroid hormone. Hyperglycemia can result from a lack of the hormone insulin, defective insulin that does not work effectively to lower blood glucose levels, or increased insulin resistance because of obesity. REF: Hyperpituitarism, page 272

OBJ: 1

2. The most common cause of hyperpituitarism is a. hepatomegaly. b. hypercalcemia. c. pituitary adenoma. d. hyperglycemia. ANS: C

The most common cause of hyperpituitarism is pituitary adenoma. Hepatomegaly is an enlargement of the liver. Hypercalcemia is an excess of calcium in the blood. Hyperglycemia is an excess of glucose in the blood. REF: Hyperpituitarism, page 272

OBJ: 1

3. Characteristics of acromegaly include a. an increase in growth hormone production before the closure of long bones. b. a rapid and dramatic onset. c. thinned lips and microglossia. d. enlargement of the maxilla and mandible, resulting in separation of the teeth and

malocclusion. ANS: D

Characteristics of acromegaly include enlargement of the maxilla and mandible, resulting in separation of the teeth and malocclusion. Acromegaly is an increase in growth hormone production during adult life. Gigantism results when there is an increase in growth hormone production before the closure of long bones. Acromegaly affects both men and women and most commonly occurs in the fourth decade of life. The onset of acromegaly is slow and insidious. Mucosal changes such as thickened lips and macroglossia have been described in patients with acromegaly.


REF: Hyperpituitarism, page 272

OBJ: 2

4. The most common cause of hyperthyroidism is a. hyperplasia of the gland. b. benign and malignant tumors of the gland. c. pituitary gland disease. d. an autoimmune disorder. ANS: D

The most common cause of hyperthyroidism is an autoimmune disorder called Graves’ disease. The exact cause of Graves’ disease is not clear, but a substance is produced that abnormally stimulates the thyroid gland. Other causes of hyperthyroidism include hyperplasia of the gland, benign and malignant tumors of the thyroid, pituitary gland disease, and metastatic tumors. Hyperplasia of the thyroid gland is one of the causes of hyperthyroidism; however, the most common cause of hyperthyroidism is an autoimmune disorder called Graves’ disease. Benign and malignant tumors of the thyroid gland are one of the causes of hyperthyroidism; however, the most common cause of hyperthyroidism is an autoimmune disorder called Graves’ disease. Pituitary gland disease is one of the causes of hyperthyroidism; however, the most common cause of hyperthyroidism is an autoimmune disorder called Graves’ disease. REF: Hyperthyroidism (Thyrotoxicosis, Graves Disease), page 272 OBJ: 3 5. Which clinical feature is associated with hyperthyroidism? a. Gigantism b. Exophthalmos c. Myxedema d. Hypercalcemia ANS: B

In addition to exophthalmos, clinical features of hyperthyroidism include rosy complexion, erythema of the palms, excessive sweating, fine hair, and softened nails. Anxiety, weakness, restlessness, and cardiac problems may be associated with the disorder. Gigantism results if an increase in growth hormone production occurs before the closure of long bones. Myxedema occurs when a decreased output of thyroid hormone in older children and adults exists. When hypothyroidism is present during infancy and childhood, it is known as cretinism. Hypercalcemia is associated with hyperparathyroidism. REF: Hyperthyroidism (Thyrotoxicosis, Graves Disease), Clinical Features and Oral Manifestations, page 273 OBJ: 3 6. An enlarged tongue is associated with a. hyperpituitarism. b. hyperthyroidism. c. hypothyroidism. d. hyperparathyroidism. ANS: C


An enlarged tongue is associated with hypothyroidism. Causes of hypothyroidism include developmental disturbances, autoimmune disease, iodine deficiency, drugs, and pituitary disease. In infants, facial and oral changes include thickened lips, enlarged tongue, and delayed eruption of teeth. Clinical features of hyperpituitarism include gigantism if the increase in growth hormone production occurs before the closure of long bones and acromegaly after the closure of long bones. Clinical features of hyperthyroidism include rosy complexion, erythema of the palms, excessive sweating, fine hair, and softened nails. Clinical manifestations associated with hyperparathyroidism are varied. Joint pain or stiffness may be present. The disease can affect the kidneys, skeletal system, and gastrointestinal system. Lethargy and coma can occur in severe disease. REF: Hypothyroidism (Cretinism, Myxedema), page 273

OBJ: 3

7. Which statement is characteristic of hyperparathyroidism? a. It is characterized by low blood levels of calcium. b. It is characterized by high levels of blood phosphorus. c. Primary hyperparathyroidism occurs when calcium is abnormally excreted by the

kidneys and the parathyroid glands increase their production of parathyroid hormone to maintain adequate blood levels of calcium. d. Kidney failure is the most common cause of secondary hyperparathyroidism. ANS: D

Kidney failure is the most common cause of secondary hyperparathyroidism. Hyperparathyroidism is characterized by elevated blood levels of calcium. Hyperparathyroidism is characterized by low levels of blood phosphorus. Primary hyperparathyroidism results from an abnormality of the parathyroid glands. Secondary hyperparathyroidism occurs when calcium is abnormally excreted by the kidneys and the parathyroid glands increase their production of parathyroid hormone to maintain adequate blood levels of calcium. REF: Hyperparathyroidism, page 273

OBJ: 4

8. Hyperparathyroidism may result in bone lesions that appear microscopically to be

indistinguishable from a(n) a. osteogenic sarcoma. b. osteoma. c. cemento-osseous dysplasia. d. central giant cell granuloma. ANS: D

Hyperparathyroidism may result in bone lesions that appear microscopically to be indistinguishable from a central giant cell granuloma. Other radiographic changes that occur in hyperparathyroidism include a generalized mottled appearance of the bone and partial loss of the lamina dura. Loosening of teeth can also occur. Hyperparathyroidism may result in bone lesions that appear microscopically to be indistinguishable from a central giant cell granuloma rather than osteogenic sarcoma. Hyperparathyroidism may result in bone lesions that appear microscopically to be indistinguishable from a central giant cell granuloma rather than osteoma. Hyperparathyroidism may result in bone lesions that appear microscopically to be indistinguishable from a central giant cell granuloma rather than cemento-osseous dysplasia.


REF: Hyperparathyroidism, Clinical Features and Oral Manifestations, page 273 OBJ: 4 9. Insulin is manufactured by a. fatty tissue in long bones. b. beta cells of the pancreas. c. the liver. d. the gallbladder. ANS: B

Insulin is manufactured by beta cells of the pancreas. Insulin is then secreted directly into the bloodstream to facilitate the uptake of glucose into fat and skeletal muscle cells. Fat and skeletal muscle cells can use glucose as an energy source in the presence of insulin. The cells are starved of energy when insulin is lacking. Insulin is manufactured by beta cells of the pancreas rather than by fatty tissue in long bones. Insulin is manufactured by beta cells of the pancreas rather than by the liver. Insulin is manufactured by beta cells of the pancreas rather than by the gallbladder. REF: Diabetes Mellitus, page 274

OBJ: 6

10. Which statement about type 1 diabetes is true? a. It is called non-insulin–dependent diabetes mellitus. b. About 30% of all patients with diabetes have type 1 diabetes. c. The onset of this type of diabetes is gradual and usually occurs in patients who are

35 to 40 years of age or older. d. It can be complicated by polydipsia, polyuria, and polyphagia. ANS: D

Type 1 diabetes can be complicated by polydipsia (excessive thirst and intake of fluid), polyuria (excessive urination), and polyphagia (excessive appetite). It is called insulindependent diabetes mellitus. Only 3% of all diabetic patients have this type of diabetes. Type 1 diabetes can occur at any age, but its time of onset is usually at a peak age of 20 years. The onset is abrupt. REF: Insulin-Dependent Diabetes Mellitus, Pathology, and Clinical Management, page 275 OBJ: 6 11. Advantages of an insulin pump for therapy of type 1 diabetes include a. blood sugar levels no longer need to be monitored. b. the pump is internal. c. the pump can maintain a more predictable control of blood sugar levels than

multiple insulin injections. d. the insulin dosage with the pump is limited to once-a-day dosing. ANS: C


Advantages of an insulin pump for therapy of type 1 diabetes include the ability of the pump to maintain more predictable control of blood sugar levels than multiple insulin injections. The patient must still monitor blood sugar levels by testing several times a day. The insulin pump is external and approximately the size of a pager; it delivers insulin through plastic tubing placed under the skin. The patient programs the pump to deliver small amounts of rapidly acting insulin on the hour over a 24-hour period according to the patient’s metabolic needs. REF: The Insulin Pump, page 276

OBJ: 6

12. The most common cause of death among patients with diabetes is a. stroke. b. gangrene. c. heart attack. d. kidney failure. ANS: C

The most common cause of death among diabetics is heart attack. Atherosclerosis of the coronary arteries increases the risk of ulceration and gangrene of the feet, high blood pressure, kidney failure, and stroke. Gangrene of the lower extremities is about 100 times more common in patients with diabetes than in the general population. The most common cause of death among patients with diabetes is heart attack and not stroke. Heart attack is caused by atherosclerosis of the coronary arteries, which also increases the risk for stroke. The most common cause of death among patients with diabetes is heart attack and not gangrene. Heart attack is caused by atherosclerosis of the coronary arteries, which also increases the risk for gangrene of the feet. The most common cause of death among patients with diabetes is heart attack and not kidney failure. Heart attack is caused by atherosclerosis of the coronary arteries, which also increases the risk for kidney failure. REF: Complications of Type 1 Diabetes., page 276

OBJ: 6

13. Which statement is true concerning oral complications for adult patients with diabetes

mellitus? a. Patients with diabetes mellitus have a decreased response to plaque. b. Slow wound healing and decreased susceptibility to infection occur as a result of the immunologic changes and defective collagen production. c. The gingiva can be hyperplastic and erythematous. d. Periodontal disease is not considered to be a complication of diabetes. ANS: C

The gingiva can be hyperplastic and erythematous, and acute and fulminating gingival abscesses can occur. Patients with diabetes mellitus have an accentuated response to plaque. Excessive periodontal bone loss, tooth mobility, and early tooth loss can also be associated with diabetes mellitus. Slow wound healing and increased susceptibility to infection occur as a result of the immunologic changes and defective collagen production. Periodontal disease is considered to be a complication of diabetes. REF: Gestational Diabetes, Clinical Features and Oral Manifestations of Diabetes, page 278 OBJ: 6 14. Pernicious anemia is caused by a deficiency of intrinsic factor, which is secreted by


a. b. c. d.

beta cells of the pancreas. parietal cells of the stomach. the gallbladder. the adrenal glands.

ANS: B

Pernicious anemia is caused by a deficiency of intrinsic factor, which is secreted by parietal cells of the stomach. Intrinsic factor is necessary for the absorption of vitamin B12. Normally, vitamin B12 is transported across the intestinal mucosa by intrinsic factor. Insulin is secreted by beta cells of the pancreas. Bile is secreted by the gallbladder. Adrenalin is secreted by the adrenal glands. REF: Pernicious Anemia, page 280

OBJ: 8

15. A megaloblastic anemia is characterized by red blood cells that are a. hypochromic and macrocytic. b. hyperchromic and macrocytic. c. microcytic and have nuclei. d. macrocytic and have nuclei. ANS: D

A megaloblastic anemia is characterized by red blood cells that are immature, macrocytic (abnormally large), and have nuclei (megaloblasts). Immature neutrophils and platelets are also seen both in the bone marrow and the circulating blood. A megaloblastic anemia is characterized by red blood cells that are macrocytic and have nuclei. In iron deficiency anemia the red blood cells appear lighter in color (hypochromic) and smaller than normal (microcytic). A megaloblastic anemia is characterized by red blood cells that are macrocytic and have nuclei. In iron deficiency anemia, the red blood cells appear lighter in color (hypochromic) and smaller than normal (microcytic). A megaloblastic anemia is characterized by red blood cells that are immature, macrocytic (abnormally large), and have nuclei (megaloblasts). REF: Pernicious Anemia, Diagnosis and Treatment, page 281

OBJ: 8

16. Thalassemia is a. caused by an abnormal type of hemoglobin in red blood cells. b. a disorder of hemoglobin synthesis. c. caused by a severe depression of bone marrow activity. d. caused by an increase in the number of circulating red blood cells. ANS: B

Thalassemia is a disorder of hemoglobin synthesis. It has an autosomal inheritance pattern; no predilection for either sex is seen. Sickle cell anemia is caused by an abnormal type of hemoglobin in red blood cells. Aplastic anemia is caused by a severe depression in bone marrow activity. Polycythemia is caused by an increase in the number of circulating red blood cells. REF: Thalassemia, page 281

OBJ: 9

17. Mediterranean or Cooley anemia a. is caused by a deficiency of vitamin B12.


b. has a sex-linked inheritance pattern and occurs predominantly in males. c. is called thalassemia major in the homozygous form. d. is caused by a dietary deficiency of folic acid. ANS: C

Mediterranean or Cooley anemia is called thalassemia major in the homozygous form, in which the genes on both chromosomes are involved. It is associated with severe hemolytic anemia, which results from damage to the red blood cell membranes and destruction of the red blood cells. Pernicious anemia is a vitamin B12 deficiency that is caused by a deficiency of intrinsic factor. Mediterranean or Cooley anemia has an autosomal inheritance pattern, and no predilection for either sex is seen. Dietary deficiencies of folic acid and vitamin B12 can result in anemia and can occur in association with malnutrition and increased metabolic requirements; however, Mediterranean or Cooley anemia is not caused by a dietary or folic acid deficiency. REF: Thalassemia, page 281

OBJ: 9

18. A “salt and pepper” effect may be noted on intraoral radiographic images of a patient with

which type of anemia? a. Sickle cell anemia b. Aplastic anemia c. Thalassemia d. Pernicious anemia ANS: C

A “salt and pepper” effect may be noted on intraoral radiographic images of a patient with thalassemia. Intraoral radiographs of a patient with thalassemia show a peculiar trabecular pattern of the maxilla and mandible. The “salt and pepper” effect is the result of the prominence of some trabeculae and a blurring and disappearance of others. Thinning of the lamina dura and circular radiolucencies in the alveolar bone have also been described. Changes in the skulls of patients with sickle cell anemia have been described as a “hair-onend” pattern because the trabeculae radiate outward. In aplastic anemia, patients experience a dramatic decrease in all circulating blood cells because of a severe depression of bone marrow activity. The diagnosis of pernicious anemia is made by laboratory testing. REF: Thalassemia, Clinical Features and Oral Manifestations, page 282 OBJ: 9 19. Because of an abnormal type of hemoglobin in the red blood cells of a patient with sickle cell

anemia, the cells develop a sickle shape in the presence of a. low blood sugar. b. decreased oxygen. c. wheat gluten. d. increased carbon dioxide. ANS: B


Because of an abnormal type of hemoglobin in the red blood cells of a patient with sickle cell anemia, the cells develop a sickle shape in the presence of decreased oxygen. Exercise, exertion, administration of a general anesthetic, pregnancy, or even sleep can trigger a sickling of the red blood cells. Low blood sugar is associated with insulin and diabetes. Celiac sprue is a chronic disorder associated with a sensitivity to dietary gluten, a protein found in wheat and wheat products. The cells develop a sickle shape in the presence of decreased oxygen rather than increased carbon dioxide. REF: Sickle Cell Anemia, page 282

OBJ: 9

20. A “hair-on-end” pattern may be noted on skull radiographic images of a patient with a. polycythemia. b. aplastic anemia. c. sickle cell anemia. d. thalassemia. ANS: C

A “hair-on-end” pattern may be noted on skull radiographic images of a patient with sickle cell anemia. This occurs because the trabeculae radiate outward. Oral manifestations are seen on dental radiographs. A loss of trabeculations takes place, with the appearance of large, irregular marrow spaces. Polycythemia is characterized by an increase in the number of circulating red blood cells. Aplastic anemia is characterized by a generalized decrease in circulating blood cells. Thalassemia is the name of a group of inherited disorders of hemoglobin synthesis. REF: Sickle Cell Anemia, Clinical Features and Oral Manifestations, page 283 OBJ: 9 21. Which disorder of red blood cells and hemoglobin is associated with sensitivity to dietary

gluten found in wheat and wheat products? a. Aplastic anemia b. Polycythemia vera c. Secondary polycythemia d. Celiac sprue ANS: D

Celiac sprue is associated with sensitivity to dietary gluten found in wheat and wheat products. When gluten is ingested, injury to the intestinal mucosa results. Malabsorption of other nutrients such as vitamin B12 and folic acid occurs as a result of mucosal injury. Aplastic anemia is characterized by a generalized decrease in circulating blood cells. Polycythemia vera is characterized by a neoplastic proliferation of bone marrow stem cells resulting in an abnormally high number of circulating red blood cells. Secondary polycythemia is characterized by an increase in red blood cells caused by a physiologic response to decreased oxygen. A decrease in oxygen in blood cells triggers an increase in erythropoietin by the kidneys, which results in increased production of red blood cells. REF: Celiac Disease, page 286

OBJ: 10

22. Which statement about primary aplastic anemia is true? a. It is caused by a virus. b. The bone marrow failure is the result of a drug or chemical agent.


c. It is associated with chemotherapy, radioactive isotopes, radium, or radiant energy. d. The cause is unknown. ANS: D

The cause of primary aplastic anemia is unknown. In secondary aplastic anemia, bone marrow failure is the result of a drug or chemical agent. Chemotherapy, radioactive isotopes, radium, or radiant energy has been associated with the development of aplastic anemia. Primary aplastic anemia occurs most frequently in young adults. It is not caused by a virus. In secondary aplastic anemia, bone marrow failure is a result of a drug or chemical agent. REF: Aplastic Anemia, page 283

OBJ: 11

23. Polycythemia vera is caused by a. a neoplastic proliferation of bone marrow stem cells. b. a physiologic response to decreased oxygen. c. decreased plasma volume. d. smoking. ANS: A

Polycythemia vera is caused by a neoplastic proliferation of bone marrow stem cells. It results in an abnormally high number of circulating red blood cells. The production of red blood cells is uncontrolled. It is not caused by a physiologic response to decreased oxygen. It is not caused by decreased plasma volume. It is not caused by smoking. REF: Polycythemia Vera (Primary Polycythemia), page 283

OBJ: 13

24. Which statement concerning polycythemia vera is true? a. It may be caused by living at high altitudes or by an elevation in carbon monoxide. b. Causes include use of diuretics, vomiting, diarrhea, and excessive sweating. c. It occurs primarily in children. d. The cause is unknown. ANS: D

The cause of polycythemia vera is unknown. It is somewhat more common in men than in women, and the age of onset is usually between 40 and 60 years of age. Symptoms include headache, dizziness, and itching. Pulmonary disease, heart disease, living at high altitudes, and an elevation in carbon monoxide are all associated with secondary polycythemia rather than polycythemia vera. Causes of acute relative polycythemia rather than polycythemia vera include use of diuretics, vomiting, diarrhea, and excessive sweating. Polycythemia vera is somewhat more common in men than in women, and the age of onset is usually between 40 and 60 years of age. REF: Polycythemia Vera (Primary Polycythemia), page 283 25. Leukopenia most commonly involves a. neutrophils. b. basophils. c. eosinophils. d. monocytes. ANS: A

OBJ: 13


Leukopenia most commonly involves neutrophils, although any of the white blood cells can be involved. Leukopenia most commonly involves neutrophils rather than basophils, although any of the white blood cells can be involved. Leukopenia most commonly involves neutrophils rather than eosinophils, although any of the white blood cells can be involved. Leukopenia most commonly involves neutrophils rather than monocytes, although any of the white blood cells can be involved. REF: Aplastic Anemia, Diagnosis and Treatment, page 283

OBJ: 15

26. Which statement is true concerning acute lymphoblastic leukemia versus acute myeloblastic

leukemia? a. Acute lymphoblastic leukemia primarily affects children and young adults and has a poor prognosis. b. Acute myeloblastic leukemia primarily affects children and young adults and has a poor prognosis. c. Acute lymphoblastic leukemia primarily affects children and young adults and has a good prognosis. d. Acute myeloblastic leukemia primarily affects children and young adults and has a good prognosis. ANS: C

Acute lymphoblastic leukemia primarily affects children and young adults and has a good prognosis. Acute myeloblastic leukemia involves adolescents and young adults (age range, 15 to 39 years), and the prognosis is not as good. Acute myeloblastic leukemia involves adolescents and young adults (age range, 15 to 39 years), and the prognosis is not as good as with acute lymphoblastic leukemia. Acute myeloblastic leukemia involves adolescents and young adults (age range, 15 to 39 years), and the prognosis is not as good. REF: Acute Leukemias, page 285

OBJ: 15

27. The Philadelphia chromosome is associated with _____________________ leukemia. a. acute lymphoblastic b. acute nonlymphoblastic c. chronic lymphoblastic d. chronic granulocytic ANS: D

The Philadelphia chromosome is associated with chronic granulocytic (myeloid) leukemia. Chronic lymphocytic leukemia is the most common form and accounts for about one quarter of the total cases of leukemia. About half of the patients with this type of leukemia have abnormal karyotypes; however, the abnormality is different from the Philadelphia chromosome. The Philadelphia chromosome is associated with chronic granulocytic (myeloid) leukemia, not acute lymphoblastic leukemia. The Philadelphia chromosome is associated with chronic granulocytic (myeloid) leukemia, not acute nonlymphoblastic leukemia. The Philadelphia chromosome is associated with chronic granulocytic (myeloid) leukemia, not chronic lymphoblastic leukemia. REF: Chronic Leukemias, page 286

OBJ: 14

28. The normal laboratory value for prothrombin time is a. 1 to 3 seconds.


b. 11 to 16 seconds. c. 25 to 40 seconds. d. 1 to 6 minutes. ANS: B

The normal laboratory value for prothrombin time is 11 to 16 seconds. The normal value for partial thromboplastin time is 25 to 40 seconds. The normal value for bleeding time is 1 to 6 minutes. The normal laboratory value for prothrombin time is 11 to 16 seconds, not 1 to 3 seconds. The normal laboratory value for prothrombin time is 11 to 16 seconds, not 25 to 40 seconds. The normal laboratory value for prothrombin time is 11 to 16 seconds, not 1 to 6 minutes. REF: Prothrombin Time, page 288

OBJ: 16

29. The International Normalized Ratio (INR) is an expression of the ratio of prothrombin time to a. fibrinogen. b. thromboplastin activity. c. proaccelerin. d. proconvertin. ANS: B

The INR is an expression of the ratio of prothrombin time to thromboplastin activity. Values less than 3 are considered within normal range. Patients taking anticoagulants such as warfarin may have INR values of 4 to 5. The INR is an expression of the ratio of prothrombin time to thromboplastin activity and not fibrinogen. Fibrinogen is factor I in coagulation. The INR is an expression of the ratio of prothrombin time to thromboplastin activity and not proaccelerin. Proaccelerin is factor V in coagulation. The INR is an expression of the ratio of prothrombin time to thromboplastin activity and not proconvertin. Proconvertin is factor VII in coagulation. REF: Prothrombin Time, page 288

OBJ: 16

30. Thrombocytopenic purpura results from a a. defect in the capillary walls. b. disorder of platelet function. c. severe reduction in circulating platelets. d. reduction in white blood cells. ANS: C

Thrombocytopenic purpura results from a severe reduction in circulating platelets. When the cause is unknown, the condition is called idiopathic thrombocytopenic purpura. Nonthrombocytopenic purpuras are bleeding disorders that can result from either a defect in the capillary walls or disorders of platelet function. Nonthrombocytopenic purpuras are bleeding disorders that can result from either a defect in the capillary walls or disorders of platelet function. Nonthrombocytopenic purpuras are bleeding disorders that can result from either a defect in the capillary walls or disorders of platelet function. Thrombocytopenic purpura results from a severe reduction in circulating platelets rather than a reduction in white blood cells. REF: Thrombocytopenic Purpura, page 288

OBJ: 17


31. Which drug may be taken during the course of radiation treatment to reduce the severity of

radiation-induced xerostomia? a. Prednisone b. Pilocarpine hydrochloride c. Aspirin d. Nonsteroidal anti-inflammatory drugs ANS: B

Pilocarpine hydrochloride may be taken during the course of radiation treatment to reduce the severity of radiation-induced xerostomia. A patient who has received radiation therapy for oral cancer is also at risk for the development of osteoradionecrosis because of decreased blood supply to the bone after radiation therapy. Aspirin is a peripheral, nonnarcotic analgesic. Nonsteroidal anti-inflammatory drugs are peripheral, nonnarcotic analgesics. REF: Radiation Therapy, page 290

OBJ: 21

32. Osteonecrosis of the maxilla and mandible is a complication associated with what type of

drug? a. Phenytoin b. Nifedipine c. Bisphosphonate d. Cyclosporine ANS: C

Bisphosphonate therapy can cause osteonecrosis of the maxilla and mandible. Phenytoin can cause gingival enlargement but not osteonecrosis. Nifedipine can cause gingival enlargement but not osteonecrosis. Cyclosporine can cause gingival enlargement but not osteonecrosis. REF: Medication-Related Osteonecrosis of the Jaw, page 291

OBJ: 22 | 23

33. Bisphosphonate therapy is used to treat all diseases except one. Which is the exception? a. Hyperparathyroidism b. Osteoporosis c. Multiple myeloma d. Metastatic breast cancer ANS: A

Endocrine disorders are commonly treated with surgery and some nonbisphosphonate medications. Oral bisphosphonate therapy is used to treat osteoporosis. Intravenous bisphosphonate therapy is used to treat multiple myeloma. Intravenous bisphosphonate therapy is used to treat metastatic breast cancer. REF: Medication-Related Osteonecrosis of the Jaw, page 292

OBJ: 23

34. Which medication can decrease the severity of radiation-induced xerostomia in patients with

oral cancer? a. Bisphosphonates b. Cyclosporine c. Indomethacin d. Pilocarpine hydrochloride ANS: D


Pilocarpine hydrochloride taken during the course of radiation treatment decreases the severity of radiation-induced xerostomia. Bisphosphonates increase patient susceptibility to osteonecrosis. They do not decrease the severity of xerostomia in patients undergoing radiation treatment for oral cancer. Cyclosporine is an immunosuppressant drug used to prevent rejection of transplanted organs and can cause gingival enlargement. It does not decrease the severity of xerostomia in patients undergoing radiation treatment for oral cancer. Indomethacin is a nonsteroidal anti-inflammatory drug that can affect platelet function. Indomethacin does not decrease the severity of xerostomia in patients undergoing radiation treatment for oral cancer. REF: Radiation Therapy, page 290

OBJ: 21

35. Type A hemophilia is less common than type B hemophilia. Type A hemophilia is caused by a

deficiency of the clotting factor called plasma thromboplastinogen or factor VIII. a. Both statements are true. b. Both statements are false. c. The first statement is true; the second is false. d. The first statement is false; the second is true. ANS: D

The first statement is false; type A hemophilia is more common than type B hemophilia. The second statement is true; type A hemophilia is caused by a deficiency of the clotting factor called plasma thromboplastinogen or factor VIII. The first statement is false; type A hemophilia is more common than type B hemophilia. The second statement is true; type A hemophilia is caused by a deficiency of the clotting factor called plasma thromboplastinogen or factor VIII. The first statement is false; the second is true. REF: Types of Hemophilia, page 289

OBJ: 19

36. A reddish-blue or purple discoloration of the skin or mucosa that results from spontaneous

extravasation of blood and is caused by an increase in capillary fragility or a deficiency in blood platelets is termed a. hemophilia. b. Bruton disease. c. purpura. d. pernicious anemia. ANS: C

Purpura is a reddish-blue or purple discoloration of the skin or mucosa that results from spontaneous extravasation of blood and is caused by an increase in capillary fragility or a deficiency in blood platelets. Hemophilia is a disorder of blood coagulation that results in severely prolonged clotting time. Bruton disease is a disorder in which B cells do not mature and plasma cells are deficient. Pernicious anemia is a vitamin B12 deficiency that is caused by a deficiency of intrinsic factor, a substance secreted by the parietal cells of the stomach. REF: Purpura, page 288

OBJ: 18

37. A malignant neoplasm of the hematopoietic stem cells and characterized by an excessive

number of abnormal white blood cells is termed a. leukemia. b. hemophilia.


c. thrombocytopenic purpura. d. DiGeorge syndrome. ANS: A

Leukemia is a malignant neoplasm of the hematopoietic stem cells characterized by an excessive number of abnormal white blood cells. Hemophilia is a disorder of blood coagulation that results in severely prolonged clotting time. Thrombocytopenic purpura is a bleeding disorder that results from a severe reduction in circulating platelets. DiGeorge syndrome is a disorder in which the thymus is deficient or lacking. REF: Leukemia, page 285

OBJ: 15

38. Which is not a chronic disorder associated with sensitivity to dietary gluten? a. Celiac disease b. Gluten-sensitive enteropathy c. Celiac sprue d. Sickle cell trait ANS: D

Sickle cell trait is an inherited disorder of blood and not a chronic disorder associated with sensitivity to dietary gluten. Celiac disease is a chronic disorder associated with sensitivity to dietary gluten. Gluten-sensitive enteropathy is another name for celiac disease and is a chronic disorder associated with sensitivity to dietary gluten. Celiac sprue is another name for celiac disease and is a chronic disorder associated with sensitivity to dietary gluten. REF: Sickle Cell Anemia, page 282

OBJ: 10

39. In aplastic anemia, patients experience a dramatic decrease in all the circulating blood cells

because of a severe depression of bone marrow activity. a. Both the statement and reason are correct and related. b. Both the statement and reason are correct but not related. c. The statement is correct, but the reason is not. d. The statement is not correct, but the reason is correct. e. Neither the statement nor the reason is correct. ANS: A

Both the statement and reason are correct and related. In aplastic anemia, patients experience a dramatic decrease in all the circulating blood cells because of a severe depression of bone marrow activity. Both the statement and reason are correct and related. Both the statement and reason are correct and related. Both the statement and reason are correct and related. Both the statement and reason are correct and related. REF: Aplastic Anemia, page 283

OBJ: 11

40. The recommended treatment for folic acid and vitamin B12 deficiency anemias is a. surgery. b. dietary supplements. c. medications. d. radiation therapy. ANS: B


Dietary supplements of folic acid and vitamin B12 are recommended for these anemias. Surgery is not the recommended treatment for folic acid and vitamin B12 deficiency anemias. Medications are not the recommended treatment for folic acid and vitamin B12 deficiency anemias. Radiation therapy is not the recommended treatment for folic acid and vitamin B12 deficiency anemias. REF: Folic Acid and Vitamin B12 Deficiency Anemia, Diagnosis and Treatment, page 281 OBJ: 8 41. All statements are true regarding type 2 diabetes mellitus except one. Which is the exception? a. It is non-insulin–dependent. b. The majority of patients are diagnosed with type 2 diabetes. c. Contributing factors include a decrease in metabolic rate and weight gain. d. Complications include polydipsia, polyuria, and polyphagia. ANS: D

Type 1 diabetes can be complicated by polydipsia (excessive thirst and intake of fluid), polyuria (excessive urination), and polyphagia (excessive appetite). These complications are not commonly found in type 2 diabetes. It is also referred to as non-insulin–dependent diabetes mellitus. Approximately 95% of all diabetic patients have type 2 diabetes. Contributing factors include a decrease in metabolic rate and weight gain. REF: Non–Insulin-Dependent Diabetes Mellitus, page 276

OBJ: 6

42. Which vitamin is needed for deoxyribonucleic acid (DNA) synthesis and, when lacking,

affects the development of rapidly dividing cells in bone marrow and epithelium? a. Thiamin b. Vitamin B12 c. Riboflavin d. Vitamin C ANS: B

Vitamin B12 is needed for deoxyribonucleic acid (DNA) synthesis and, when lacking, affects the development of rapidly dividing cells in bone marrow and epithelium. Thiamin is needed for energy metabolism and is important in nerve function. Riboflavin is needed for energy metabolism and is important for vision and skin health. Vitamin C is an antioxidant. It is needed for protein metabolism and is important for immune system health and iron absorption. REF: Folic Acid and Vitamin B12 Deficiency Anemia, page 281 OBJ: 8 43. Elevated blood levels of calcium, low levels of blood phosphorus, and abnormal bone

metabolism are characteristic of which endocrine disorder? a. Hyperpituitarism b. Hyperthyroidism c. Hypothyroidism d. Hyperparathyroidism ANS: D


Elevated blood levels of calcium, low levels of blood phosphorus, and abnormal bone metabolism are all characteristics of hyperparathyroidism. Hyperpituitarism is characterized by excess hormone production by the anterior pituitary gland. The diagnosis involves measurement of serum growth hormones after ingestion of glucose. Hyperthyroidism is characterized by excessive production of thyroid hormone. Characteristics of hypothyroidism include a decreased output of thyroid hormone. REF: Hyperparathyroidism, page 273

OBJ: 4

44. Which type of hyperparathyroidism is the result of an overproduction of parathyroid hormone

(PTH) in response to long-term decreased levels of serum calcium often associated with chronic renal disease? a. Primary hyperparathyroidism b. Secondary hyperparathyroidism c. Tertiary hyperparathyroidism d. Familial hyperparathyroidism ANS: B

Secondary hyperparathyroidism is the result of an overproduction of PTH in response to longterm decreased levels of serum calcium often associated with chronic renal disease. Primary hyperparathyroidism may be the result of hyperplasia of the parathyroid glands, a benign tumor of one or more of the parathyroid glands, or a malignant parathyroid tumor. It is not the result of an overproduction of PTH in response to long-term decreased levels of serum calcium often associated with chronic renal disease. Tertiary hyperparathyroidism is not a type of hyperparathyroidism. Familial hyperparathyroidism is not a designated type of hyperparathyroidism. However, studies show that some hyperparathyroidism may be familial. REF: Hyperparathyroidism, page 273

OBJ: 4

45. Myxedema is caused by _____________ of the ____________ gland. a. hypersecretion; thyroid b. hyposecretion; thyroid c. hypersecretion; adrenal d. hyposecretion; adrenal ANS: B

When hypothyroidism occurs in older children and adults, it is referred to as myxedema. Myx-edema is not caused by an excessive production of the thyroid gland. Myxedema is caused by hyposecretion of the thyroid gland, not hypersecretion of the adrenal gland. Myxedema is not a function of the adrenal gland; it is caused by hyposecretion of the thyroid gland. REF: Hypothyroidism (Cretinism, Myxedema), page 273

OBJ: 3

46. Diabetes is characterized by abnormally high blood _________ levels. a. phosphorus b. calcium c. glucose d. neutrophil ANS: C


Diabetes is characterized by abnormally high blood glucose levels. These high levels result from a lack of the hormone insulin, defective insulin that does not work effectively to lower blood glucose levels, or increased insulin resistance because of obesity. Diabetes is characterized by abnormally high blood glucose and not phosphorus levels. Hyperparathyroidism is characterized by elevated levels of blood calcium and low levels of blood phosphorus. Diabetes is characterized by abnormally high blood glucose and not calcium levels. Diabetes is characterized by abnormally high blood glucose and not neutrophil levels. Leukemia is characterized by an overproduction of atypical white blood cells. REF: Diabetes Mellitus, page 274

OBJ: 6

47. ______________ is characterized by insufficient production of adrenal steroids. a. Addison disease b. Addison syndrome c. Cushing disease d. Cushing syndrome ANS: A

Addison disease is characterized by insufficient production of adrenal steroids. Addison disease, rather than Addison syndrome, is characterized by insufficient production of adrenal steroids. Addison syndrome does not exist. Cushing syndrome, rather than Cushing disease, is characterized by excess production of adrenal steroids. Cushing syndrome rather than Cushing disease is characterized by excess production of adrenal steroids. REF: Addison Disease (Primary Hypoadrenocorticism), page 278 OBJ: 7 48. The production of adrenal steroids is normally stimulated by release of ___________

hormone from the pituitary gland. a. follicle-stimulating b. luteinizing c. adrenocorticotropic d. growth ANS: C

The production of adrenal steroids is normally stimulated by release of adrenocorticotropic hormone from the pituitary gland. As a result of decreased production of adrenal steroids caused by Addison disease, the pituitary gland increases its production of adrenocorticotropic hormone. This hormone is similar to melanin-stimulating hormone and causes stimulation of melanocytes. Brown pigmentation (bronzing) of the skin occurs, and melanotic macules can appear on the oral mucosa. Follicle-stimulating hormone and luteinizing hormone stimulate the release of estrogen and progesterone in females and testosterone in males. Growth hormone is also released by the pituitary gland. REF: Addison Disease (Primary Hypoadrenocorticism), Clinical Features and Oral Manifestations, page 279 OBJ: 8 49. Plummer–Vinson syndrome can develop as a result of long-standing __________ anemia. a. pernicious b. iron-deficiency c. thalassemia


d. aplastic ANS: B

Plummer–Vinson syndrome can develop as a result of long-standing iron deficiency anemia. This syndrome includes dysphagia, atrophy of the upper alimentary tract, and a predisposition to the development of oral cancer. Pernicious anemia is a vitamin B12 deficiency caused by a deficiency of intrinsic factor, which is secreted by the parietal cells of the stomach. Thalassemia is the name of a group of inherited disorders of hemoglobin synthesis. Aplastic anemia results in a dramatic decrease in all the circulating blood cells because of a severe depression of bone marrow activity. REF: Iron Deficiency Anemia, page 280

OBJ: 8

50. Iron deficiency anemia may first result in loss of __________ papillae on the dorsum of the

tongue because these papillae have the highest metabolic requirement. a. filiform b. fungiform c. circumvallate d. foliate ANS: A

Iron deficiency anemia may first result in loss of filiform papillae on the dorsum of the tongue because these papillae have the highest metabolic requirement. Disappearance of the fungiform papillae can also occur in chronic and severe cases. Disappearance of the fungiform papillae can also occur in chronic and severe cases; however, the filiform papillae on the dorsum of the tongue disappear first because these papillae have the highest metabolic requirement. Iron deficiency anemia may first result in loss of filiform papillae on the dorsum of the tongue rather than circumvallate papillae because filiform papillae have the highest metabolic requirement. Iron deficiency anemia may first result in loss of filiform papillae on the dorsum of the tongue rather than foliate papillae because filiform papillae have the highest metabolic requirement. REF: Iron Deficiency Anemia, Clinical Features and Oral Manifestations, page 280 OBJ: 8 51. The red blood cells appear __________ and __________ in cases of iron deficiency anemia. a. microcytic; hypochromic b. microcytic; hyperchromic c. macrocytic; hypochromic d. macrocytic; hyperchromic ANS: A

The red blood cells appear microcytic and hypochromic in cases of iron deficiency anemia. Microcytic means smaller than normal, and hypochromic means lighter in color than normal. The red blood cells appear microcytic and hypochromic rather than hyperchromic in cases of iron deficiency anemia. The red blood cells appear microcytic rather than macrocytic and hypochromic in cases of iron deficiency anemia. The red blood cells appear microcytic rather than macrocytic and hypochromic rather than hyperchromic in cases of iron deficiency anemia. REF: Iron Deficiency Anemia, Diagnosis and Treatment, page 280


OBJ: 8 52. The Schilling test detects an inability to absorb an oral dose of vitamin _____. a. B6 b. B12 c. C d. E ANS: B

The Schilling test detects an inability to absorb an oral dose of vitamin B12. It may be used in the diagnosis of pernicious anemia. Diagnostic features of pernicious anemia include low serum vitamin B12 levels and gastric achlorhydria. The treatment of pernicious anemia consists of injections of vitamin B12. The oral mucosa improves in time, but the papillae on the dorsum of the tongue may not completely regenerate. The Schilling test detects an inability to absorb an oral dose of vitamin B12 rather than vitamin B6. The Schilling test detects an inability to absorb an oral dose of vitamin B12 rather than vitamin C. The Schilling test detects an inability to absorb an oral dose of vitamin B12 rather than vitamin E. REF: Pernicious Anemia, Diagnosis and Treatment, page 281

OBJ: 8

53. A platelet count of less than ________/mm3 is considered thrombocytopenia. a. 20,000 b. 100,000 c. 150,000 d. 400,000 ANS: B

A platelet count of less than 100,000/mm3 is considered thrombocytopenia. Spontaneous gingival bleeding can occur when a patient’s platelet count is less than 20,000/mm3. A normal platelet count should be 150,000 to 400,000/mm3. A platelet count of less than 100,000/mm3 rather than 20,000/mm3 is considered thrombocytopenia. A platelet count of less than 100,000/ mm3 rather than 150,000/mm3 is considered thrombocytopenia. REF: Platelet Count, page 287

OBJ: 16

54. ___________________ is an autosomal-dominant inherited disorder of platelet function. a. Bruton disease b. von Willebrand disease c. DiGeorge syndrome d. Hemophilia ANS: B

von Willebrand disease is an autosomal-dominant inherited disorder of platelet function. Bruton disease is a disorder in which B cells do not mature. DiGeorge syndrome is a disorder in which the thymus is lacking and T cells do not mature. Hemophilia is a disorder of blood coagulation that results in severely prolonged clotting time. REF: Nonthrombocytopenic Purpura, page 289

OBJ: 18

55. Type A hemophilia is caused by a deficiency of factor _____. a. VII


b. VIII c. IX d. X ANS: B

Type A hemophilia is caused by a deficiency of factor VIII, the clotting factor called plasma thromboplastinogen. This deficiency is characterized by severe hemorrhage after even mildto-moderate injury or surgery. Type A hemophilia is caused by a deficiency of factor VIII, the clotting factor called plasma thromboplastinogen, rather than factor VII, which is proconvertin. Type B hemophilia, or Christmas disease, is less common than type A hemophilia. The clotting defect is in plasma thromboplastin, or factor IX. Type A hemophilia is caused by a deficiency of factor VIII, the clotting factor called plasma thromboplastinogen, rather than factor X, which is Stuart factor. REF: Types of Hemophilia, page 289

OBJ: 19

56. Which term describes an opposite condition to anemia? a. Polycythemia b. Arthralgia c. Hemolysis d. Neutropenia ANS: A

Anemia is the reduction of the number of red blood cells, and polycythemia is the increase in the total red blood cell mass in the blood. Arthralgia is severe pain in a joint. Hemolysis is the release of hemoglobin from red blood cells by destruction of the cells. Neutropenia is a decreased number of neutrophils in the blood. REF: Vocabulary, page 271

OBJ: 1

57. Which condition necessitates emergency intervention? a. Hemostasis b. Ketoacidosis c. Insulin shock d. Myalgia ANS: C

Insulin shock refers to a state of profound hypoglycemia (or low blood sugar) that necessitates emergency intervention. Hemostasis is the stoppage or cessation of bleeding. Ketoacidosis is the accumulation of acid in the body resulting from the accumulation of ketone bodies. Myalgia is a term to describe muscle pain. REF: Vocabulary, page 271

OBJ: 1

58. All terms describe a decrease in circulating red or white blood cells except one. Which is the

exception? a. Thrombocytopenia b. Anemia c. Hypochromic d. Neutropenia ANS: C


Hypochromic refers to being stained less intensely than normal; this term does not relate to a decrease in cells. Thrombocytopenia is a decrease in the number of platelets in circulating blood. Anemia is a reduction of the number of red blood cells to less than normal. Neutropenia is a decreased number of neutrophils in the blood. REF: Vocabulary, page 271

OBJ: 1

59. Which system consists of a group of integrated glands and cells that secrete hormones? a. Endocrine b. Viral c. Neoplastic d. Immune ANS: A

The endocrine system consists of a group of integrated glands and cells that secrete hormones. The viral system is involved with viruses. Neoplastic refers to new growth or the formation of tumors. The immune system does not secrete hormones. REF: Endocrine Disorders, page 272

OBJ: 1

60. All are clinical features of gigantism except one. Which is the exception? a. Headache b. Chronic fatigue c. Extreme height d. Gastrointestinal issues ANS: D

Gastrointestinal issues are not typically seen in cases of gigantism. Persons affected with gigantism experience headaches. Persons affected with gigantism experience chronic fatigue. Gigantism includes excessive growth of the overall skeleton, and individuals may be over 7 feet tall. REF: Hyperpituitarism, Clinical Features and Oral Manifestations, page 272 OBJ: 2 61. Persons with acromegaly may experience an enlargement of the maxillary sinuses that may

lead to what condition? a. Deeper voice b. Nasal septum deformities c. Perforation of the oral mucosa d. Maxillary molars requiring endodontic treatment ANS: A

An enlargement of the maxillary sinus causes a characteristically deep voice. Acromegaly may cause an enlargement of the nasal bones but not septum deformities. Perforation of the oral mucosa is not seen in persons with acromegaly. Maxillary molars do not require endodontic therapy in cases of acromegaly. REF: Hyperpituitarism, Clinical Features and Oral Manifestations, page 272 OBJ: 2 62. Children affected with hyperthyroidism may demonstrate which characteristic?


a. b. c. d.

Lack of energy Fatigue Hyperactivity Dry skin

ANS: C

Too much thyroid hormone is produced causing an increase in metabolism that may mimic hyperactivity, especially in children. Hyperthyroidism is characterized by excessive thyroid hormone production; a lack of energy is not seen. The increased thyroid hormone levels increase the patient metabolism; fatigue is not a characteristic finding. Dry skin is not related to hyperthyroidism. REF: Hyperthyroidism (Thyrotoxicosis, Graves Disease), page 272 OBJ: 3 63. Which condition may cause children to demonstrate premature exfoliation of the primary

dentition as well as premature eruption of the permanent teeth? a. Diabetes b. Hyperthyroidism c. Hypothyroidism d. Hyperparathyroidism ANS: B

Hyperthyroidism in children may lead to premature exfoliation of deciduous teeth and premature eruption of permanent teeth. Diabetes does not cause premature exfoliation or eruption of teeth. Hypothyroidism may cause delayed eruption of teeth. The chief oral manifestation of hyperparathyroidism is the appearance of well-defined unilocular and multilocular radiolucencies. REF: Hyperthyroidism (Thyrotoxicosis, Graves Disease), Clinical Features and Oral Manifestations, page 273 OBJ: 3 64. In infants, facial and oral changes in hypothyroidism include these characteristics except one.

Which is the exception? a. Thickened lips b. Delayed eruption of teeth c. Macroglossia d. Exophthalmos ANS: D

Exophthalmos is a significant clinical characteristic of hyperthyroidism. Thickened lips may be seen in infants with hypothyroidism. Delayed eruption of teeth may be seen as the child grows to ages 2 to 4. Macroglossia is another oral manifestation of hypothyroidism. REF: Hypothyroidism (Cretinism, Myxedema), page 273

OBJ: 3

65. Secondary hyperparathyroidism occurs when calcium is abnormally excreted by the liver. The

most common cause of secondary hyperparathyroidism is liver failure. a. Both statements are true. b. Both statements are false. c. The first statement is true; the second statement is false.


d. The first statement is false; the second statement is true. ANS: B

Secondary hyperparathyroidism occurs when calcium is abnormally excreted by the kidneys. The most common cause of secondary hyperparathyroidism is kidney failure. Both statements are false. REF: Hyperparathyroidism, page 273

OBJ: 3

66. Oral manifestations of a person with Addison disease include which finding? a. Intraoral melanotic macules b. Gingival hyperplasia c. Tooth mobility d. Macroglossia ANS: A

Stimulation of melanocytes occurs causing bronzing of the skin and melanotic macules on the oral mucosa. Addison disease is not associated with gingival hyperplasia. Tooth mobility is not a feature of Addison disease. Macroglossia is not an oral manifestation of Addison disease. REF: Addison Disease (Primary Hypoadrenocorticism), Clinical Features and Oral Manifestations, page 279 OBJ: 7 67. Identify a common nutritional deficiency associated with anemia: a. Calcium b. Phosphorus c. Iron d. Vitamin D ANS: C

Nutritional anemias occur when a necessary substance for the normal development of red blood cells is in limited supply in the bone marrow. Iron is a common deficiency. A calcium deficiency is not associated with anemia. Anemia is not associated with deficiency in phosphorus. A vitamin D deficiency is not related to anemia. REF: Anemia, page 280

OBJ: 9

68. Skin and mucosal pallor, angular cheilitis, and loss of filiform and fungiform papilla on the

tongue are characteristics of what condition? a. Type 1 diabetes b. Cretinism c. Graves’ disease d. Anemia ANS: D

Oral manifestations for anemia include skin and mucosal pallor, angular cheilitis, atrophy of the oral mucosa, and loss of filiform and fungiform papilla on the dorsum of the tongue. Type 1 diabetes does not exhibit these oral manifestations. Cretinism may cause thickened lips, enlarged tongue, and delayed eruption of teeth. Graves’ disease may cause premature exfoliation and eruption of teeth.


REF: Anemia, page 280

OBJ: 9

69. Physical characteristics of a person with type 1 diabetes include all except one. Which is the

exception? a. Peak age of 50 years b. Thin body build c. Polydipsia, polyuria, polyphagia d. Abrupt onset ANS: A

Type 1 diabetes can occur at any age, but its time of onset is usually at a peak age of 20 years. Patients usually have a thin body build. The primary signs of type 1 diabetes are the 3 P’s: polydipsia, polyuria, and polyphagia. The onset of type 1 diabetes is abrupt. REF: Insulin-Dependent Diabetes Mellitus, Pathology, and Clinical Management, page 275 OBJ: 6 70. When a patient is taking long-term antibiotic therapy, what is a common oral manifestation

that may be seen? a. Risk of candidiasis b. Xerostomia c. Tooth discoloration d. Gingival enlargement ANS: A

Antibiotics can increase the risk of candidiasis. Xerostomia may be seen in persons taking drugs to control blood pressure. Tooth discoloration is seen if a person has taken tetracycline during tooth development. Gingival enlargement may be seen in an epileptic patient or one who has experienced a transplanted organ. REF: Effects of Drugs on the Oral Cavity, page 291

OBJ: 22

MATCHING

For each vocabulary word below, select the corresponding definition from the list provided. a. Severe pain in a joint b. Reduction of the number of red blood cells c. Chronic excessive thirst and intake of fluid d. Small, flat, hemorrhagic patch larger than a petechia on the skin e. Cell surface protein to which a specific molecule (e.g., a hormone) can bind f. Stoppage or cessation of bleeding g. Decrease in number of granulocytes h. Muscle pain i. Insoluble protein that is essential for the clotting of blood j. Platelet k. Enlargement of the liver l. Increase in the total red blood cell mass in the blood m. Abnormal rarefaction of bone n. Decrease in the number of platelets in circulating blood 1. Hepatomegaly


2. 3. 4. 5. 6. 7. 8. 9. 10. 11. 12. 13. 14.

Myalgia Polydipsia Thrombocyte or mucous membrane Osteoporosis Polycythemia Ecchymosis Receptor Fibrin Anemia Agranulocytosis Thrombocytopenia Arthralgia Hemostasis

1. 2. 3. 4. 5. 6. 7. 8. 9. 10. 11. 12. 13. 14.

ANS: K ANS: H ANS: C ANS: J ANS: M ANS: L ANS: D ANS: E ANS: I ANS: B ANS: G ANS: N ANS: A ANS: F

REF: REF: REF: REF: REF: REF: REF: REF: REF: REF: REF: REF: REF: REF:

Vocabulary, page 271 Vocabulary, page 271 Vocabulary, page 271 Vocabulary, page 271 Vocabulary, page 271 Vocabulary, page 271 Vocabulary, page 271 Vocabulary, page 271 Vocabulary, page 271 Vocabulary, page 271 Vocabulary, page 270 Vocabulary, page 271 Vocabulary, page 270 Vocabulary, page 271

OBJ: OBJ: OBJ: OBJ: OBJ: OBJ: OBJ: OBJ: OBJ: OBJ: OBJ: OBJ: OBJ: OBJ:

1 1 1 1 1 1 1 1 1 1 1 1 1 1


Chapter 10: Orofacial Pain and Diseases Affecting the Temporomandibular Joint Ibsen: Oral Pathology for the Dental Hygienist, 7th Edition MULTIPLE CHOICE 1. The temporomandibular joint (TMJ) is the articulation between the condyle of the mandible

and the a. zygomatic process of the temporal bone. b. glenoid fossa of the temporal bone. c. external acoustic meatus. d. spine of the sphenoid bone. ANS: B

The TMJ is the articulation between the condyle of the mandible and the glenoid fossa of the temporal bone. It is a highly specialized joint that differs from other joints because of the fibrocartilage that covers the bony articulating surfaces, its ginglymoarthrodial movement, its function and overall health being dictated by jaw movement, and its dependence on the contralateral joint. The TMJ is the articulation between the condyle of the mandible and the glenoid fossa of the temporal bone rather than the zygomatic process of the temporal bone. The TMJ is the articulation between the condyle of the mandible and the glenoid fossa of the temporal bone rather than the external acoustic meatus. The TMJ is the articulation between the condyle of the mandible and the glenoid fossa of the temporal bone rather than the spine of the sphenoid bone. REF: Anatomy of the Temporomandibular Joint, page 306

OBJ: 1

2. Which statement about the articular disc in the TMJ joint is true? a. It is attached to the anterior and posterior aspects of the condyle. b. It is attached to the inferior belly of the medial pterygoid muscle. c. The disc is highly vascular. d. The disc is surrounded and protected by the fibrous connective tissue joint capsule. ANS: D

The disc is surrounded and protected by the fibrous connective tissue joint capsule. The articular disc in the TMJ is attached to the lateral and medial rather than the anterior and posterior aspects of the condyle, to the superior belly of the lateral pterygoid muscle, and to the joint capsule. The articular disc in the TMJ is attached to the lateral and medial aspects of the condyle, to the superior rather than the inferior belly of the lateral pterygoid muscle, and to the joint capsule. The disc and the bony surfaces are avascular. REF: Anatomy of the Temporomandibular Joint, page 307

OBJ: 1

3. During opening of the jaw a. rotational movement precedes translation. b. rotational movement follows translation. c. only rotational movement occurs; translation occurs only during closure. d. only translational movement occurs; rotation occurs only during closure. ANS: A


During opening of the jaw, rotational movement precedes translation. During translation the disc assumes a more posterior position in relation to the condyle. The inferior and superior joint spaces assume different configurations during each of these movements. During opening of the jaw, rotational movement precedes rather than follows translation. REF: Normal Function of the Temporomandibular Joint, page 308 OBJ: 1 4. Which choice represents an indirect form of trauma affecting the TMJ? a. Clenching, bruxism b. Whiplash injury c. Psychosocial factors d. Osteoarthritis ANS: B

Whiplash injury represents an indirect form of trauma affecting the TMJ. Trauma affecting the TMJ is classified as direct (assault), indirect (whiplash injury), or secondary to parafunctional habits (clenching, bruxism). Other contributing factors include dentofacial deformities and psychosocial factors. TMJ abnormalities are also associated with a number of different systemic diseases. Rheumatoid arthritis and osteoarthritis are the most common. Whiplash injury represents an indirect form of trauma affecting the TMJ. Trauma affecting the TMJ is classified as secondary to parafunctional habits when it is caused by clenching or bruxism. REF: Pathophysiology of Temporomandibular Disorders, page 309 OBJ: 4 5. Clicking and popping most commonly reflect a. parafunctional habits such as bruxism and clenching. b. disc displacement with reduction. c. malocclusion. d. trismus. ANS: B

Clicking and popping commonly reflect disc displacement with reduction. It occurs in approximately 33% of asymptomatic patients. It is of little clinical consequence in the absence of pain or other symptoms relating to the TMJ because it is controversial as to whether these noises represent an adaptive response or an early symptom of progression to disc displacement without reduction. Clicking and popping, not parafunctional habits, commonly reflect disc displacement with reduction. Clicking and popping, not malocclusion, commonly reflect disc displacement with reduction. Clicking and popping, not trismus, commonly reflect disc displacement with reduction. REF: History, page 309

OBJ: 5

6. Which dental radiographic procedure is best for examining disc position, function, form, and

the presence of joint effusions? a. Panoramic imaging b. Transcranial imaging c. Computerized tomography (CT) d. Magnetic resonance imaging (MRI)


ANS: D

MRI is mandatory for examining disc position, function, and form and the presence of joint effusions. Arthrography may be useful when MRI is not tolerated and information regarding the position and morphology of the disc is required. Panoramic imaging radiographs are limited to identifying gross changes in bone. Transcranial imaging radiographs are limited to identifying gross changes in bone. CT is the most accurate for identifying bone abnormalities. REF: Imaging, page 311

OBJ: 6

7. An open lock of the TMJ that the patient cannot reduce is termed a. subluxation. b. dislocation. c. ankylosis. d. myofascial pain. ANS: B

An open lock that the patient cannot reduce is called dislocation. Dislocation occurs when one or both of the condyles translate anterior to the articular eminence. Subluxation refers to hypermobility in which the patient is able to relocate the mandible back into the glenoid fossa. Ankylosis of the TMJ is defined as immobility of the condyle because of fibrous or bony union between the articulating structures of the joint. Myofascial pain is characterized as a dysfunctional muscle hyperactivity with regional pain, tenderness of the affected muscles, and variable amounts of reduced opening and complaints of malocclusion. REF: Hypermobility Disorders, page 312

OBJ: 7

8. Which statement is characteristic of a patient with osteoarthritis? a. It is characterized by pain that is worse in the morning. b. It is characterized by pain that is worse in the evening. c. It is an inflammatory, autoimmune disorder of the joints. d. Approximately 50% to 75% of patients have involvement of the TMJ during the

course of the disease. ANS: B

Osteoarthritis is characterized by pain that is worse in the evening, as well as limited opening, muscle splinting, and crepitus of the TMJ. Osteoarthritis is the most common disease affecting the TMJ. In rheumatoid arthritis patients may complain of pain that is worse in the morning, limited opening, occlusal changes, and preauricular edema and tenderness. Rheumatoid arthritis is an inflammatory, autoimmune disorder of the joints. Approximately 50% to 75% of patients with rheumatoid arthritis have involvement of the TMJ during the course of the disease. Osteoarthritis is characterized by pain that is worse in the evening rather than in the morning. Rheumatoid arthritis rather than osteoarthritis is an inflammatory, autoimmune disorder of the joints. Approximately 50% to 75% of patients with rheumatoid arthritis rather than osteoarthritis have involvement of the TMJ during the course of the disease. REF: Arthritis, page 312

OBJ: 7

9. The purpose of the synovial fluid is to a. provide nourishment and lubrication. b. divide the space into upper and lower compartments. c. govern mandibular movement.


d. separate forces resulting from movement. ANS: A

The purpose of the synovial fluid is to provide nourishment and lubrication of the avascular structures to the TMJ. An articular disc divides the space into an upper and lower compartment. The muscles of mastication govern the movement of the mandible. The function of the articular disc is to separate forces resulting from movement. REF: Anatomy of the Temporomandibular Joint, page 307

OBJ: 1

10. Trauma affecting the TMJ is classified as direct, indirect, and secondary to parafunctional

habits. Which is an example of a parafunctional habit? a. Whiplash injury b. Assault c. Surgery d. Clenching ANS: D

Clenching and bruxism are examples of parafunctional habits affecting the TMJ. Whiplash injury is an example of indirect trauma to the TMJ. Assault is an example of direct trauma to the TMJ. Surgery is not an example or direct trauma, indirect trauma, or parafunctional habits. REF: Pathophysiology of Temporomandibular Disorders, page 309 OBJ: 4 11. The most common systemic conditions that affect the TMJ are a. Sjögren syndrome and scleroderma. b. rheumatoid arthritis and juvenile rheumatoid arthritis. c. gout and Reiter syndrome. d. ankylosing spondylitis and mixed connective tissue disease. ANS: B

Rheumatoid arthritis and juvenile rheumatoid arthritis are the most common systemic conditions that affect the TMJ. Sjögren syndrome and scleroderma are less likely to affect the TMJ than are rheumatoid arthritis and juvenile rheumatoid arthritis. Gout and Reiter syndrome are less likely to affect the TMJ than are rheumatoid arthritis and juvenile rheumatoid arthritis. Ankylosing spondylitis and mixed connective tissue disease are less likely to affect the TMJ than are rheumatoid arthritis and juvenile rheumatoid arthritis. REF: Pathophysiology of Temporomandibular Disorders, page 309 OBJ: 4 12. Temporomandibular dysfunction (TMD) can be caused by disorders of the muscles of

mastication or by internal derangements of the components of the joint. Three cardinal features that suggest TMD include orofacial pain, joint noise, and restricted jaw function. a. Both statements are true. b. Both statements are false. c. The first statement is true; the second is false. d. The first statement is false; the second is true. ANS: A


Both statements are true. TMD can be caused by disorders of the muscles of mastication or by internal derangements of the components of the joint. Three cardinal features that suggest a TMD include orofacial pain, joint noise, and restricted jaw function. REF: Evaluation of Temporomandibular Disorders, page 309

OBJ: 4 | 5

13. Tenderness over the lateral pole of the condyle is indicative of a. crepitus. b. capsulitis. c. trismus. d. auscultation. ANS: B

Capsulitis is tenderness over the lateral pole of the condyle. Crepitus is a dry crackling sound; tenderness over the lateral pole of the condyle is not indicative of crepitus. Trismus is the inability to fully open the mouth. Auscultation refers to listening to sounds within the body, using a stethoscope. REF: Clinical Examination, page 309

OBJ: 4

14. Caution is recommended with long-term use of nonsteroidal anti-inflammatory drugs

(NSAIDs) because of what complications? a. Gastrointestinal b. Cardiovascular c. Psychiatric d. Arthritis ANS: B

Caution is recommended with long-term use of NSAIDs because of cardiovascular complications. No direct correlation exists between long-term use of NSAIDs and gastrointestinal complications. There is no direct correlation between long-term use of NSAIDs and psychiatric complications. There is no direct correlation between long-term use of NSAIDs and arthritic complications. REF: Nonsurgical Treatment, page 313

OBJ: 8

15. All are benign tumors that arise in the condyle except one. Which is the exception? a. Osteoblastoma b. Synovial chondromatosis c. Osteoma d. Chondroblastoma ANS: B

Synovial chondromatosis is the most common benign neoplasm of the synovium, not the condyle. The three most common benign tumors of the condyle are the osteoblastoma, osteoma, and chondroblastoma. REF: Neoplasia, page 313

OBJ: 9

16. Which dental radiographic imaging is best for identifying bone abnormalities such as

osteophytes, condylar erosion, fractures, ankylosis, and condylar hyperplasia? a. Panoramic imaging


b. Transcranial imaging c. Computerized tomography (CT) d. Magnetic resonance imaging (MRI) ANS: C

CT is the most accurate for identifying bone abnormalities such as osteophytes, condylar erosion, fractures, ankylosis, and condylar hyperplasia. Panoramic imaging radiographs are limited to identifying gross changes in bone. Transcranial imaging radiographs are limited to identifying gross changes in bone. MRI is used for examining disc position, function, and form and the presence of joint effusions. REF: Imaging, page 310

OBJ: 6

17. Hypermobility in which the patient is able to relocate the mandible back into the glenoid fossa

is referred to as a. subluxation. b. dislocation. c. ankylosis. d. myofascial pain. ANS: A

Subluxation refers to hypermobility in which the patient is able to relocate the mandible back into the glenoid fossa. Dislocation occurs when one or both of the condyles translate anterior to the articular eminence, resulting in an open lock that the patient cannot reduce. Ankylosis of the TMJ is defined as immobility of the condyle because of fibrous or bony union between the articulating structures of the joint. Myofascial pain is characterized as a dysfunctional muscle hyperactivity with regional pain, tenderness of the affected muscles, and variable amounts of reduced opening and complaints of malocclusion. REF: Hypermobility Disorders, page 312

OBJ: 7

18. The TMJ exhibits ginglymoarthrodial, or __________, movement. a. rotational and translational b. opening c. closing d. maximal interdigitation ANS: A

The TMJ exhibits ginglymoarthrodial, or rotational and translational, movement. Translational movement occurs in the upper compartment, whereas the lower compartment functions primarily as the hinge or rotational component. The first phase of opening movement is characterized by rotational movement of the condyle followed by anterior translation to approximately the anterior peak of the articular eminence. The opening is accomplished mainly by the lateral pterygoid muscle, with some help from the anterior digastric muscle. The masseter, medial pterygoid, and temporalis are elevator muscles that close the mandible when activated. The point of maximal interdigitation is called centric occlusion. REF: Anatomy of the Temporomandibular Joint, page 307

OBJ: 1

19. Opening, or depressor function, of the mandible is accomplished primarily by the

_____________ muscle.


a. b. c. d.

medial pterygoid lateral pterygoid masseter temporalis

ANS: B

Opening, or depressor function, of the mandible is accomplished primarily by the lateral pterygoid muscle with some help from the anterior digastric muscle. The medial pterygoid is an elevator muscle that closes the mandible when activated. The masseter is an elevator muscle that closes the mandible when activated. The temporalis is an elevator muscle that closes the mandible when activated. REF: Anatomy of the Temporomandibular Joint, page 307

OBJ: 2

20. While up to _____% of the adult population has at least one sign and/or symptom of a

temporomandibular disorder (TMD), most studies suggest that clinically significant TMDrelated jaw pain, dysfunction, or both affects about _____% of the general population. a. 90; 10 b. 75; 13 c. 50; 10 d. 25; 5 ANS: B

Although up to 75% of the adult population has at least one sign and/or symptom of a temporomandibular disorder, most studies suggest that clinically significant TMD-related jaw pain, dysfunction, or both affects about 13% of the general population. Significantly more frequent and more severe TMD signs and symptoms are seen in women than in men and older adults. Although up to 75%, not 90%, of the adult population has at least one sign and/or symptom of a temporomandibular disorder, most studies suggest that clinically significant TMD-related jaw pain, dysfunction, or both affects about 5% rather than 10% of the general population. Although up to 75% and not 50% of the adult population has at least one sign and/ or symptom of a temporomandibular disorder, most studies suggest that clinically significant TMD-related jaw pain, dysfunction, or both affects about 5% rather than 10% of the general population. Although up to 75% and not 25% of the adult population has at least one sign and/ or symptom of a temporomandibular disorder, most studies suggest that clinically significant TMD-related jaw pain, dysfunction, or both affects about 5% of the general population. REF: Epidemiology of Temporomandibular Disorders, page 309 OBJ: 5 21. ____________ ankylosis is the most common type seen in the TMJ. a. Fibrotic intraarticular b. Bony intraarticular c. Fibrotic extraarticular d. Bony extraarticular ANS: A


Fibrotic intraarticular ankylosis is the most common type seen in the TMJ. Ankylosis of the TMJ is defined as immobility of the condyle because of fibrous or bony union between the articulating structures of the joint. Ankylosis can be classified by tissue type (fibrous, bony), location (intraarticular, extraarticular), and extent of fusion (complete, incomplete). Fibrotic intraarticular ankylosis rather than bony intraarticular ankylosis is the most common type seen in the TMJ. Fibrotic intraarticular ankylosis rather than fibrotic extraarticular ankylosis is the most common type seen in the TMJ. Fibrotic intraarticular ankylosis rather than bony extraarticular ankylosis is the most common type seen in the TMJ. REF: Ankylosis, page 313

OBJ: 7

22. About _____% of all TMJ ankylosis cases result from joint infection, usually after trauma. a. 10 b. 30 c. 50 d. 70 ANS: C

About 50% of all TMJ ankylosis cases result from joint infection, usually after trauma, but 30% result from trauma without infection. About 50% rather than 10% of all TMJ ankylosis cases result from joint infection, usually after trauma. About 50% rather than 30% of all TMJ ankylosis cases result from joint infection, usually after trauma. About 50% rather than 70% of all TMJ ankylosis cases result from joint infection, usually after trauma. REF: Ankylosis, page 313

OBJ: 7

23. Children are _____ prone to TMJ ankylosis than adults because of greater osteogenic potential

and _____ development of the joint meniscus. a. more; more b. more; less c. less; more d. less; less ANS: B

Children are more prone to TMJ ankylosis than adults because of greater osteogenic potential and less development of the joint meniscus. Children are more prone to TMJ ankylosis than adults because of greater osteogenic potential and less rather than more development of the joint meniscus. Children are more rather than less prone to TMJ ankylosis than adults because of greater osteogenic potential and less rather than more development of the joint meniscus. Children are more rather than less prone to TMJ ankylosis than adults because of greater osteogenic potential and less development of the joint meniscus. REF: Ankylosis, page 313

OBJ: 7

24. Of the various surgical techniques used to treat TMDs, __________ involves surgical

repositioning of the condyle. a. arthrocentesis b. arthroscopy c. condylotomy d. open joint surgery


ANS: C

Of the various surgical techniques used to treat TMDs, condylotomy involves surgical repositioning of the condyle. Arthrocentesis involves lavaging the joint through a needle. Arthroscopy allows direct visualization and manipulation of the joint. Open joint surgery is used to perform disc repositioning, replacement or excision, and total joint reconstruction using a prosthetic device or autogenous graft. REF: Surgical Treatment, page 314

OBJ: 8

25. Synovial _____________ is the most common benign neoplasm of the synovium. a. osteoblastoma b. chondroblastoma c. osteoma d. chondromatosis ANS: D

Synovial chondromatosis is the most common benign neoplasm of the synovium. It is characterized by the development of metaplastic, highly cellular cartilaginous foci in the synovial membrane that results in degenerative changes consistent with osteoarthritis. It is characterized by swelling, pain, and limitation of movement. Synovial chondromatosis and not osteoblastoma is the most common benign neoplasm of the synovium. Synovial chondromatosis and not chondroblastoma is the most common benign neoplasm of the synovium. Synovial chondromatosis and not osteoma is the most common benign neoplasm of the synovium. REF: Neoplasia, page 313

OBJ: 9

26. The three elevator muscles of mastication that when activated, close the mandible, include the

masseter, the medial pterygoid, and the ___________. a. lateral pterygoid b. temporalis c. anterior digastric d. mylohyoid ANS: B

The temporalis muscle is one of the three elevator muscles of mastication that close the mandible. The lateral pterygoid muscle opens, or depresses, the mandible. The anterior digastric muscle opens, or depresses, the mandible. The mylohyoid muscle opens, or depresses, the mandible. REF: Anatomy of the Temporomandibular Joint, page 307 27. Which term describes the inability to fully open the mouth? a. Trismus b. Articulation c. Fusion d. Crepitus ANS: A

OBJ: 2


Trismus is defined as the inability to fully open the mouth. Articulation is another term for joint. Fusion refers to the union of two adjoining tooth germs. Crepitus is the dry, crackling sound that may be associated with the temporomandibular (TMJ) joint. REF: Vocabulary, page 305

OBJ: 2

28. The TMJ is the articulation between the mandibular condyle and which other structure? a. Zygomatic process b. Sphenoid bone c. Glenoid fossa of the temporal bone d. Occipital bone ANS: C

The TMJ is the articulation between the condyle of the mandible and the glenoid fossa of the temporal bone. The zygomatic process is a portion of the maxilla. The sphenoid bone is a bone of the neurocranium, which sits in front of the temporal and occipital bones. The occipital bone is situated at the back and lower part of the skull. REF: Anatomy of the Temporomandibular Joint, page 306

OBJ: 1

29. Which muscle of mastication is most superior in its location? a. Temporalis b. Lateral pterygoid c. Masseter d. Medial pterygoid ANS: A

The temporalis muscle is a broad muscle that covers much of the temporal bone and is superior in location when compared to the other muscles of mastication. The lateral pterygoid is more inferior in location compared to the temporalis muscle, functioning to open the mandible. The masseter muscle is inferior to the temporalis muscle, functioning to close the mandible. The medial pterygoid is found on the medial side of the lateral pterygoid plate, functioning to close the mandible. REF: Anatomy of the Temporomandibular Joint, page 307

OBJ: 2

30. Which muscle of mastication is responsible for opening the mandible? a. Temporalis b. Medial pterygoid c. Masseter d. Lateral pterygoid ANS: D

The function of the lateral pterygoid muscle is the opening of the mandible. The function of the temporalis muscle is to close the mandible. The function of the medial pterygoid muscle is to close the mandible. The function of the masseter muscle is to close the mandible. REF: Anatomy of the Temporomandibular Joint, page 307

OBJ: 2

31. What is the demographic group that typically presents with temporomandibular disorders

(TMD)? a. Children


b. Females, age 20–40 c. Males, age 40 d. Senior citizens ANS: B

The majority of patients with TMDs are female aged between 20 and 40 years. Children do not typically present with TMDs. Up to 75% of the population has at least one sign of TMD, and the majority of patients are younger females. Senior citizens are not reported to demonstrate a high incidence of TMD. REF: Epidemiology of Temporomandibular Disorders, page 309 OBJ: 4 32. What is the most likely cause of TMD? a. Stress b. Trauma c. Malocclusion d. Third molar extractions ANS: B

Trauma has been suggested as the most likely cause of TMDs. Stress has been implicated in masticatory muscle pain, yet it is not considered the most likely cause of TMDs. Although it is a condition to be addressed, malocclusion is not the primary cause of TMDs. Sufficient evidence is not present to suggest that oral surgery procedures (such as third molar extraction) can cause TMDs. REF: Pathophysiology of Temporomandibular Disorders, page 309 OBJ: 4 33. What is the most common systemic condition that may affect the TMJ? a. Lupus b. Gout c. Arthritis d. Scleroderma ANS: C

The most common systemic conditions that may affect the TMJ are rheumatoid arthritis and juvenile rheumatoid arthritis. Less common systemic diseases that may affect the TMJ include systemic lupus erythematosus. Less common systemic diseases that may affect the TMJ include gout. Less common systemic diseases that may affect the TMJ include scleroderma. REF: Pathophysiology of Temporomandibular Disorders, page 309 OBJ: 5 34. Three cardinal features for TMD include all except one. Which is the exception? a. Restricted jaw function b. Orofacial pain c. Previous dental surgery d. Joint noise ANS: C


A history of previous dental surgery is not considered a cardinal feature of TMD. Three cardinal features suggest a TMD, and restricted jaw function is one of the three. Three cardinal features suggest a TMD, and orofacial pain is one of the three. Three cardinal features suggest a TMD, and joint noise is one of the three. REF: Evaluation of Temporomandibular Disorders, page 309

OBJ: 3

35. During a dental appointment, dental hygienists can help a patient with TMD by doing which

procedure? a. Set the patient up slowly after the appointment. b. Maintain a shorter appointment time. c. Schedule the patient the first time of the day. d. Avoid loud background music in the office. ANS: B

Keeping a shorter appointment time allows the patient time to relax in keeping their mouth open. Setting the patient up slowly may aid with persons with hypertension. Scheduling the patient with the first appointment of the day does not help patients with TMD. Avoiding loud background music has no effect on patients with TMD. REF: Evaluation of Temporomandibular Disorders, page 309

OBJ: 4

36. From the choices below, which type of radiographic imaging would be least helpful in

diagnosing TMJ disease? a. MRI b. CBCT c. Panoramic imaging d. Arthrography ANS: C

Of the choices listed, panoramic imaging is limited to identifying gross changes in bone, which is not often seen until a significant volume of destruction has occurred. MRI is mandatory for examining disc position, function, and morphology. CBCT is advantageous because of its accessibility, higher spatial resolution, reduced radiation exposure, and faster imaging. Arthrography uses a radiopaque contrast agent that is injected into the joint and may be useful when MRI is not tolerated and information about the position and morphology of the disc is needed. REF: Imaging, pages 310-311

OBJ: 6

37. Tumors arising in the TMJ area are common. The most common benign tumors that arise in

the condyle include the osteochondroma. a. Both statements are true. b. Both statements are false. c. The first statement is true; the second statement is false. d. The first statement is false; the second statement is true. ANS: D


Tumors arising in the TMJ area are rare. The most common benign tumors that arise in the condyle include the osteochondroma, osteoblastoma, chondroblastoma, and osteoma. Tumors arising in the TMJ area are rare. The most common benign tumors that arise in the condyle include the osteochondroma. The first statement is false; the second statement is true. REF: Neoplasia, page 313

OBJ: 9

38. Which term describes the condition where one or both of the condyles translate anterior to the

articular eminence, resulting in an open lock that the patient cannot reduce? a. Dislocation b. Degenerative c. Displacement d. Odontogenic ANS: A

Dislocation occurs when one or both of the condyles translate anterior to the articular eminence, resulting in an open lock that the patient cannot reduce. Degenerative denotes a progressive or irreversible deterioration. Displacement signifies moving something from its place or position. Odontogenic refers to arising from tooth-forming tissues. REF: Hypermobility Disorders, page 312

OBJ: 4

39. Approximately what percentage of persons with rheumatoid arthritis will have involvement of

the TMJ? a. <10% b. 20–30% c. 50–75% d. Nearly 100% ANS: C

Rheumatoid arthritis is an inflammatory autoimmune disorder of the joints. Approximately 50–75% of patients with rheumatoid arthritis have involvement of the TMJ during the course of the disease. REF: Arthritis, page 312

OBJ: 4

40. Which term describes a procedure induced inadvertently by a health care provider or by

medical treatment or a diagnostic procedure? a. Sign b. Symptom c. Iatrogenic d. Auscultation ANS: C

Iatrogenic is a term to describe the inadvertent induction by a health care provider or by medical treatment or diagnostic procedure. Iatrogenic causes of TMDs include the indiscriminate use of corticosteroid injection into the joint. A sign refers to objective evidence of disease that can be observed by a health care provider rather than by the patient. A symptom refers to subjective evidence of disease or a physical disorder that is observed by the patient. Auscultation is listening to sounds within the body.


REF: Vocabulary, page 304

OBJ: 2

41. What is the function of the synovial fluid inside the joint? a. To provide nourishment and lubrication of avascular structures b. To allow the bones to fit together properly c. To absorb shock d. To allow for correct palpation of the joint area ANS: A

The synovial fluid is produced by the synovial membrane that lines the joint. The fluid provides nourishment and lubrication of the avascular structures. The synovial fluid does not have an affect on the bones of the TMJ. The synovial fluid does not absorb shock to the joint. The synovial fluid does not allow for correct palpation of the joint area. REF: Anatomy of the Temporomandibular Joint, page 307

OBJ: 2

MATCHING

For each vocabulary word below, select the corresponding definition from the list provided. a. Method for evaluating and manipulating a joint via the insertion of a camera and instruments b. Dry, crackling sound c. Surgical puncture of a joint followed by lavage of joint space d. Physical examination using pressure of the hand or fingers e. Inability to fully open the mouth f. Radiography of a joint after injection of opaque contrast material g. Noninvasive diagnostic technique that uses radio waves to produce computerized images of internal body tissues h. Listening to sounds within the body, using a stethoscope i. Subjective evidence of disease or a physical disorder that is observed by the patient j. Objective evidence of disease that can be observed by a health care provider rather than by the patient k. Joint l. Induced inadvertently by a medical or dental care provider or by medical treatment or a diagnostic procedure 1. 2. 3. 4. 5. 6. 7. 8. 9. 10. 11. 12.

Symptom Trismus MRI Arthrocentesis Palpation Articulation Sign Crepitus Iatrogenic Auscultation Arthrography Arthroscopy


1. 2. 3. 4. 5. 6. 7. 8. 9. 10. 11. 12.

ANS: I ANS: E ANS: G ANS: C ANS: D ANS: K ANS: J ANS: B ANS: L ANS: H ANS: F ANS: A

REF: REF: REF: REF: REF: REF: REF: REF: REF: REF: REF: REF:

Vocabulary, page 305 Vocabulary, page 305 Vocabulary, page 304 Vocabulary, page 304 Vocabulary, page 304 Vocabulary, page 304 Vocabulary, page 305 Vocabulary, page 304 Vocabulary, page 304 Vocabulary, page 304 Vocabulary, page 304 Vocabulary, page 304

OBJ: OBJ: OBJ: OBJ: OBJ: OBJ: OBJ: OBJ: OBJ: OBJ: OBJ: OBJ:

5|6|8 5|6|8 5|6|8 5|6|8 5|6|8 5|6|8 5|6|8 5|6|8 5|6|8 5|6|8 5|6|8 5|6|8


Turn static files into dynamic content formats.

Create a flipbook
Issuu converts static files into: digital portfolios, online yearbooks, online catalogs, digital photo albums and more. Sign up and create your flipbook.